You are on page 1of 168

 

SBA 
OF  
ORTHO​PAEDICS  
(​ORTHO​BULLETS​) 
FOREWORD 3

PAEDIATRIC ORTHO 4
ANSWER: 20

SPORT ORTHO 33
ANSWER: 40

DIVISION OF JOINT RECONSTRUCTION 45


ANSWER: 51

TRAUMA 55
ANSWER: 80

ONCOLOGY 97
Answer: 109

FOOT AND ANKLE 117


ANSWER: 128

SPINE 135
ANSWER: 145

HAND 155
ANSWER: 162

2
FOREWORD

“Our greatest glory is not in never falling, but in rising every time we fall. “
~Confucius

3
PAEDIATRIC ORTHO
1. A 6-week-old female infant is referred to your practice for concerns of developmental dysplasia of the
hip. On physical exam, you note a positive Ortolani test on the left side. Pavlik harness treatment is
initiated. Which of the following imaging modalities should be utilized at the two week follow-up visit?
a.Magnetic Resonance Imaging (MRI)
b.Computed Tomography (CT)
c.Ultrasound (US)
d.Plain Radiographs
e.Arthrogram and Dynamic Fluroscopy

2. Which of the following concepts regarding pediatric hips is true?


A .The proximal femoral physis and greater trochanteric apophysis develop from different
cartilaginous physes
B. The proximal femoral physis grows at a rate of 9 mm per year
C. Normal infant femoral anteversion is between 10-20 degrees
D. The ossific nucleus of the proximal femur is visible on radiographs by 6 months of age in most
children
E. ​Slipped capital femoral epiphysis (SCFE) typically occurs through the zone of proliferatio​n

3. Failure to achieve reduction of a dislocated hip in an otherwise healthy 4 month old infant which did
not reduce after 3 weeks in a Pavlik harness and 3 weeks in an abduction brace is best treated with
which of the following?
A. Adjusting the harness to 75 degrees of abduction and maintaing 90 degrees of hip flexion
B. Adjusting the harness to 75 degrees of abduction and increasing hip flexion to 120 degrees
C. Closed reduction with hip arthrogram, adductor tenotomy if necessary, and hip spica casting
D. Open reduction and femoral shortening osteotomy
E. Open reduction, femoral shortening osteotomy, and pelvic acetabular osteotomy

4. A 2-week-old infant girl is referred for a hip clunk noticed by the pediatrician. She has a history of a
normal spontaneous vaginal delivery and is otherwise healthy. Examination demonstrates a right hip
Ortolani sign. A coronal ultrasound showed an abnormal acetabular shape and femoral head position.
What is the most appropriate next step in treatment?
A. Observation with repeat ultrasound in 1 month
B. Pavlik harness application
C. Closed reduction and spica casting
D. Open reduction and spica casting
E. Open reduction, acetabular osteotomy, femoral shortening, and spica casting

5. In patients older than 12-months of age with developmental dysplasia of the hip, all of the following
physical exam findings are likely present EXCEPT?
A. Limited hip abduction
B. Positive Ortolani maneuver
C. Abnormal leg lengths on Galeazzi testing
D. Trendelenburg gait

4
E. Pelvic obliquity

6. In infants with developmental dysplasia of the hip (DDH), anatomic closed reduction may be prevented
by all of the following anatomic structure EXCEPT.
A. Interposition of gluteus medius
B. Limbus formed by fibrous tissue and hyaline cartilage
C. Ligamentum teres and prominent fibrofatty pulvinar tissue
D. Contracted transverse acetabular ligament
E. Inverted acetabular labrum

7. A five-year-old boy with cerebral palsy presents to the clinic with a dislocated right hip, what quadrant
of the acetabulum is most likely deficient?
A. Anterior-inferior
B. Anterior-superior
C. Posterior-superior
D. Posterior-inferior
E. Anterior-inferior and anterior-superior

8. Which of the following is true regarding matrix metaloproteinases (MMPs)?


A. They are activated by chelating agent
B. They mediate the destruction of cartilage in septic arthritis
C. Toll-like receptors inhibit the formation of MMPs
D. They have a anabolic effect on cartilage
E. Stromelysin is an indirect antagonist of many MMPs

9. A 3-year-old presents with a 24-hour history of limping and progressive inability to bear weight. The
parents recount no history of trauma, but note that he recently had an upper respiratory infection. A
clinical photo is shown in Figure A. The patient’s vital signs are stable. Physical exam is limited because
of pain. A hip ultrasound is shown in Figure B. Laboratory values are as follows: WBC-15.0 (97% PMN),
ESR-120, CRP-5.0. What is the next best step for this patient?

A. Admit for observation


B. Repeat hip ultrasound
C. Obtain an MRI
D. Start the patient on IV antibiotics
E. Emergent hip arthrotomy with irrigation and debridement

5
10. A 2-year-old child is diagnosed with a septic hip. Initially, no organisms grew on the standard blood
agar plate. However, after 1 week, the offending organism was recovered in an aerobic blood culture
medium. Which of the following organisms was the most likely cause?
A. Kingella kingae
B. Mycobacterium tuberculosis
C. Mycobacterium avium
D. Neisseria
E. E-coli

11. A 2-year-old boy is seen for evaluation of a limp. His history is significant for a left knee infection
treated with IV antibiotics as a neonate and a family history of cancer. Laboratory testing demonstrates a
normal ESR and CRP. The remainder of his workup is negative. An AP pelvis is seen in Figure A. What
was the most likely etiology of his condition?

A. Untreated neonatal hip infection


B. Chondrosarcoma
C. Legg-Calve-Perthes disease
D. Slipped capital femoral epiphysis
E. Osteosarcoma

12. An 8-day-old infant is admitted to the hospital for septic arthritis of the hip. Which of the following will
most likely be the causative organism by culture?
A. Group B Streptococcus
B. Staph Aureus
C. Staph Epidermidis
D. Haemophilus Influenzae
E. Neisseria Gonnorhea

13. A 7-year-old boy developed a limp with right leg pain five days ago, and today has trouble bearing
weight. On exam, he is lethargic and has chills. His temperature is 38.4 degrees centigrade. He points to
his right inguinal region as the source of the discomfort. He winces with compression of his pelvis. Lab
studies reveal a white blood cell count of 11,400/ul, CRP of 0.9 mg/dL (normal < 1.0 mg/dL), and
erythrocyte sedimentation rate of 55 mm/h. A pelvis radiograph is shown in Figure A. Ultrasound guided
aspiration of the right hip joint yields 9,000 leukocytes per mL. What is the most appropriate next step in
management?

6
A. Further imaging of the pelvis
B. Open drainage and irrigation of the right hip joint
C. Repeat aspiration of the hip joint
D. Percutaneous screw fixation of the proximal femoral physis
E. Nonsteroidal antiinflamatory medications and observation

The paper by Beaupre et al discusses that iliac osteomyelitis is a rare cause of pediatric hip pain, and it
can usually be effectively treated with antibiotics alone. Repeat aspiration of the joint is indicated if there
is suspicion of a poorly done procedure, but it was image guided in this case. It is notable that a joint
infection secondary to osteomyelitis is possible in the pediatric hip as result of the synovial reflections
facilitating bacterial migration from the metaphysis to joint space. The synovial fluid analysis is critical in
identifying presence or absence of joint infection.

14. An 18 month-old child has been brought to the emergency room by his mother. He had the sudden
onset of hip pain 3 days ago and now won't put weight on the affected limb. The child is febrile and an
ultrasound (longitudinal view of the proximal femur) shown in Figure A shows the unaffected hip on the
left and affected hip on the right. The patient is taken to the operating room for hip aspiration which
reveals 60,000 leukocytes with 95% polymorphonucleocytes. What is the most likely diagnosis?

A. Traumatic effusion
B. Toxic synovitis
C. Acute rheumatic fever
D. Juvenille rheumatoid arthritis (JRA)
E. Septic arthritis

7
15. An 8-month old infant is brought by his parents to your office for fever and malaise. Your inspection
of the patient is detailed in Image A. An oral temperature of greater than 38.5 has been found to be the
best predictor of this child's condition. What is the second best predictor?

A. Elevated neutrophil count


B. Elevated ESR
C. Elevated rheumatoid factor
D. Elevated CRP
E. Presence of bacteria on CSF gram stain
16. A 6-week old (corrected age) boy refused to move his left hip. The patient was delivered by
C-section 4 weeks premature, but otherwise is healthy. He has been afebrile. Examination reveals some
mild, diffuse swelling about the left proximal thigh. Passive motion of the hip elicits discomfort. An AP
pelvis radiograph is shown in Figure A. What is the most appropriate next step in management?

A. MRI
B. CT scan
C. Observation
D. Aspiration
E. Pavlik Harness

8
17. A 3-year-old boy presents with his caregiver with concerns regarding a long-standing gait
disturbance. The birth history is unknown except for a prolonged ICU stay for sepsis. A pelvic radiograph
is shown in Figure A. What is the most likely cause for this child's limp?

A. Slipped capital femoral epiphysis


B. Legg-Calve-Perthes disease
C. Developmental dysplasia of the hip
D. Residual effects of previous untreated septic hip arthritis
E. Acute femur fracture secondary to child abuse

18. In differentiating pediatric septic hip from transient synovitis, an elevated ESR (>40), history of fever,
refusal to bear weight and what other finding has been identified as predictive of a septic hip?
A. Elevated absolute neutrophil count
B. Serum white blood cell count > 12,000 cells/cubic millimeter
C. Positive blood cultures
D. Pain with hip extension
E. Symptoms greater than 3 days

19. A 10-month-old infant is brought to the emergency department for fevers, irritability, and avoidance of
motion in the right leg. On physical exam, passive motion of the right hip elicits crying. The AP pelvis
shows soft-tissue swelling with mild subluxation of the right hip due to a septic effusion, and the
ultrasound also shows a hip effusion.respectively. A hip aspiration yields 82,000 WBC with >80% PMNs.
Which of the following is the strongest predictor of a poor prognosis?
A. CRP > 5mg/L
B. Delay in treatment >4 days
C. Age > 6 months
D. Absence of associated osteomyelitis
E. ESR > 40mm/hr

20. A 9-year-old male is brought in for initial evaluation of persistent painless limping favoring the left leg.
His symptoms began 6 months ago, and have been progressively worsening. He has nearly full
abduction. Radiographs and an MRI are shown in Figures A, B, and C. What is the next most
appropriate step in treatment?

9
A. Left hip aspiration and culture under fluoroscopic guidance
B. Continued activity limitation and bracing
C. Femoral or pelvic osteotomy
D. Core decompression of the femoral head
E. Work-up for underlying metabolic bone disease

21. A six-year-old boy presents with left leg pain and limping. Radiographs are shown in Figures A and
B. The radiographic changes necessary for accurate lateral pillar classification of his disease are usually
evident how long after the onset of symptoms?

A. 1 month
B. 3 months
C. 6 months
D. 12 months
E. 18 months

22. For children with Legg-Calve-Perthes(LCP) disease, all of the following factors are associated with
femoral head incongruity and worse clinical outcome EXCEPT:
A. Maintenance of less than 50% of lateral pillar height
B. Presentation at 5 years of age
C. Lateral subluxation of the femoral head
D. Calcification lateral to the epiphysis
E. Presence of a radiolucency in the shape of a V in the lateral portion of the epiphysis (Gage sign)

23. A 12-year-old mildly overweight female presents with complaints of left hip pain. She is not
dependent on crutches for ambulation. Physical examination reveals external rotation of the extremity

10
with hip flexion. Her parents indicate that outside radiographs were interpreted to be normal. They
present an MRI of the pelvis, as shown in Figures A and B. What is next best step in management?

A. Observation
B. Arthroscopic labral repair
C. In situ screw fixation
D. Closed reduction and percutaneous pinning
E. Debridement of CAM impingement femoral lesion

24. A 12-year-old girl presents with groin pain six months after treatment of a slipped capital
femoral epiphysis. Preoperative radiographs are seen in Figure A, radiographs six months after
in situ fixation are seen in Figure B. Which of the following is associated with the radiographic
abnormality seen in Figure B?

A. Lack of reduction prior to fixation


B. Single screw fixation
C. Female sex
D. Inability to bear weight preoperatively
E. Obesity

25. A 13-year-old boy complains of a 3-month history of left knee, thigh and groin pain. His pain
has significantly worsened over the past week. He denies pain in the right leg. Radiographs are

11
taken and shown in Figures A and B. The history and physical do not reveal any findings
concerning for an endocrine disorder. What is the preferred method of treatment?

A. Subtrochanteric valgus, extension, and external rotational osteotomy


B. Non weight bearing on the left side for 6 weeks.
C. Bilateral in situ single screw insertion across the proximal femoral physis
D. In situ single screw insertion across the left proximal femoral physis only
E. Varus derotational osteotomy of the proximal femur

26. ​Southwick angle (epiphyseal-shaft angle) serves what purpose in the evaluation of a slipped
capital femoral epiphysis (SCFE)?
A. Determine prognosis for AVN
B. Determine the severity of the slip
C. Determine the presence or absence of a slip
D. Determine the etiology of a slip
E. Determine the chronicity of the slip

27. ​Which of the following treatment techniques decreases the risk of osteonecrosis in patients
with unstable slipped femoral capital epiphysis (SCFE)?
A. Open reduction and pinning with multiple cannulated screws in an inverted triangle
configuration
B. Closed reduction and pinning with multiple cannulated screws in an inverted triangle
configuration
C. Closed reduction and pinning with a single cannulated screw
D. In situ percutaneous pinning with multiple cannulated screws in an inverted triangle
configuration
E. In situ percutaneous pinning with a single cannulated screw

28. Hypothyroidism is most commonly associated with which of the following pediatric
conditions?
A. Legg Calve Perthes

12
B. Slipped capital femoral epiphysis
C. Toxic synovitis
D. Achondroplasia
E. Rickets

29. n 11-year-old girl with hypothyroidism and obesity presents with groin pain and the inability
to ambulate. Her radiograph is shown in Figure A. What is the most appropriate treatment?

A. Toe-touch weightbearing for 3 weeks


B. Hip spica cast and non-weight bearing for 4 weeks
C. In situ pinning of the right hip
D. Open reduction and pinning of the right hip
E. In situ pinning of both hips

30. A 13-year-old Polynesian boy presents with left groin pain and inability to place weight on
the left leg. His radiographs are shown in Figures A. All of the following are true regarding this
condition EXCEPT:

A. The left hip is more commonly involved


B. Forceful manipulation is not indicated because it is associated with an increased risk of
complications

13
C. Associated with decreased femoral anteversion and decreased femoral neck-shaft angle
D. Pain is localized to the knee more often than the hip on initial presentation
E. Males are more commonly affected than females

31. A 14-year-old boy presents with left groin and knee pain for 3 weeks. He is now unable to
place weight on the left lower extremity, even with the assistance of crutches. AP pelvis
radiograph showed SUFE of left hip. He is treated with percutaneous pin fixation. What is the
most common limb length and rotational profile found as a sequelae of this condition?
A. Limb shortening, decreased hip flexion and decreased hip internal rotation
B. Limb lengthening, increased hip flexion, and increased hip internal rotation
C. Limb lengthening, decreased hip flexion, and decreased hip external rotation
D. Limb shortening, decreased hip flexion, and increased hip internal rotation
E. Limb shortening, increased hip flexion, and decreased hip internal rotation

32. A 14-year-old overweight boy complains of vague left knee pain which worsens with activity.
He has an antalgic gait and increased external rotation of his foot progression angle compared
to the contralateral side. Knee radiographs, including stress views, are negative. What is the
next step in management?
A. Knee MRI
B. Knee CT
C. AP pelvis and frog-lateral views
D. Diagnostic knee arthroscopy
E. Hip MRI

Questions 33-36
For each case listed below, select the most likely diagnosis. Each lettered option may be used
once, more than once, or not at all.
a. Legg-Calvé-Perthes disease
b. Slipped capital femoral epiphysis
c. Osteomyelitis
d. Septic arthritis of the hip
e. Transient synovitis
33. An afebrile, obese 14-year-old boy has developed pain at the right knee and a limp.

34. A 6-year-old boy has developed a limp and has limited mobility of the hip, but denies pain
and fever.

35. A 2-year-old refuses to walk, has fever, has significant pain with external rotation of the right
leg, and has an elevated WBC count.

14
36. A 3-year-old refuses to walk, is afebrile, had an upper respiratory tract infection a week ago,
has right hip pain with movement, and has a normal WBC count.

37. ​A 19-year-old female presents with metatarsalgia and difficulty with wearing closed-toe
shoes on her left foot. She is found to have a muscular strength imbalance between the anterior
tibialis and peroneus longus on the left side. A clinical image and lateral foot radiograph are
shown in Figures A and B, respectively. Which congenital condition most likely contributed to
the development of the current foot deformity?

A. Congenital calcaneonavicular coalition


B. Congenital vertical talus
C. Congenital oblique talus
D. Congenital talipes equinovarus
E. Calcaneovalgus foot

38. ​Which of the following components of the clubfoot deformity should be addressed first when
using the Ponseti method?
A. Cavus
B. Equinus
C. Pronation
D. Hindfoot alignment
E. Metatarsal adduction

39. What is the first line treatment for a congenital talipes equinovarus in a 9 months old baby?
A. Observation
B. Serial casting
C. Manipulation under anesthesia followed by a single casting
D. Surgical re-alignment
E. Serial manipulation and casting followed by surgical release and talonavicular reduction
with pinning

40. ​Which of the following statements regarding child abuse is correct?


A. It is the 2nd most common cause of death in children over 1 year of age
B. Fractures in child abuse occur more often children greater than 5 years of age

15
C. Burns are the most frequent cause of long-term physical morbidity
D. Fractures are the most common presenting injury
E. Metaphyseal fractures four-times more common than diaphyseal fractures

41. ​A 12-year-old basketball player is seen for a routine physical. During the physical, he is
reluctant to partake in the full physical exam. Upon further questioning, he states that a member
of the coaching staff has touched him inappropriately on repeated occasions. Which of the
following best outlines your legal responsibility as a physician?
A. Inform the parents
B. Inform the school board
C. Inform the athletic director
D. Inform the child health and protective services
E. Inform the school counselor

42. ​Which of the following must be done whenever a non-ambulatory infant presents to the ER
with a diaphyseal long bone fracture?
A. Immediate consultation with child protective services and possible admission to the
hospital
B. Order serum vitamin D levels
C. Order serum calcium and phosphorus levels
D. MRI of the cervical spine
E. Perform genetic testing for COL1-A1 and COL1-A2

43. ​A 10-month-old child is brought to the emergency room with left elbow swelling and an intact
neurovascular exam. The parents report no history of trauma. A radiograph demonstrates a
minimally displaced distal humerus physeal separation with abundant callus formation. The next
step in managament should include?
A. Closed reduction and percutaneous pinning
B. Casting only
C. A skeletal survey and contacting the appropriate authorities
D. Elbow arthrogram
E. Open reduction and internal fixation

44. Which of the following is NOT a common fracture pattern seen in abused children?
A. Metaphyseal corner fractures
B. Spiral long bone fractures
C. Multiple fractures in different stages of healing
D. Anterior translation of the femoral neck relative to the femoral epiphysis
E. Posterior rib fractures

45. ​A 15-year-old male presents with pain and progressive deformity about his left knee. A
standing AP radiograph is seen in figure A, with the tibial growth plate nearly closed. Physical

16
examination reveals significant varus and a leg-length discrepancy of 2.5cm right greater than
left. Which of the following is the most appropriate method of management at this time?

A. Orthotics
B. Hemiepiphysiodesis of the left proximal tibia, medial side only
C. Epiphysiodesis of the left proximal tibia
D. Left proximal tibia osteotomy with placement of lengthening external fixator
E. Left proximal tibia osteotomy with plate fixation

46. ​A 12-year-old girl has been diagnosed with a severe form of osteogenesis imperfecta that
has resulted in thin bones and multiple fractures. She now presents for follow-up of scoliosis
which was noticed by her mother 1 year ago. She has no back pain and is neurologically intact.
Radiographs show a 42-degree right thoracic curve, which has increased from 31-degree from
her previous radiographs taken 9 months earlier. What is the most appropriate management for
her spinal deformity?
A. Observation alone with serial radiographic and clinical monitoring.
B. Posterior spinal fusion with instrumentation
C. Anterior spinal fusion with instrumentation
D. Fixation with telescoping rods
E. Custom-molded bracing with serial radiographic and clinical monitoring.

47. An adolescent male with a known Osteogenesis Inperfecta presents to the emergency
department with symptoms of unsteady gait and difficulty with buttoning his shirt. On physical
exam, when the patient holds his fingers extended and adducted, the small finger
spontaneously abducts. Snapping of the patient's distal phalanx of the middle finger leads to

17
spontaneous flexion of the other fingers. What is the most likely cause of the patient's
symptoms?
A. Basilar invagination
B. C6 cervical disc foraminal herniation
C. Atlantoaxial instability
D. Ossification of the posterior longitudinal ligament (OPLL)
E. Epidural abscess

48. ​A 12-year-old girl presents with a history of numerous long bone fractures in the past.
Examination reveals normal-appearing sclera, and the dentin abnormality. A radiograph of her
lower extremities showed lower extremity bowing and malunion. The patient’s disorder is the
result of which of the following?
A. Vitamin D deficiency
B. Abnormal osteoclast function
C. Qualitative defect of type I collagen synthesis
D. Mutated fibroblast growth factor receptor
E. Defective N-Ac-Gal-6 sulfate sulfatase enzyme

49. ​A 7-year-old patient presents with a fracture of her left supracondylar humerus and distal
radius. She is neurovascularly intact and the skin shows no evidence of open wounds.
Radiographs of the elbow show a displaced supracondylar fracture. Radiographs of the wrist
show an extra-articular distal radius fracture with 25 degrees of dorsal angulation. This injury is
most appropriately treated with which of the following?
A. Closed reduction and casting of the supracondylar humerus fracture and distal radius
fracture
B. Closed reduction and pinning of both the supracondylar humerus fracture and distal
radius fracture
C. Closed reduction and casting of the supracondylar humerus fracture and pinning of distal
radius fracture
D. Open reduction and pinning of both the supracondylar humerus and the distal radius
fracture
E. Closed reduction and pinning of the supracondylar humerus fracture and closed
reduction and casting of distal radius fracture

50. ​A 7-year-old boy falls off the playground and sustains a flexion type supracondylar fracture
of his humerus. What motor deficit is associated with the nerve most commonly injured in this
fracture pattern?
A. Weakness of the flexor digitorum profundus to the index finger
B. Weakness of the extensor pollicis longus
C. Wrist drop
D. Weakness of the flexor pollicis longus
E. Hand intrinsic weakness

18
51. ​What is the etiology of cubitus varus following a supracondylar humerus fracture in a child?
A. Overgrowth of the lateral physis
B. Malreduction of the fracture
C. Growth arrest of medial physis
D. Injury to the ulnar nerve
E. Radial head dislocation

52. ​ An 18-month-old girl is brought to clinic by her mother for in-toeing. All of the following
features should prompt the physician to perform further evaluation (including radiographs) if
found in conjunction with in-toeing EXCEPT:
A. limb length discrepancy
B. metatarsus adductus
C. pain
D. family history of skeletal dysplasias
E. limb rotational profiles 2 standard deviations outside of normal

19
ANSWER:
1.​Ans. C
Initial ultrasound is performed to confirm reduction of the hip in question (generally after 1 or 2 weeks)
followed by repeat ultrasound 6 weeks later. Ultrasound is necessary to avoid leaving an infant in a
harness with an unreduced hip which can erode the acetabulum.

Weinstein et al. provide a thorough overview of the presentation, evaluation, and treatment of DDH. The
use of office-based ultrasound has helped to confirm hip reduction and proper acetabular development in
children being treated with Pavlik harness.

Swaroop and Mubarak performed a retrospective, comparative study looking at children with
Ortolani-positive hips who were treated with either Pavlik Harness and education or with Pavlik Harness,
serial ultrasound exams, and a hip abduction orthosis upon failure of Pavlik harness. They achieved 93%
success in the latter group.
Incorrect Answers:
Answer 1: MRI requires sedation in patients of this age and is not required as ultrasound can provide the
needed information. MR can be useful in confirming closed reduction in older children (6-12 months).
Answer 2: CT exposes the child to unnecessary radiation and is not needed to confirm hip reduction in
the harness. CT can be useful in confirming closed reduction in a spica cast.
Answer 4: Plain radiographs can help make the diagnosis of hip dysplasia however they do not provide
as much information as ultrasound imaging as the femoral head is not yet ossified in infants.
Answer 5: Arthrogram and fluoroscopy require sedation as well and are more appropriate if a closed
reduction in the operating room is required.

2. Ans.: D
The ossific nucleus of the proximal femur is visible on radiographs by 6 months of age in most children.

The proximal femoral physis and greater trochanteric apophysis develop from the same cartilage physis
in the infant which undergoes apoptotic division in the child. The distal femoral physis (not proximal)
grows at a rate of 9 mm per year. The normal infant femoral anteversion is between 30-40 degrees.
SCFE typically occurs through the zone of hypertrophy, not the zone of proliferation.

3. ANS: C
A 4-month-old who fails Pavlik harness treatment is best treated with closed versus open reduction of the
hip and spica casting. Continued harness treatment can be detrimental as there is risk of posterior
acetabular erosion. Osteotomies are not necessary to achieve reduction in a patient of this age cohort.

In the review by Guille et al, he reports unsuccessful treatment of DDH with Pavlik Harness treatment if
the hip does not become reduced within 2 weeks of treatment and recommends transitioning to
alternative treatment options.

Mubarak, et al reports 12 failures in 18 infants with hip dislocations due to improper physician technique
and/or inappropriate harness. Failure to recognize inadequate reduction portends worse clinical
outcomes. Another option for failed Pavlik harness treatment in infants is a hip abduction brace.

20
Hedequist reports on 13 of 15 patients who failed Pavlik harness treatment, and went on to resolve DDH
with an abduction brace avoiding the operating room and anesthesia for a closed reduction and spica
casting.

4. ANS: B
This patient has a right hip dislocation (DDH), as demonstrated by the positive Ortolani sign. Pavlik
harness application is indicated for treatment. If the hip does not stay reduced within a few weeks, the
next option is an arthrogram under anesthesia, closed reduction, and spica casting. Open reduction and
casting is reserved for when closed reduction has failed. Acetabular osteotomy and femoral shortening
are procedures used for children with DDH typically older than 1.5 years.

5. Ans: B
There are many exam maneuvers which are used in the diagnosis of developmental dysplasia of the hip
(DDH). Exam findings differ based on the age of the patient.

Vitale et al emphasize that physical exam findings associated with DDH in a child older than 12-months
can be different than those seen in the newborn. Specifically, limited hip abduction, a positive Galeazzi
test, a positive Trendelenburg gait, and asymmetry of hip abduction are all useful exams tests that are
likely to be positive. The Barlow and Ortolani maneuvers are of limited use in older children (> 6 months)
because the soft tissues about the hip tighten.

6. Ans: A
Interposition of gluteus medius is not associated with blocked reduction in patients with DDH.

The review article by Guille et al reviews the soft-tissue adaptations in DDH, and discusses the various
blocks to reduction which include:
Fibrous tissue can merge with the hyaline cartilage of the acetabulum rim forming the limbus, which may
then prevent concentric reduction of the hip.
The ligamentum teres and fibrofatty tissue, known as the pulvinar, may be found within the depths of the
acetabulum and can also be an obstacle to reduction and are both depicted by the black arrows in
Illustration B.
The transverse acetabular ligament at the caudal aspect of the acetabulum, contracts in patients with
persistent hip dislocation and is a block to concentric reduction of the hip.
In the older infant with DDH, the acetabular labrum may be inverted and may mechanically block
concentric reduction of the hip.

7. Ans: C
In patients with cerebral palsy, the hip is normal at birth, but a combination of muscle imbalance and
bony deformity leads to progressive hip dysplasia. The review article by Flynn notes that spasticity or
contracture usually involves the adductor and iliopsoas muscles. Because of the pull of these muscles,
the majority of hips subluxate in the posterosuperior direction. Because physical examination is
unreliable, an AP of the pelvis is required for diagnosis.

8. Ans: B
Matrix metalloproteinases have been associated with the destruction of cartilage in septic arthritis.

Matrix metalloproteinases and their inhibitors, tissue inhibitors of metalloproteinases (TIMPs), are crucial
to extracellular matrix remodeling in normal tissue development and maintenance. Additionally, their

21
over-expression has been associated with cartilage degradation in diseases such as rheumatoid arthritis,
osteoarthritis, and septic arthritis. In septic arthritis, toll-like receptors activate the NF-kB pathway which
leads to the production of MMPs and resulting cartilage destruction.

9. ANS: E
Based on the clinical findings and figures shown, the patient has developed a septic arthritis of the left
hip. As the patient has 3 out of the 4 Kocher criteria, he has a 93% chance of having a septic hip. The
next best step in management would be to take the patient to the operating room for an emergent
irrigation and debridement of the affected hip.

Septic arthritis in the pediatric population often occurs in the first few years of life, with 50% of cases
occurring in those less than 2 years of age. Patients may present with a toxic appearance. The likelihood
of a patient having a septic hip can be ascertained with use of the Kocher criteria (WBC > 12, ESR > 40,
T > 38.5 and an inability to bear weight on the affected hip). Patients meeting all four criteria have a 99%
chance of having a septic hip, whereas those meeting just one of the criteria have a 3% chance of
having a septic hip. Rapid breakdown of the hyaline articular cartilage occurs via enzymes (matrix
metalloproteinases & hyaluronidase) produced by the bacteria. This may be mitigated with an emergent
surgical irrigation and debridement.

Rutz et al. review septic arthritis of the pediatric hip. Diagnosis in infants may be difficult because this
subset of patient do not always develop fevers. They recommend arthroscopic irrigation and
debridement for those patients with an acute presentation and no evidence of osseous complications on
radiographs. For those with a subacute presentation or radiographically visible complications of the
femoral head, an open arthrotomy should be completed.
Figure A shows a patient with a hip effusion, holding the extremity in an flexed, abducted and externally
rotated position. Figure B shows an ultrasound demonstrative of a joint effusion. The region of capsular
distention can be seen anterior to the femoral neck.

10. ANS: A
Kingella kingae is a fastidious organism which is recovered on blood culture medium, recently with the
addition of the BACTEC blood culture system. Mycobacterium tuberculosis is grown on Lowenstein
Jensen (LJ) medium, while Mycobacterium avium can be grown on the LJ or Middlebrook medium.
Neisseria is typically grown on the Thayer-Martin medium. E-coli is grown on the Luria Bertani medium.
Petti et al studied the use of standard and extended blood culture incubation for Haemophilus,
Actinobacillus, Cardiobacterium, Eikenella, and Kingella (HACEK) bacteria. Although all are fastidious
organisms, they determined that standard incubation time on a blood culture medium is sufficient to
recover HACEK bacteria.

11. ANS: A
The clinical scenario and images are consistent with a neglected pediatric septic hip. The AP pelvis in
Figure A shows an absent left femoral head due to an untreated infection. Failure to diagnose an
infected adjacent joint can lead to joint destruction and physeal damage with resultant deformity. Surgical
options for hip deformity following a neglected infection include trochanteric osteotomy, proximal femoral
varus osteotomy, and a modified Albee arthroplasy.

22
12. ANS: A
Group B Streptococcus is the most common causative organism of neonatal septic arthritis. It should be
noted in the subgroup of neonates who acquire nosocomial septic arthritis, Staph Aureus is more
common than Group B Strep. These infections often arise from invasive procedures such as central lines
and indwelling catheters. Children in the first 2 years of life have the highest frequency of septic arthritis.
In children 3 months of age to 2 years, H. Influenzae was traditionally the most common organism, but its
prevalence has fallen significantly since the introduction of H. Flu vaccinations. In children over 2 years
of age, Staph Aureus is the most common causative organism.

13. ANS: A
This patient has clinical signs of infection with symptoms localized to the pelvis. The differential diagnosis
of an infectious presentation with NWB in a child should include: discitis, sacroilitis, transient synovitis,
septic hip, osteomyelitis, and Iliopsoas abscess. Further imaging is required to confirm the diagnosis.
The radiographs are not consistent with a slipped capital femoral epiphysis.

An appropriate workup has been completed for septic arthritis, which is a surgical emergency and
prompts drainage and debridement of the hip joint. The Kocher criteria for septic arthritis include
fever>38.5 degrees centigrade, inability to bear weight, ESR>40 mm/h, and WBC count>12,000/ul. In
this case, 2/4 of the criteria are positive (inability to bear weight & ESR>40mm/h), which indicates
approximately a 40% likelihood of septic arthritis. Synovial fluid analysis is used to either confirm or
reject the hypothesis of suspected septic arthritis; an aspiration of < 50,000 leukocytes per mL virtually
rules out sepsis of the joint.

14. ANS: E
This patient most likely has a septic hip based on clinical, radiographic, and laboratory data. Traumatic
effusions have less than 5,000 leukocytes, toxic synovitis (5,000-15,000 leuckocytes with <25% PMNs),
rheumatic fever (10,000-15,000 leukocytes with 50% PMNs), and JRA (15,000-80,000 leukocytes with
75% PMNs). Synovial fluid analysis for septic arthritis includes >50,000 leukocytes and >75% PMNs.
The review article by Sucato et al notes that JRA can be present similarly to a septic joint with a hip
effusion with high leukocyte count and >75% PMN's. However, they mention that JRA patients often
have gradual onset of symptoms, less pain, usually continue weightbearing activities, and lack
constitutional symptoms. Illustration A compares the sonographic findings in a normal hip compared to a
hip with an effusion.

15. ANS: D
The patient's clinical image reveals a flexed, abducted, and externally rotated left hip. This hip position
maximizes intracapsular volume and indicates a left hip effusion as described in the review by Sucato.

Caird et al performed a Level 1 study that concluded that a temperature above 38.5 was the best
predictor of septic arthritis followed in decreasing order by CRP, ESR, refusal to bear weight, and serum
WBC count. An elevated rheumatoid factor can be seen in juvenile rheumatoid arthritis but fever is not
part of the ACR (American College of Rheumatology) criteria. Presence of bacteria on CSF gram stain
indicates a diagnosis of bacterial meningitis, which in isolation does not cause a flexed, abducted, and
externally rotated hip. Illustration A shows the predicted probability of septic arthritis based on the

23
number of factors present in the patient and compares Kocher's original 4 factors (ESR, WBC, fever,
refusal to bear weight) and Caird's addition of CRP as a 5th factor.

16. ANS: D
This question is describing a scenario in which you must rule out a septic hip, and therefore the most
appropriate next step in management is aspiration.
Prematurity and c-section are both risk factors for a septic hip in the new born.While there is no fever in
this case, there is swelling, pain with passive motion, no active motion secondary to pain, and a
radiograph which indicates an effusion in the hip as seen by lateral displacement of the left proximal
femur. Suspicion for a septic hip should be high, and the next most appropriate step is a hip aspiration. If
no fluid is obtained, arthrography should be performed to confirm intra-articular position of the needle.
The aspirate should be sent for a stat CBC with diff, culture (aerobic, anaerobic and acid-fast bacilli +/-
fungal), and gram stain. A WBC greater than 50,000/uL or a positive gram stain suggest septic arthritis
and are an indication for surgical incision and drainage and initiation of IV antibiotics.

17. ANS: D
This child is presenting with radiographic and clinical findings consistent with the sequelae of untreated
septic hip arthritis. Specifically, the child is limping and there is complete destruction of the femoral
head/neck in the pelvic radiograph. Furthermore, the radiograph is not consistent with any of the
remaining choices.
Incorrect answers:
Slipped capital femoral epiphysis - radiographs would show the epiphysis displaced upon the femoral
neck and typically occurs in an older age group (LEFT)
Legg-Calve-Perthes disease - radiographs would show the epiphysis undergoing some stage of
resorption or reossification (MIDDLE)
Developmental dysplasia of the hip - radiographs would show the femoral head incompletely covered by
a dysplastic acetabulum (RIGHT)
Femur fracture - radiographs would show a fracture

24
18. ANS. B.
In the classic article by Kocher et al, the authors reviewed pediatric patients with septic arthritis and
concluded that four independent multivariate clinical predictors were identified to differentiate between
septic arthritis and transient synovitis: history of fever, non-weight-bearing, erythrocyte sedimentation
rate of at least forty millimeters per hour, and serum white blood-cell count of more than 12,000 cells per
cubic millimeter (12,000 cells/cubic millimeter).

19. ANS: B
The clinical scenario is consistent with a pediatric septic hip. The hip aspiration is consistent with an
infectious process. An aspirate with WBC >50,000 is highly suggestive of a septic hip. Jackson et al
reviewed pediatric septic arthritis and describe four poor prognostic signs: age <6 months, joint effusion
with underlying osteomyelitis, hip involvement, and delay in treatment >4 days. In a review of pediatric
septic hips, Sucato et al state that hip aspiration is the most sensitive test and that I&D is required to
prevent late sequlae.

20. ANS: C
This patient is presenting with late stage Legg-Calve-Perthes (LCP)disease. The radiographs and MRI
scan demonstrate density changes and collapse of the femoral head consistent with lateral pillar group
B.

21. ANS: C
The lateral pillar classification of Legg-Calve-Perthes (LCP) disease is determined from anteroposterior
radiographs of the pelvis made in the early fragmentation stage of the disease. The lateral pillar is
defined as the lateral portion of the femoral head, on the anteroposterior radiograph, that is demarcated
from the central portion of the head by a lucent line of fragmentation. Below show the lateral pillar
classification (A normal, B 50-100% C <50%)

22. Ans: B
LCP is a disease of children in which the vascular supply to the femoral head is compromised leading to
avascular necrosis of the femoral head and can subsequently result in resorption, collapse and repair.
Children who present at an age < 6 years have an improved prognosis.

25
Catterall described five "at-risk" signs, which indicate a more severe disease course including: 1) Gage
sign as shown in Illustration A (radiolucency in the shape of a V in the lateral portion of the epiphysis), 2)
calcification lateral to the epiphysis, 3) lateral subluxation of the femoral head 4) a horizontal physis and
5) metaphyseal cysts.

23.ANS: C
Based on the clinical history and imaging provided, the patient has a diagnosis of a stable
slipped capital femoral epiphysis. Considering the normal interpretation of the radiographs,
either the read was incorrect or the slip had worsened by the time the MRI was obtained. Based
on these findings, the next best step in management is in situ percutaneous screw fixation.

Slipped capital femoral epiphysis (SCFE) is a slippage of the epiphysis relative to the femoral
neck through the zone of hypertrophy. The ability to bear weight without crutches indicates
minimal risk of AVN (< 10%). An MRI can be useful in the diagnosis of SCFE when plain
radiographs are normal; there is increased edema within the metaphysis and physeal widening.
Treatment requires percutaneous in-situ pinning.

24.ANS: D
The patient presents with femoral head osteonecrosis (Figure B), following in-situ screw fixation
of a severe SCFE (Figure A). The inability to bear weight, even with assistive devices,
preoperatively indicates an unstable SCFE, which is associated with significantly increased
rates of osteonecrosis.

25. ANS: D
Acute or acute on chronic slipped capital femoral epiphysis (SCFE) can be adequately stabilized
with a single percutaneous screw fixation across the proximal femoral physis.

Goodman et al retrospectively demonstrated that in situ, single screw fixation across the
proximal femoral physis in patients with acute or acute on chronic SCFE, was sufficient to allow
closure of the physis with no change in head-shaft angles from the preoperative radiographs.

26
Morrissy in an instructional course lecture, illustrates the principles of in situ fixation in chronic
SCFE. Pinning of the contralateral physis should strongly be considered for boys under 12 and
girls under 10, the presence of an open triradiate cartilage, or evidence of an endocrinopathy.

26. ANS: B
The epiphyseal-shaft angle, as described by Southwick, is measured on the frog-leg lateral
radiograph to determine the degree of the slip, which is calculated by subtraction of the angle on
the normal side from the angle of the affected hip. The Southwick angle is also helpful when
planning an osteotomy for post-SCFE impingement.

The classification of SCFE based on grading the severity of the slip by xrays is the Southwick
angle and is based on the difference between the two hips on the cross-table lateral: 1) mild, 0
to 29 degrees; 2) moderate, 30 to 60 degrees; severe, 3) greater than 60 degrees. The classic
xrays to view a slip are the AP pelvis and frog-leg lateral. The frog leg lateral often gives a better
image of the slip compared to the AP b/c the epiphysis is posterior relative to the neck and this
is better seen on a lateral view.

27. ANS: E
As described in the review article by Loder, an unstable SCFE is one where the child cannot
walk, with or without crutches. Reduction attempts of unstable SCFE have been associated with
a higher rate of osteonecrosis after pinning. Osteonecrosis is also more likely to develop in
patients treated with multiple pins than in those treated with a single cannulated screw.
However, in unstable SCFE's surgeons often elect to utilize two screws for stabilization.
Inverted triangle screw placement is utilized for adults with femoral neck fractures. Tomkmakova
concluded that: "Pinning in situ without reduction with a single cannulated screw is the method
of choice for the treatment of a slipped capital femoral epiphysis."

28. ANS: B
SCFE is a disorder of the proximal femoral epiphysis caused by weakness of the perichondral
ring and slippage through the hypertrophic zone of the growth plate. The femoral neck displaces
anteriorly and externally rotates. An AP radiograph, shown in Illustration A, and lateral (as the
epiphysis most commonly appears posterior to the metaphysis) views often can illustrate the
deformity seen with SCFE. Consideration is given to obtaining a cross table lateral view as a
frog-lateral may worsen the unstable slip. SCFE is seen most commonly in African American,
obese, and adolescent boys (10 – 16 yrs old) with a positive family history. Twenty-five to 50%
of cases are bilateral and thought to be related to hormonal disorders in young children, such as
hypothyroidism or renal osteodystrophy. Careful endocrine screening is warranted in at risk
patients. Loder et al reviewed 85 pediatric patients with both SCFE and endocrine disorders and
found hypothryoidism in 40% and growth hormone deficiency in 25%. All the patients with
hypothyroidsim developed bilateral SCFE.

29. ANS: E

27
The radiographs show a slipped capital femoral epiphysis (SCFE) of the right hip. Patients with
endocrine disease such as hypothyroidism commonly demonstrate prevalence of bilaterality (as
high as 80% in some reports), prophylactic treatment of the opposite hip should be considered.
When adding additional risks of contralateral hip slip of obesity and young age and the
significant morbidity of displacement of slipped capitol femoral epiphysis, the controversy of
prophylactic contralateral pinning is diminished.

The general recommendations for prophylactic pinning include inability to obtain regular follow
up, metabolic/endocrine disorders, open triradiate cartilage, girls younger than 10, and boys
younger than 12.5 years.

30. ANS: D
This patient has a left slipped capital femoral epiphysis. Option D is incorrect as hip pain is more
common then knee pain. Other options are all true statements as stated in AAOS
Comprehensive Orthopaedic Review.

31. ANS: A
This patient underwent in-situ pinning of an unstable slipped capital femoral epiphysis (SCFE).
Forceful manipulation is not indicated because it is associated with an increased risk of
osteonecrosis. Patients with SCFE can present with an out-toeing gait, limb shortening,
decreased hip flexion, decreased hip abduction, and decreased hip internal rotation. A
frequently seen sign associated with SCFE includes obligatory abduction and external rotation
during passive hip flexion from an extended position.

32. ANS: C
In an adolescent boy with knee pain, always examine the hips and consider hip pathology,
especially if the knee workup is negative. Matava et al discusses knee pain as the initial
symptom of SCFE. This retrospective review of 65 patients found that 15 (23%) noted distal
thigh pain, knee pain, or both as the presenting symptom. 12 were chronic slips (>3 weeks of
pain) and 3 acute-on-chronic. Knee and thigh pain resulting from intra-articular hip pathology is
a referred pain phenomenon, and is a common reason for misdiagnosis of SCFE leading to
delay in treatment, possible further displacement, and worse prognosis.

33-36 The answers are 33-b, 34-a, 35-d, 36-e.


Slipped capital femoral epiphysis is a disease of unknown etiology and occurs typically in
adolescents; the disorder is most common among obese boys with delayed skeletal maturation
or in thin, tall adolescents having recently enjoyed a growth spurt. Onset of this disorder is
frequently gradual; pain referred to the knee in 20% of cases can mask the hip pathology.

Legg-Calvé-Perthes disease is avascular necrosis or idiopathic osteonecrosis of the femoral


head; the cause of this disorder is unknown. Boys between the ages of 2 and 12 years are most
frequently affected (incidence in boys is greater by four- to fivefold), with a mean of 6 to 7 years

28
old. Presenting symptoms include a limp and pain in the anterior thigh, groin, or knee, although
classic symptoms include a painless limp.

Septic arthritis requires urgent intervention to preserve joint mobility. Joint aspiration is
diagnostic and can be helpful in treatment. Opening the joint space may be required in a septic
hip to assist in draining purulent material. These children need treatment for 4 to 6 weeks.

Transient synovitis is a disorder of unknown etiology, affecting children usually from 2 to 6 years
of age. These children usually present with a painful limp. This is a diagnosis of exclusion;
septic hip and osteomyelitis must be ruled out. While the WBC count and ESR may be normal,
or they also may be slightly elevated. Early aspiration of the joint space may assist in
diagnosis.Transient synovitis is a self-limited disorder.
Osteomyelitis usually presents with focal bone tenderness and fever. Early evaluation is best
done through nuclear medicine studies, as plain film bony changes usually take a week or so
before becoming evident.

37. D
The history, clinical image, and radiograph is consistent with a dorsal bunion and is most likely
secondary to a history of congenital clubfoot (talipes equinovarus).

The anterior tibialis muscle is the antagonist of the peroneus longus muscle and its integrity or
overactivity is required for the development of a dorsal bunion. An abnomality of its bone
insertion or its retraction can be found in congenital clubfoot and partly explains the natural
tendency of these feet to develop a dorsal bunion. Dorsal bunions can also occur following
surgical clubfoot release secondary to a weak Achilles tendon, excessive power of the flexor
hallucis longus muscle, a strong anterior tibial tendon, and weakness of the peroneus longus
tendon.

38. A
Cavus should be addressed first when using the Ponseti method to treat clubfoot.

Idiopathic clubfoot is characterized by forefoot adductus, hindfoot varus, ankle and subtalar
equinus, and supination of entire foot. The forefoot is pronated relative to hindfoot. Correction of
clubfoot requires an organized and sequential methodology. The helpful acronym is CAVE
(midfoot cavus, forefoot adductus, hindfoot varus, hindfoot equinus) which describes both the
clinical position and the general order of deformity correction in the Ponseti method.

39. B
The Ponseti serial casting method is reportedly effective for treating clubfoot in children up to 9
years of age.

40. A
Child abuse is the 2nd most common cause of death in children > 1 year of age, behind
accidental injury.

29
More than 1 million children are victims of substantiated abuse or neglect in United States each
year. Red flags in the history include long bone fractures in an infant that is not yet walking,
multiple bruises, multiple fractures in various stages of healing.
Incorrect Answers:
Answer B: Child abuse is most common in children < 3 years old.
Answer C: Head injury is the most frequent cause of long term physical morbidity.
Answer D: Skin lesions are the most common presentation.
Answer E: Diaphyseal fractures are four-times more common than metaphyseal fractures.

41. D
he scenario described strongly suggests child abuse. Any suspicion of child (<18 yrs) abuse
should be reported to Child Protective Services (CPS) as this is typically required by law. Laws
regarding reporting and age of consent between minors varies from state to state and
physicians must be aware of their state requirements.

42. A
Each of the answers could be performed in this scenario as part of a diagnostic evaluation.
However, missing a case of child abuse could result in further abuse of the child or even death,
making this the most important issue to address.

43. C
Distal humeral physeal separations in the infant or young child are most often the result of
violent traction or rotation and are commonly associated with child abuse. They may initially be
difficult to recognize in newborns and young infants when the secondary ossification center is
not apparent. In these cases, arthrography, ultrasound, or magnetic resonance imaging may be
required to make the diagnosis. For young children presenting more than 3 weeks out abundant
callus formation may be noted. In children where there is ossification of the epiphyseal ossific
nucleus the diagnosis is typically made on plain radiographs. Once the diagnosis is confirmed
the next step should be further workup to assess for other injuries and to contact the appropriate
authorities.

44. D
Anterior translation of the femoral neck relative to the femoral epiphysis best describes Slipped
Capital Femoral Epiphysis (SCFE) and is not typically associated with abuse or non-accidental
trauma.

Fracture patterns associated with child abuse which should raise one's suspicion include:

1) metaphyseal corner fractures


2) spiral fractures
3) multiple fractures at different stages of healing
4) single transverse long bone fractures
5) posterior rib fractures
6) skull fractures

45. D

30
Adolescent Blount's disease with significant varus malalignment, a coexisting leg-length
discrepancy, and closed growth plates is best treated with a proximal tibia osteotomy with
placement of an external fixator.

Adolescent Blount's disease is defined as pathologic genu varum with onset at greater than 10
years of age. Non-operative management is considered for only very mild cases, as the
deformity has a tendency to progress and cause medial joint pain. Surgical management
consists of either hemiepiphysiodesis or osteotomy. If insufficient growth is remaining to allow
for correction via hemiepiphysiodesis, osteotomy is the best option for correction of the
deformity. Placement of an external fixator after osteotomy allows for correction of the
coexistent leg-length discrepancy.

46. B
Children with severe forms of osteogenesis imperfecta and progressive scoliosis should be
treated with posterior spinal fusion with instrumentation when the curve exceeds 35 degrees.

Spinal deformity is common in children with osteogenesis imperfecta, particularly in severe


forms. Curves develop early and generally progress. Bracing is not indicated due to the
complications associated with the brace on the soft bones of the rib cage. In the patient with
severe disease with thin bones and numerous fractures, posterior correction and fusion is
indicated, and should be done early when the curve is greater than 35 degrees. In the patient
with mild disease and thick bones surgery is indicated when the curve is greater than 45
degrees.

47. A
The clinical presentation is consistent for symptoms of myelopathy in a patient with
Osteogenesis imperfecta (OI). Basilar invagination is the most likely cause.

Osteogenesis imperfecta (OI) is a genetic disease resulting from mutations in type I collagen
genes causing bone fragility and deformities (including wormian bone appearance). Patients
with osteogenesis imperfecta are known to develop basilar invagination, defined as a protrusion
of the odontoid process into the foramen magnum. Basilar invagination is commonly seen with
Klippel-Feil syndrome, occipitocervical synostosis, achondroplasia, osteogenesis imperfecta,
Morquio syndrome, and spondyloepiphyseal dysplasia.

48. ​C
The question stem is describing a patient with osteogenesis imperfecta Type IV, which is
caused by a qualitative defect in the synthesis of Type I collagen. The quantitative disorders of
type I collagen are associated with milder forms of OI (Type I), whereas the qualitative disorders
are associated with more severe phenotypes (Types II, III and IV). Moderate clinical severity
with white sclera and dentinogenesis imperfecta, are classically associated with Type IV OI.
Lower extremity bowing and malunion are also characteristic of OI.

49. B

31
This clinical presentation is consistent with a "floating elbow" with displaced fractures of both the
elbow and and wrist. The most appropriate treatment is prompt closed reduction and pinning of
both the supracondylar humerus fracture and distal radius fracture.

Displaced floating elbow injuries in the pediatric patient are most appropriately treated with
prompt closed reduction and percutaneous pinning of both injuries to prevent the occurrence of
compartment syndrome prompted by casting. This combined pinning should occur even if one
of the injuries treated in isolation is most appropriately treated with closed reduction and casting
as the upper extremity is subject to a large amount of swelling not safe for a circumferential
cast.

50. E
The ulnar nerve is most commonly injured with this type of fracture, and is manifested with
intrinsic hand weakness.

In flexion type supracondylar humerus fractures, the distal fragment is displaced anteriorly.
Specifically, the sharp spine of the proximal fragment most commonly injures the ulnar nerve.
Extension type SCH fractures are more common than flexion type and are commonly
associated with injury to the anterior interosseous nerve (AIN). Weakness of the flexor digitorum
profundus to the index finger and flexor pollicis longus would be associated with AIN palsy.
Wrist drop would be associated with radial nerve palsy. Weakness of extensor pollicis longus
correlates with posterior interosseous nerve palsy. Ulnar nerve palsy would cause hand intrinsic
weakness and clawing.

51. B
Cubitus varus is typically caused by malreduction of the fracture at the time of fixation; not
usually by growth arrest.

52. B
Intoeing is a common, benign rotational variation seen in children. The 5 components of
Staheli's rotational profile are discussed in the review article by Lincoln and Suen. These
components include internal and external hip rotation, thigh-foot axis, transmalleolar axis,
heel-bisector, and foot progression angle. Normal values for clinical measurements are: foot
progression angle -5 to 20 degrees, IR and ER up to 70 degrees, and thigh-foot axis between
-10 to 20 degrees. "Red Flags" to prompt further evaluation include progressive deformity, limb
length discrepancy (possible hip dysplasia), extreme limb rotational profiles and pain (possible
malignancy or fracture). A positive family history for rickets, achondroplasia, skeletal dysplasias,
or mucopolysaccharidoses are also a cause for further evaluation. Metatarsus adductus with or
without internal tibial torsion is the most common cause of in-toeing in this age group.

32
SPORT ORTHO
1. A patient has shoulder pain and dysfunction. The radiograph is shown in Fig A. If this patient
undergoes shoulder arthroscopy, which structure is most likely to be abnormal?

A. supraspinatus
B. infraspinatus
C. glenohumeral articular cartilage
D. superior labrum
E. biceps tendon (long head)

2. A 73-year-old right-hand dominant female presents with the right shoulder injury. Arthroscopy of right
shoulder showed a U-shaped rotator cuff tear. She denies having any shoulder pain prior to a fall at work
after slipping on some water 4 weeks ago. She smokes a pack of cigarettes per week. Which of the
following characteristics of this patient confer the highest risk of not healing the injury following surgical
repair?
A. Pack of cigarette smoking per week
B. Surgical repair 4 weeks after injury
C. Worker's compensation case
D. 73 years of age
E. Right-handed dominance

3. A 45-year-old patient presents with pain and swelling after undergoing an arthroscopic rotator cuff
repair 10 weeks ago. On physical exam the portal sites are healed and there is no drainage. Testing of
the integrity of the rotator cuff is limited secondary to pain. He has a WBC of 11.0 (reference range, 3-11
cells/mL), ESR of 40 mm/hr (reference range, 0-22 mm/hr), and CRP of 1.5 mg/dL (reference range, 0-1
mg/dL). An aspiration is completed and no organisms are seen on the gram stain. Twelve days after the
aspiration, positive cultures are reported. Which organism is most likely to have grown in culture
medium?
A. Staphylococcus aureus
B. Propionibacterium acnes
C. Corynebacterium sp.
D. Staphylococcus epidermidis

33
E. Pseudomonas aeruginosa

4. A 55-year-old carpenter presents with 6 weeks of right shoulder pain after installing ceiling drywall. He
has no symptoms of night pain. His examination reveals 30 degrees lack of full flexion and abduction. He
has full strength of the right shoulder. MRI of right shoulder showed a partial articular-sided
supraspinatus tear . What is the next most appropriate step in management?
A. Physical therapy
B. Platelet rich plasma (PRP) injection
C. Arthroscopic rotator cuff repair
D. Arthroscopic SLAP repair
E. Arthroscopic subacromial decompression

5. A 64-year-old male suffers a fall while working on his farm and presents to the ER with the shoulder
injury. AP X-ray radiograph showed an anterior dislocation of left glenohumeral joint. He undergoes
reduction without complications, post-reduction radiographs show no bony injury. At his 10 day clinic
follow-up is noted to have an inability to abduct his arm. Which of the following studies will best confirm
the most likely diagnosis in this patient?
A. MRI of the shoulder
B. EMG
C. CT-angiogram of the affected extremity
D. Repeat shoulder x-rays
E. MRI of the brachial plexus

6. A 50-year-old man sustains a left shoulder injury after falling from a motorcycle. A physical
examination test to examine for this shoulder injury is found in Figure A. What is the most likely
diagnosis?

A. SLAP tear
B. Supraspinatus tear
C. Infraspinatus tear
D. Teres minor tear
E. Subscapularis tear

7. A 65-year-old right-hand-dominant man reports acute right shoulder pain and inability to lift his arm
overhead after a glenohumeral dislocation while skiing 2 weeks ago. Physical exam reveals active
forward elevation to 30 degrees and 3/5 external rotation strength, pain with motion, and intact lateral
arm sensation. An MRI is contraindicated due to a pacemaker, and therefore an arthrogram is performed

34
and showed extravasation of the dye into the subacromial space with no evidence of arthritis. What is the
most appropriate treatment option?
A. Shoulder hemiarthroplasty
B. Rotator cuff repair
C. Proximal humerus ORIF
D. Total shoulder arthroplasty
E. Sling immobilization

8. ​Which of the following structures shares the same origin site as the tendon that undergoes
angiofibroplastic hyperplasia during the pathogenesis of tennis elbow?
A. Brachioradialis
B. Anconeus
C. Annular ligament
D. Flexor carpi ulnaris
E. Palmaris longus

9. A 50-year-old carpenter has chronic pain over the lateral aspect of the elbow. He notes pain
when using a hammer. On exam, he has pain with resisted wrist extension while the elbow is
fully extended. Which muscle attachment is likely to be involved?
A. Distal biceps brachii
B. Brachioradialis
C. Extensor carpi radialis brevis
D. Extensor carpi radialis longus
E. Supinator

10. Which of the following bone bruise patterns seen on magnetic resonance imaging (MRI) is
most consistent with an anterior cruciate ligament (ACL) tear?
A. Medial tibial spine and medial femoral condyle
B. Medial facet of patella and lateral femoral condyle
C. Posterolateral tibia and lateral femoral condyle
D. Posterolateral tibia and medial femoral condyle
E. Medial tibial spine and lateral femoral condyle

11. ​A 16-year-old high school basketball player sustains a non-contact knee injury when she
lands from a rebound. She develops immediate swelling and is noted to have a hemarthrosis.
What is the most likely diagnosis?
A. MCL tear
B. Medial meniscus tear
C. ACL tear
D. Patellar dislocation
E. Contusion

35
12. ​A 25-year-old male soccer player twisted his left knee 4 days ago and developed immediate
swelling and pain. On exam, he has a 2+ effusion and pain with active range of motion.
Passively, he tolerates range of motion from 0-90 degrees. He has difficulty performing a
straight leg raise exercise. MRI scan shows an ACL tear What is the most appropriate initial
management for his injury?
A. Knee immobilization
B. Physical therapy for range of motion
C. Acute reconstruction followed by mobilization
D. Arthrocentesis to rule out infection
E. Rest, nonsteroidal anti-inflammatories, and follow-up in 4 weeks

13. A high school girls basketball player sustains a non-contact knee injury and develops an
acute hemarthrosis. What is the likelihood that she has an ACL tear?
A. 0-15%
B. 15-30%
C. 30-45%
D. 45-60%
E. >60%

14. You are called by a 35-year-old male patient who had ACL reconstruction with hamstring
autograft 5 days ago. He reports his knee pain and swelling have significantly increased in the
last day, and now it is difficult for him to raise his leg off the bed and is having more difficulty
tolerating the continuous passive motion (CPM) machine. Upon questioning he denies fever,
chills, or any new trauma to the knee. What is the next step in management?
A. Ice, NSAIDS, elevation, compression wrap and restart therapy once symptoms improve
B. Go to the ER immediately for knee aspiration with gram stain and cultures
C. Increase CPM use to 10 hours a day
D. Call the office staff in the morning to schedule an appointment
E. Start physical therapy visits once daily

36
15. Which of the following physical exam maneuvers would be MOST expected for a patient
with the following radiograph?

A. Positive Lachman's test


B. Positive McMurray's test with leg internally rotated
C. Positve McMurray's test with leg externally rotated
D. Positive external rotation dial test with knee flexed at 30 degrees
E. Positive external rotation dial test with knee flexed at 30 degrees and 90 degrees

16. An 18-year-old football player sustained a twisting injury to his knee approximately 1 month
ago. He complains of continued knee pain with occasional locking and catching.T1 sagittal MRI
shows a medial meniscus tear. What physical exam finding is classically seen with this injury?
A. No endpoint with valgus stressing of the knee
B. Positive apprehension sign with passive lateral patellar translation
C. Painful click is elicited as the knee is brought from flexion to extension with internal or
external rotation
D. No endpoint with varus stressing of the knee
E. A positive posterior drawer and quadriceps active test

17. An 18-year-old competitive tennis player sustains a twisting injury to his knee. He develops
pain and swelling and is unable to straighten his knee. MRI showed a large bucket-handle tear
of lateral meniscus. What is the most approriate treatment?
A. Arthroscopic medial meniscectomy or repair
B. Arthroscopic lateral meniscectomy or repair
C. Meniscus transplantation
D. Physical therapy with gradual stretching exercises
E. Corticosteroid injection for acute inflammation

37
18. A 13-year-old male patient presents with complaints of bilateral knee pain. He notes that
the pain is increased with jumping, squatting or running up a flight of stairs. No other joints are
affected. On physical exam, he has tenderness to palpation directly over his tibial tubercles. No
effusion is noted. Radiographs are shown in Figures A and B. What is the most likely diagnosis?

A. Lyme disease
B. Stress fractures of the tibial plateau
C. Osgood-Schlatter disease
D. Patellar tendinopathy
E. Sinding-Larsen-Johansson syndrome

19. Which area of the knee is most likely to be affected by a juvenile osteochondritis dissecans
(JOCD) lesion?
A. Lateral aspect of the medial femoral condyle
B. Lateral aspect of the lateral femoral condyle
C. Medial aspect of the lateral femoral condyle
D. Medial facet of the patella
E. Lateral facet of the patella

20. A 10-year-old boy has atraumatic, progressive right knee pain for 2 months. He denies
fevers or mechanical knee symptoms. His exam is completely normal and symmetric to his left
knee. Radiographs of the right knee demonstrate open growth plates and a well circumscribed
1x1cm area of sclerotic subchondral bone with a radiolucent halo separating this area from his
femoral epiphysis. What is the best initial treatment plan?
A. Arthroscopic micro-fracture
B. Activity modification
C. Antegrade drilling
D. Arthroscopic reduction and fixation
E. Arthroscopic excision

38
21. A 22-year-old college pole vaulter has chronic anterior pain on her jumping knee. The
patient has had over 6 months of physical therapy without improvement. It initially only bothered
her during training, but she is now no longer able to compete and has pain with daily activities.
Her MRI shows increased signal intensity over her right patellar tendon with thickening. What is
the recommended treatment?
A. Use of a Chopat strap
B. Intra-tendinous injection of corticosteroid
C. Intra-articular corticosteroid injection
D. Arthroscopic patellar chondroplasty
E. Surgical excision of the affected tissue

39
ANSWER:
1. ANS: A
The radiograph shows calcific tendinitis of the supraspinatus tendon. This is the most common structure
involved in this disease process.

Uthoff describes stages of calcific tendonosis. The "Pre-calcific" stage shows metaplasia of tenocytes
into chondrocytes. The "Calcific" stage has three sub-parts: Formative phase, Resting phase, Resorptive
phase. The "Post-calcific" stage is last.

Two types are seen on X-Ray: Type I has as fluffy and fleecy appearance with a poorly defined
periphery. This is associated with an acute pain and is usually seen during the formative phase of the
calcific stage. Type II is characterized by discrete homogeneous deposits with uniform density and a well
defined periphery. This is seen in subacute and chronic cases. Persistent cases may respond to
arthroscopic debridement of the deposits with a shaver or spinal needle.

2. ANS. D
Patient age older than 65 is the highest risk factor for nonhealing of the surgically repaired rotator cuff.

Relative surgical indications for rotator cuff tears include acute (<3 months from time of injury), traumatic
tears in patients younger than age 60 years old. Additionally, full-thickness and partial-thickness rotator
cuff tears that fail nonsurgical treatment are relative indications for surgical intervention. Advanced fatty
infiltration and muscle atrophy as detected on MRI and significant glenohumeral arthritis are relative
contraindications for rotator cuff repair.

3. Ans:
Based on the delayed clinical presentation and laboratory findings, the most likely bacterium to have
grown in culture is Propionibacterium acnes (P. acnes).

Infections after arthroscopic rotator cuff repairs have a reported prevalence of 0.006%-3.4%. The typical
skin flora includes staph and strep as well as P. acnes, which has a propensity for the shoulder. Because
it is an anaerobic organism, cultures may only become positive after 7-21 days.

4. Ans: A
The history, examination, and imaging are consistent with a partial articular-sided supraspinatus tendon
avulsion (PASTA). Physical therapy including shoulder range of motion and rotator cuff/periscapular
stabilizer strengthening is the most appropriate initial treatment for the options provided.

Partial-thickness rotator cuff tears are most commonly classified by location (articular- or bursal-sided)
and size (greater or less than 50% thickness). If conservative treatment options fail, then
partial-thickness, articular-sided rotator cuff tears >50% can be treated with completion and repair (open
or arthroscopic). If the tear is <50% then treatment consists of débridement of the tuberosity and
undersurface rotator cuff.

40
5. Ans: A
This patient is most likely suffering from a massive rotator cuff tear which is accounting for his inability to
raise his arm. The incidence of rotator cuff tear after acute dislocation in patients older than age 40
ranges from 35% to 86%. Axillary nerve injury can happen after acute shoulder dislocations, but usually
are not severe enough to account for the significant weakness noted in this patient. Brachial plexus
injuries are rare, and vascular injuries are associated with ischemic changes to the affected extremity.
Shoulder fracture is unlikley in this case as the post-reduction radiographs show no bony injury.

6. Ans. E
The concept tested in this question is whether you know what physical exam finding is associated with a
subscapularis tear. The key physical exam findings are positive Belly-press and Lift-off maneuvers, as
well as weakness in internal rotation and increased passive external rotation. The MRI confirms the
diagnosis with discontinuity of the subscapularis and the long-head of the biceps not located in the
bicipital groove.

7. Ans: B
The clinical presentation is consistent with an acute rotator cuff tear following a shoulder dislocation, so
the most appropriate treatment is a rotator cuff repair.
A shoulder dislocation in a patient >40 years-old commonly results in a rotator cuff tear. An arthogram
may be helpful to confirm the diagnosis when an MRI is contraindicated. A rotator cuff tear allows the
dye to leak into the subacromial space, whereas in a normal MRI arthrogram the dye is contained within
the joint capsule.

8. B
Lateral epidondylitis is classically thought to be caused by histopathologic angiofibroblastic
hyperplasia at the origin of the extensor carpi radialis brevis. ECRB originates from the common
extensor wad, that also includes ECRL, ED, ECU. The anconeus shares the same attachment
site at the lateral epicondyle as the ECRB.

9. C
The clinical presentation is consistent with lateral epicondylitis, which is caused from pathologic
changes at the origin of the extensor carpi radialis brevis (ECRB).

Physical exam findings consistent with lateral epicondylitis include tenderness over the lateral
epicondyle at the origin of the ECRB, and pain that is reproduced with gripping, resisted long
finger extension, resisted wrist extension while the elbow is fully extended, and maximum
passive wrist flexion. This should be distinguished with the pain with resisted supination with the
arm and wrist in extension characteristically seen with radial tunnel syndrome.

10. C
Bone bruising occurs in more than half of all ACL tears and is seen most commonly on the
posterolateral tibia and middle 1/3 of the lateral femoral condyle (sulcus terminalis).

41
ACL tears usually occur as a result of a non-contact pivoting injury. Bone bruise patterns
correlate with the direction of the abnormal anterior translation and abutment of the
posterolateral tibia against the middle 1/3 of the lateral femur during the injury.

11. C
This is the classic history for an ACL tear. Women's basketball has one of the highest rates of
ACL tears. While all of the answers are possible, the incidence of ACL tears in adolescents with
an acute knee injury with hemarthrosis is the highest.

12. B
The clinical presentation, physical exam, and imaging are consistent with an acute anterior
cruciate ligament (ACL) tear. If the patient wants to return to sports requiring twisting/pivoting,
an ACL reconstruction is recommended. He currently has an acute effusion (hemarthrosis) with
decreased motion. Acute ACL reconstructions in patients with limited range of motion and
weakness have been shown to lead to postoperative arthrofibrosis and weakness.

The other answers would not optimize postoperative results. Immobilization would promote
arthrofibrosis, as would an acute reconstruction in this stiff, weak knee. Arthrocentesis is a
reasonable diagnostic/therapeutic option acutely. However, it does expose the patient to a risk
of infection. Furthermore, in this patient with a 4 day old injury, the hemarthrosis is likely mostly
coagulated and would frustrate attempts at aspiration. Rest and NSAIDs would not help to
promote range of motion and strength.

13. E
The classic scenario is a non-contact deceleration, jumping or cutting action. The patient might
hear or feel a "pop". The acute hemarthrosis is caused by bleeding from branches of the middle
geniculate artery. Women's basketball has one of the highest rates of ACL injury. With the
above history, the literature states that the likelihood of ACL injury is greater than 70%.

14. B
Any patient who presents with a sudden increase in knee effusion in a delayed manner after
ACL surgery should raise suspicion for infection, whether or not a fever is present. If suspected,
an aspiration should be performed immediately and fluid sent for gram stain and cultures. If
positive, immediate arthroscopy is indicated.

15. A
The radiograph shows an example of a Segond fracture, most commonly caused by an anterior
cruciate ligament (ACL) injury. An ACL injury would correspond best with a positive Lachman's
test on physical examination.

42
16. C
The MRI shows a medial meniscus tear which can be clinically tested using a McMurray test.
McMurray's test for medial meniscal pathology consists of: flexing the knee, applying a valgus
force, placing a hand on the medial joint line of the knee, and finally bringing the knee from
flexion to extension while rotating the tibia. A palpable pop or click, or more commonly the
elicitation of pain during this maneuver is considered a positive test and can correlate with a
meniscal tear.

17. B
The clinical presentation is consistent for an acute bucket-handle lateral meniscus tear. He has
a locked knee, meaning that he cannot fully extend because the meniscus has flipped into the
notch. Arthroscopic lateral meniscectomy or repair is the treatment of choice. A repair is usually
possible in large bucket-handle tears because the meniscus is torn in the red-red zone where
most of the vascular supply is located. If the torn portion is badly frayed or damaged, a partial
meniscectomy may be performed. Passive stretching in answer D may further damage the
meniscus.

18. C
Based on the history and clinical findings, the most likely diagnosis is Osgood-Schlatter disease.

Osgood-Schlatter disease is a traction apophysitis of the tibial tubercle that commonly affects
males between the ages of 12-15. Bilateral involvement is found in 20-30% of patients. The
condition responds well to conservative measures, including anti-inflammatories, resting,
strapping of the tibial tubercle and quadriceps stretching. Ninety-percent of patients will have
complete recovery with these measures.
Incorrect Answers:
Answer A: While Lyme disease can affect the knee, it is likely to produce an isolated knee
effusion. Patients often have difficulty with weight bearing or with range of motion.
Answers B, D, E: While these choices could cause knee pain, they are much less likely given
the history and radiographic findings.

19. A
More than 70% of JOCD lesions are found in the “classic” area of the posterolateral aspect of
the medial femoral condyle, with inferior-central lateral condylar lesions accounting for only 15%
to 20% of cases and femoral trochlear lesions seen in less than 1%. The knee is the most
common site of osteochondrosis in growing children, which is seen in an estimated 0.002% to
0.003% of knee radiographs.

20. B
This child has osteochondritis dissecans (OCD) of lateral femoral condyle with open growth
plates. The cited review by Schenck and Goodnight concluded that the outcomes of distal femur
OCD in skeletally immature patients are good and these lesions usually heal without surgical

43
treatment. Limitation of activity may diminish stresses across the OCD lesion and prevent
displacement. In adult patients, the rate of non-operative healing is very low. Other indications in
this child for operative intervention would be failure of non-operative treatment, mechanical
locking from a loose body, or radiographic evidence of a displaced fragment.

21. E
The clinical presentation and imaging studies are consistent with Blazina Stage III patellar
tendonitis. This is supported by the chronic inflammation and mucoid degeneration seen within
the patellar tendon on the MRI. Therefore, surgical excision of the affected tissue is indicated.

44
DIVISION OF JOINT RECONSTRUCTION
1.Which of the following is the most common intraoperative complication in a patient with sickle
cell disease undergoing a total hip arthroplasty?
A. Periprosthetic fracture distal to the implant
B. Iatrogenic fracture causing pelvic discontinuity
C. Perforation of the femoral canal
D. Cardiac arrest from fat embolization to lungs
E. Injury to the sciatic nerve

2. A 65-year-old patient was treated with an open reduction/internal fixation for a left femoral
neck fracture sustained 25 years ago. Five years ago he developed hip pain and was converted
to a left hip hemiarthroplasty. He presents with complaints of groin pain for the past 6 weeks. A
recent radiograph is shown in Figure A. The patient’s physical exam is limited secondary to
pain. Serum laboratory values are WBC-8.0, ESR-20, CRP-0.5. A synovial fluid aspirate of the
hip demonstrates < 500 cells (60% PMN). What is the most likely cause of this patient's
symptoms?

A. Acetabular protrusio
B. Infected hip hemiarthroplasty
C. Lumbar radiculopathy
D. Impingement of the hip hemiarthroplasty
E. Iliopsoas tendinitis

3. ​A 47-year-old man presents with 1 week of left leg pain. 6 months prior he underwent a
vascularized free-fibula bone graft from his left leg to his right hip for avascular necrosis. The
pain is located at the level of his donor site and is worse with weight-bearing and relieved by
rest. Physical exam shows focal tenderness over his tibia. A lateral radiograph from the day of
presentation is shown in Figure A. WBC, ESR, and CRP are all within normal limits. What is the
next best step in management to confirm the diagnosis?

45
A. Compartment pressure measurements
B. CT scan
C. MRI scan
D. Ultrasound to rule out deep abscess
E. Bone biopsy

4. A 40-year-old man complains of increasing groin pain. Radiographs show femoral head
avascular necrosis with subchondral lucency but without femoral head collapse. Which of the
following medical treatments have been shown to decrease the risk of subsequent femoral head
collapse?
A. Cyclic parathyroid hormone therapy
B. Bisphosphonate therapy
C. RANK ligand therapy
D. RANK therapy
E. Selective estrogen receptor modulator therapy

5. A 41-year-old male has steroid-induced avascular necrosis of the hip and decides to undergo
metal on polyethylene total hip arthroplasty. His 80-year-old, sedentary father had a total hip
replacement 5 years ago. With comparison to his father, the patient should be informed of the
following risk?
A. Increased risk of sciatic nerve palsy
B. Increased longevity of prothesis
C. Increased risk for polyethylene wear and osteolysis
D. Reduced range of motion
E. Lower likelihood of revision surgery

6. Which of the following non-operative treatments for osteoarthritis has the best evidence to
support its use?
A. Combination of supervised and home exercise programs
B. Hyaluronic acid injections
C. Lateral heel wedge

46
D. Acetaminophen
E. Glucosamine

7. According to the 2011 American Academy of Orthopaedic Surgeons' Guidelines for the
treatment of symptomatic osteoarthritis of the hip or knee, which of the following
recommendations was most strongly supported?
A. Acupuncture
B. Hyaluronic acid injections
C. Intraarticular corticosteroid​s
D. Tramadol for purposes of pain alleviation
E. Glucosamine

8. According to the latest recommendations made by the AAOS in their clinical guidelines for the
treatment of osteoarthritis (OA) of the knee, which of the following nonoperative treatment
modalities has the weakest supporting evidence for the treatment for knee osteoarthritis?
A. Weight loss
B. Activity modifications
C. Quadriceps strengthening
D. Intra-articular hyaluronic acid injections
E. Intra-articular corticosteroid injections

9. A 68-year-old woman underwent a right total knee arthroplasty 5 years ago and has
increasing right knee pain over the past 2 months. Radiographs are seen in Figures A and B,
respectively. Laboratory studies demonstrate a C-reactive protein of 10 mg/dL (normal < 2.0
mg/dL) and an erythrocyte sedimentation rate of 50 mm/h (normal < 20 mm/h). Knee aspiration
shows white blood cell count of 3,400/mm3 with 90% polynuclear cells. The patient's gram stain
and cultures are negative. What is the most appropriate next step in management?

47
A. Irrigation and debridement with polyethylene spacer exchange
B. One-stage revision
C. Two-stage revision
D. One-stage revision with antibiotic impregnated cement
E. One-stage revision with direct antibiotic infusion into knee joint via hickman catheter

10. A 72-year-old man reports persistent, progressively worsening pain in his hip after
undergoing a total hip arthroplasty 15 months ago. A current AP hip radiograph is shown in
Figure A. What is the next most appropriate step in the care of this patient?

A. IV Antibiotics
B. Obtain serum metal ion values
C. Obtain ESR, CRP, and WBC
D. Obtain CT and MRI of the hip
E. Urgent debridement and component explantation

48
11. A 50-year-old woman underwent cemented total knee arthroplasty 3 weeks ago. She reports
that she has 1 week of drainage the size of a quarter on a gauze pad that she places over the
incision three times daily. Her body mass index is 53 and her medical problems include
hypertension and type 2 diabetes. Blood work shows a CRP of 1.1mg/L (normal 1-3mg/L). Knee
aspiration yields WBC of 673 cells/mm(3) with 30% polymorphonucleocytes, and a negative
gram stain. There is no surrounding erythema but there is a 1cm area at the inferior aspect of
the wound that has a large amount of serous drainage able to be expressed. She has a painless
range of motion is 0° to 117°. What would be the next most appropriate step in management?\
A. Removal of all components with antibiotic spacer placement and staged revision
B. One-stage irrigation and debridement with removal of components to a cementless
prosthesis
C. Empiric oral antibiotics for 4 weeks and steri-strips over the area of drainage
D. Surgical exploration with debridement and possible polyethylene exchange
E. Bone scan and repeat aspiration with empiric intravenous antibiotics for 4 weeks

12. A 65-year-old male presents with a painful right total knee arthroplasty, which was
performed ten years ago. CRP is 15 mg/L. Knee aspiration reveals a purulent fluid with 2,100
WBC's with 78% PMN's. Culture results are pending. Which of the following is the best
management option?
A. Physical therapy, ice, and follow-up evaluation in 2 weeks
B. Repeat aspiration if cultures are positive
C. Oral antibiotics
D. Intravenous antibiotics
E. Surgical explant of components

13. A 65-year-old male who had a total knee arthroplasty 8 years ago comes into the office with
worsening knee pain. The orthopaedic surgeon is concerned about infection and aspirates the
knee. Which of the following are the lowest laboratory values from a synovial aspirate
suggestive of infection?
A. WBC of 500 cells/ml and PMN 25%
B. WBC of 1,000 cells/ml and PMN 25%
C. WBC of 1,500 cells/ml and PMN 70%
D. WBC of 5,000 cells/ml and PMN 70%
E. WBC of 25,000 cells/ml and PMN 70%

14. A 67-year-old diabetic male presents 4 months status post right total knee arthroplasty
(TKA) complaining of pain and stiffness for the last four weeks. A clinical photograph is shown in
Figure A. Radiographs and a bone scan are shown in Figures B, C and D. Blood work shows an
ESR of 14mm/hr (normal 0-12mm/hr) and a CRP of 2mg/L (normal 1-3mg/L). Knee aspiration
yields WBC of 1000, 30% PMNs, and a negative gram stain. He finished a 14-day course of
antibiotics prescribed to him by his primary care physician one week ago. Which of the following
is the most appropriate next step in management

49
A. Broad-spectrum, empiric oral antibiotics
B. Repeat aspiration after one week
C. Irrigation and debridement of the right knee with a polyethylene liner exchange
D. One-stage irrigation and debridement of the right knee with a component exchange
E. Two-stage component removal, antibiotic spacer placement and subsequent revision

50
ANSWER:
1.ANS: C
Perforation of the femoral canal during preparation of the femur is not an uncommon
complication, with rates ranging from 4.9-18.2%.

While total hip arthroplasty is extremely effective for pain relief in patients with
osteonecrosis of the hip secondary to sickle cell disease, the procedure carries a higher
rate of complications compared with non-sickle cell disease patients. Particular attention
should be given to the preparation of the femur as femoral medullary widening from chronic
marrow hyperplasia adjacent to patchy areas of dense sclerosis can make preparation of
the canal difficult. Some surgeons prefer to ream over a guide-wire to avoid perforation.
Periprosthetic fracture usually occurs at the area of perforation, not distal to the implant.
Answer B; Acetabular fractures are more common in this patient population as well, but the
rate of iatrogenic pelvic discontinuity is lower than that of femoral perforation.
Answer D: The rate of cardiac arrest from fat embolization to the lungs is quite low.
Answer E: While injury to the sciatic nerve is possibly, it has not been shown to be more
common in this patient population. The rate of post-operative hematoma causing sciatic
nerve dysfunction may be higher in this patient population.

2. ANS: A
Based on the history, radiographs, and laboratory values, the patient has developed failure of
his hip hemiarthroplasty. At this point in time he warrants a conversion to a total hip arthroplasty.
Avascular necrosis (AVN) of the femoral head after traumatic injury to the femoral neck occurs
at an incidence of 10-45%. Although the risk increases with failure to anatomically reduce the
fracture, it can still occur in non displaced settings. Treatment of avascular necrosis in older
patients includes hip hemiarthroplasty or a total hip replacement. With the former, development
of acetabular protrusio can contribute to groin symptoms. Functional outcomes have been
reported to be higher in those receiving total hip replacement for AVN of the femoral head.
While impingement could be a cause of pain, it is not as likely given the history, clinical findings
and radiographs seen here.

3. ANS: C
The clinical presentation is suspicious for a stress fracture of the tibia following free-fibula bone
grafting. If plain radiographs are negative, more sensitive imaging such as a MRI or bone scan
should be performed.
Tibial stress fractures are a known complication following free-fibula bone grafting. Radiographs
may be normal (as is the case in figure A), or might show the "dreaded black line" and/or new
periosteal bone formation. If a stress fracture is confirmed with imaging, appropriate
management would then consist of protective weight bearing until symptoms subside.

51
4. ANS: B
Bisphosphonate therapy is a proven method of preventing femoral head collapse in patients
with avascular necrosis and subchondral lucency.

5. ANS: C
A younger active patient will sustain more polyethylene wear and osteolysis due to greater
activity levels and more years of use.

6. A
Of the options listed, a combination of home and supervised exercise has the best supporting
evidence for the treatment of osteoarthritis.

The AAOS has recently developed guidelines for the treatment of osteoarthritis. Therapies that
are recommended by the AAOS include weight loss, home and supervised exercise programs,
and NSAIDs/tramadol. Therapies that remain inconclusive (lack of supporting evidence) include
electrotherapeutic modalities, manual therapy, bracing, acetaminophen/opioid, steroid injections
and PRP. Glucosamine, lateral heel wedges and hyaluronic acid injections are not
recommended, as current literature has shown them to be ineffective. Keep in mind that these
guidelines are subject to change as new literature is published.

7. D
The only recommendation that was supported in the above mentioned answer choices was the
use of NSAIDs or tramadol for patients with symptomatic osteoarthritis of the knee.

8. D
Intra-articular hyaluronic acid is no longer recommended as an effective method of treatment for
patients with symptomatic knee arthritis based on the revised AAOS clinical guidelines from
2013. The previous review from 2009 guidelines was reported as inconclusive.

Nonoperative treatment modalities whose use is supported by the literature include: activity
modifications, weight loss, quadriceps strengthening, patellar taping, NSAIDs, tylenol (Now
3000mg/24hr is recommended from 4000mg/24hr), and intra-articular steroids. Treatment
options that are NOT supported by the literature (or are considered inconclusive) include:
intra-articular hyaluronic acid injections, lateral heel wedges for medial knee OA, glucosamine
and chondroitin, needle lavage, and arthroscopy in patients with primary OA.

9. C
The patients history, labs, and imaging are consistent with an infected total joint prosthesis.
Two-stage resection and replacement arthroplasty for hip and knee arthroplasty is the gold
standard for treatment of infection beyond 4 weeks. Reimplantation within 2 weeks has a 35%
success rate compared to success rates of 80% with delayed reimplantation (>6 weeks) and
more extensive antimicrobial therapy.

52
10. C
The key to this question is recognizing the radiographic findings of periprosthetic infection.
Figure A shows new, lacey periosteal bone formation about the metadiaphyseal region of the
femur with scalloping resorption. This is suggestive for a deep periprosthetic infection. Initial
work-up starts with ESR, CRP and WBC. If these are elevated, joint aspiration to confirm
periprosthetic infection is warranted.

11. D
Irrigation and débridement with possible polyethylene exchange is the most appropriate
treatment for persistent drainage within a few weeks from total joint arthroplasty surgery. ( in this
case, 3 weeks) illustration below shows the different part of a prosthetic implant for knee.

12. E
This patient is infected and requires explantation to cure the infection.

13. C
WBC of 1,500 cells/ml and PMN 70% indicates the lowest synovial aspirate suggestive of
infection.

14. B
The clinical scenario describes a patient with an equivocal presentation of a periprosthetic joint
infection (PJI) and recent history of antibiotic use. As such, a repeat aspiration in one week is
indicated.

The work-up of a suspected PJI after TKA includes an evaluation of radiological (x-ray +/- bone
scan and PET scan) and laboratory (ESR and CRP) parameters as well as analysis of joint
aspirate fluid (cell count and differential, culture, gram stain +/- PCR).

53
Figure A is a clinical photograph demonstrating a swollen, erythematous right knee with a
well-healed incision from a previous TKA. Figure B and C are AP and lateral radiographs of the
right knee with no obvious acute findings. Figure D is a bone scan demonstrating increased
uptake in the post-operative knee, which is consistent with the 4 month follow-up.
Incorrect Answers:
Answer A, C, D and E: Broad-spectrum antibiotics, I & D +/- liner exchange, one stage and two
stage revision would not be appropriate at this time point as the diagnosis remains unclear.

54
TRAUMA

1. A 41-year-old female sustains the injury shown in Figure A as a result of a high-speed motor
vehicle collision. After a successful attempt at closed reduction in the emergency room using
conscious sedation, repeat radiographs show a reduced hip joint. What is the next most
appropriate step in treatment?

A. Femoral skeletal traction


B. CT scan of hip and pelvis
C. Dynamic fluoroscopic examination under general anesthesia
D. Hip spica dressing
E. Touch down weight bearing mobilization

2.A 30-year-old driver is involved in a motor vehicle collision and sustains the injury shown in
Figure A. What is the most likely concomitant injury?

A. Right knee meniscus tear


B. Left knee ACL tear
C. Subdural hematoma
D. Right ankle fracture-dislocation
E. Lumbar burst fracture

55
3. A 7-year old boy presents to the emergency room following a ATV accident with complaints of
left pelvic pain. In the emergency room he is alert and oriented and is hemodynamically stable.
On physical exam he is unable to bear weight on his left lower extremity. There is no tenderness
to palpation at the posterior pelvis. A radiograph is performed and shown in Figure A and CT
examination shows the posterior ring is stable and age-appropriate. What is the most
appropriate treatment for this injury pattern?

A. Nonoperative management with weight bearing as tolerated


B. Percutaneous sacroiliac screw
C. Pelvic external fixation
D. Anterior pelvic ring plating
E. Anterior and posterior pelvic ring plating

4. Which of the following is an appropriate initial step in the management of a multiply injured
patient with an unstable pelvic ring fracture and hemodynamic instability?
A. Application of an external fixator
B. Pelvic angiography
C. Pelvic packing
D. Application of a pelvic binder
E. Percutaneous Iliosacral screws

5. A 37-year-old male is struck by a car while walking at night. He is hemodynamically unstable


at initial evaluation in the trauma bay. Advanced Trauma Life Support protocols are started, and
an initial survey is completed. A chest radiograph and a pelvis AP radiograph (Figure A) are
obtained. What is the most appropriate next step?

56
A. The patient should be taken directly to the OR for percutaneous placement of a pelvic
external fixator
B. Dedicated inlet and outlet views of the pelvis to better classify the fracture
C. Continued resuscitation and immediate CT of the chest, abdomen and pelvis
D. Emergent trip to interventional radiology for pelvic embolization
E. Immediate application of pelvic binder, continued resuscitation and re-evaluation of
hemodynamic status

6. A 23-year-old female is an unrestrained driver in a motor vehicle collision, sustaining the


injury shown in Figure A. She subsequently undergoes reduction and percutaneous bilateral
iliosacral screw placement. Which of the following is the most likely neurologic complication
associated with percutaneous iliosacral screw insertion?

A. Weakness in knee extension


B. Decreased patellar reflex
C. Weakness in great toe extension
D. Weakness in ankle plantar flexion
E. Decreased Achilles reflex

7. A 35-year-old male involved in a high-speed motor vehicle collision presents to the trauma
bay hypotensive and with a clinically unstable pelvis. A pelvis radiograph is shown in Figure A.

57
He is placed in a pelvic binder, and his blood pressure normalizes temporarily. An abdominal
CT demonstrates free fluid and air in the intraperitoneal cavity, and a laparotomy is indicated.
What is the most appropriate next step in orthopaedic management?

A. Percutaneous SI screw placement


B. External fixation placement
C. Pubic symphysis plating
D. Posterior pelvic plating
E. Anterior sacroiliac plating

8. A 34-year-old female presents to the trauma bay with hemodynamic instability following a
motor vehicle collision. A chest radiograph shows a left-sided hemothorax and her pelvis
radiograph is shown in Figure A. Which of the following is the next most appropriate step in
managment?

A. Circumferential pelvic sheeting


B. Retrograde urethrogram to evaluate for associated urologic injury
C. Emergent transport to OR for pelvic anterior external fixator placement
D. CT scan to assess for occult femoral neck fracture
E. Bedside posterior pelvic C-clamp application

58
9. A 25-year-old male is involved in a motor vehicle accident and sustains the opened book
pelvic fracture. He is intubated in the field and receives 2 liters of ringer lactate and continues to
be tachycardic and hypotensive. A massive transfusion protocol is initiated. Which of the
following is true regarding the transfusion of packed red blood cells, platelets, and fresh frozen
plasma?
A. PRBC should be transfused until Hgb>8
B. PRBC, platelets, and FFP should be transfused in equal ratios
C. Platelets and fresh frozen plasma should be given when INR >1.4, platelet count
<100,000
D. FFP is not needed unless INR>1.5
E. Platelets should not be transfused unless platelet count <10,000

10. A 24-year-old male sustains the injury seen in Figure A after being thrown from a
motorcycle at a high speed. Which of the following fixation methods has been shown to be the
most stable fixation construct for this injury?

A. Posterior bridge plating and anterior ring external fixation


B. Percutaneous iliosacral screw and anterior ring external fixation
C. Percutaneous iliosacral screw and anterior ring internal fixation
D. Transiliac screw
E. Two percutaneous iliosacral screws

11. For a patient with an unstable pelvic fracture, the amount of blood tranfusions required in the
first 24 hours has shown to be most predictive for what variable?
A. Length of hospital stay
B. Association with neurological deficit(s)
C. Length of intensive care stay
D. Cardiac collapse
E. Mortality

59
12. A 32-year-old male is involved in a motor vehicle collision and sustains an opened book
pelvic fracture. What is the most common urological injury associated with this injury pattern?
A. Testicular torsion
B. Posterior urethral tear
C. Bladder denervation
D. Testicular rupture
E. Renal hematoma

13. A 27-year-old woman gives birth by normal spontaneous vaginal delivery. Two weeks after
delivery she reports anterior pelvic pain and a radiograph of pelvis showed symphyseal
diasthesis of 3.4cm. What is the next step in management?
A. Pelvic external fixator
B. Open reduction and reconstruction plating of the symphysis
C. Protected weightbearing and binder as needed and observation
D. Open reduction and wiring of the symphysis
E. Symphysiotomy

14. A 40-year-old male laborer sustained a fall from height and has isolated pelvic pain. He is
otherwise hemodynamically stable. Pelvic radiograph AP showed a symphyseal diasthesis of
2.1cm. A stress examination under anesthesia does not show any further anterior diastasis or
posterior pelvic ring displacement. Computed tomography reveals no asymmetry of the
sacroiliac joints. What is the most appropriate management of this injury?
A. protected weight-bearing and pain control
B. open reduction and internal fixation
C. skeletal traction followed by open reduction and internal fixation
D. pelvic external fixation
E. pelvic external fixation followed by sacroiliac screws

15. A 31-year-old male sustains an ipsilateral displaced transverse acetabular fracture, pubic
rami fractures, and a sacroiliac joint dislocation. What structure should be reduced and
stabilized first?
A. Pubic rami
B. Posterior column
C. Anterior column
D. Sacroiliac joint
E. Quadrilateral plate

16. A computed tomography (CT) scan has been shown to be indicated for evaluation of all of
the following aspects of acetabular fractures, EXCEPT:
A. Determination of surgical planning
B. Intra-articular loose bodies
C. Marginal impaction

60
D. Fracture piece size and position
E. Determination of pre-existing degenerative changes

17. An 18-year-old male sustains a right hip injury after being tackled on the football field. Figure
A shows his radiograph upon presentation to the emergency room three hours later. On
physical exam, he is noted to have a foot drop and decreased sensation globally throughout his
entire lower leg. Closed reduction under conscious sedation is immediately performed, and the
hip is able to be ranged through a stable arc of motion following reduction. A post-reduction
radiograph is shown in Figure B. Shortly after the reduction, the patient continues to have a foot
drop, but his sensation is slightly improved. Which of the following is the most appropriate next
step in management?

A. Exploration of his sciatic nerve


B. EMG
C. CT scan of his right hip
D. Touch-down weight bearing of his right leg and observation of his sciatic nerve palsy
E. Skeletal traction on the distal femur to relax tension on the sciatic nerve

18. A 32-year-old male sustains a posterior wall acetabulum fracture as the result of a
high-speed motor vehicle collision. Improved patient-reported outcomes after surgical treatment
are associated with which of the following variables?
A. Increased age
B. Increased hip flexion-extension arc
C. Immediate weight-bearing
D. Increased hip muscle strength
E. Decreased stride length

19. A 33-year-old male sustains the injury seen in Figure A as a result of a high-speed motor
vehicle collision. Based on this image, what is the most likely acetabular fracture pattern?

61
A. Both column
B. Anterior column
C. Anterior column posterior hemitransverse
D. Transverse
E. T-type

20. An acetabular fracture with all segments of the articular surface detached from the intact
posterior ilium is defined as what fracture pattern?
A. Transverse
B. Both column
C. Anterior column posterior hemitransverse
D. Posterior column with posterior wall
E. Anterior column with anterior wall

21. A 25-year-old male is involved in a motor vehicle accident and presents with the left
acetabular fracture. Early fixation of this fracture pattern is associated with all of the following
EXCEPT?
A. Decreased length of hospital stay
B. Improved functional outcome
C. Greater organ dysfunction
D. Higher likelihood of being discharged to home as opposed to a rehab facility
E. Improved fracture reduction

22. Which statement is true with respect to acetabular fracture surgery as the time between
injury and surgery increases?
A. decreased chance of anatomic fracture reduction
B. decreased risk of heterotopic ossification
C. decreased rate of neurologic injury
D. decreased rate of infection
E. decreased rate of multi-organ failure

62
23. An 74-year-old community-ambulating male presents with complaints of right hip pain for 4
months. He does not recall any specific trauma though his pain is quite severe at this point. A
radiograph showed a sustained right acetabular fracture. What is the most appropriate definitive
treatment for this patient?
A. Skeletal traction
B. Conservative treatment with delayed physical therapy and shoe lifts
C. Open reduction and internal fixation
D. Right hip reconstruction
E. Closed reduction and percutaneous fixation

24. All of the following have been shown to negatively affect clinical outcomes in treating
displaced acetabular fractures, EXCEPT:
A. Increased age
B. Intraoperative complications
C. Ipsilateral femoral head injury
D. Involvement of both columns
E. Non-anatomic fracture reduction

25. When viewing pelvic injury radiographs, which of the following describes the findings
diagnostic of an isolated transverse acetabular fracture?
A. Fracture line crossing the acetabulum with disruption of the iliopectineal and ilioischial
lines
B. Disruption of the iliopectineal and ilioischial lines, with extension into the iliac wing and
obturator ring
C. Disruption of the iliopectineal and ilioischial lines, with extension into the obturator ring
D. Isolated disruption of the iliopectineal line, with an intact ilioischial line
E. Isolated disruption of the ilioischial line, with an intact iliopectineal line

26. A 65-year-old female sustained a right Garden IV neck of femur fracture after a slip and fall
getting out of the shower. She is an avid golfer and walks the course on most days. Her past
medical history includes borderline hypertension and migraine headaches. Which treatment
option has shown to have the lowest re-operation rate and best clinical outcomes scores in this
patient population?
A. Closed reduction with cannulated screw fixation
B. Open reduction with cannulated screw fixation
C. Closed reduction and short intramedullary nail fixation
D. Hemiarthroplasty
E. Total hip arthroplasty

27. A 65-year-old patient sustains the displaced NOF fracture. His medical history is significant
for hypertension, Type 2 diabetes and dialysis dependent chronic kidney failure. A postoperative

63
radiograph showed an appropriately placed hip hemiarthroplasty. Based on his risk factors,
what is his most likely post operative mortality at two years after surgery?
A. 13%
B. 25%
C. 45%
D. 60%
E. 100%

Hip fractures often occur older patients from low energy type mechanisms of injury. In femoral
neck fractures, the relative pre-injury mobility is the most significant determining factor for the
postoperative survival.

28. A 48-year-old active female runner underwent percutaneous screw fixation of a minimally
displaced femoral neck fracture six months ago. There were no immediate post-operative
complications, and she was progressed to full weight bearing three months after surgical
fixation. Initial post-operative radiographs, and radiographs taken 3 months post-op revealed
anatomic reduction of the fracture with no shortening. At her latest clinic visit she reports severe
right groin pain, and difficulty ambulating. A current radiograph is shown in Figure A. What is the
most appropriate surgical treatment?

A. Hemiarthroplasty
B. In situ dynamic hip screw revision fixation
C. Core decompression and bone grafting
D. Valgus intertrochanteric osteotomy with blade fixation
E. Open reduction, bone grafting, and revision percutaneous screw fixation

29. A 25-year-old man sustains a displaced right NOF fracture and is seen in pre-op holding
prior to surgery. What position of his lower extremity would result in the lowest intracapsular hip
pressure?
A. Neutral rotation and flexion

64
B. External rotation and flexion
C. Internal rotation and extension
D. External rotation and extension
E. Internal rotation and flexion

30. A 79-year-old cyclist is involved in an accident and sustains a left displaced femoral neck
fracture as seen in the AP radiograph. What is the optimal treatment?

A. Open reduction internal fixation


B. Bipolar hemiarthroplasty
C. Unipolar hemiarthroplasty
D. Total hip arthoplasty
E. Nonoperative treatment

31. A 27-year-old man sustains a displaced femoral neck fracture and undergoes urgent open
reduction internal fixation. What is the most prevalent complication after this injury?
A. flexion contracture
B. Hip instability
C. Nonunion
D. Abductor lurch
E. Osteonecrosis

32. A 70-year-old woman trips on the grass while playing golf and sustains a displaced
comminuted femoral neck fracture. What is the optimal treatment for this patient?
A. Open reduction internal fixation
B. Bipolar hemiarthroplasty
C. Total hip arthroplasty

65
D. Unipolar hemiarthroplasty
E. Traction and non operative treatment

33. A 65-year-old male falls from a standing height and sustains a displaced right NOF fracture
and undergoes right hemiarthroplasty. Compared to a total hip arthroplasty, this treatment is
associated with which of the following:
A. Increased blood loss
B. Lower incidence of revision surgery
C. Increased risk of peri-prosthetic fracture
D. Lower dislocation risk
E. Increased risk deep venous thrombosis

34. An 82-year-old female sustains a valgus-impacted subcapital femoral neck fracture and
undergoes cannulated screw fixation as shown in Figure A. She returns for her first follow-up
visit one week later following another fall and now complains of severe hip pain. She is unable
to bear weight on the limb, and a new radiograph reveals varus displacement of her fracture.
She subsequently undergoes revision fixation but during this procedure, the femoral neck
fracture displaces and becomes comminuted. Which is the most appropriate next step in
management?

A. Skeletal traction
B. Revision fixation of the femoral neck fracture
C. Hardware removal and placement of a sliding hip screw device
D. Hardware removal and hip arthroplasty
E. Resection hip arthroplasty

35. Which of the following statements is true regarding treatment of intertrochanteric hip
fractures with an intramedullary nail versus a sliding hip screw?
A. The use of intramedullary nail has increased in the last ten years
B. The use of sliding hip screws has increased in the last ten years

66
C. Medicare reimbursement is more for a sliding hip screw
D. Intramedullary nails have demonstrated superior outcomes in randomized-controlled
studies
E. Sliding hip screw is superior for treatment of reverse obliquity intertrochanteric fractures

36. Which of the following is not an appropriate implant for treatment of the fracture seen in
Figure A?

A. Cephalomedullary nail
B. External fixation
C. Proximal femoral locking plate
D. 95 degree blade plate
E. Sliding hip screw

37. Anterior perforation of the distal femur from antegrade femoral nailing has been attributed to
what factor?
A. Non-anatomic reduction
B. Mismatch of the radius of curvature of implant and bone
C. Usage of too large an implant
D. Lateral patient positioning
E. Lateral proximal starting point

67
38. Which of the following is a recognized predictor of mortality after hip fracture?
A. American Society of Anesthesiologist (ASA) classification
B. Post-operative weight bearing status
C. Fracture comminution
D. Fixation device used
E. Type of anesthetic used

39. A 55-year-old male is involved in a motor vehicle accident and sustains the injury seen in
Figure A. What is the most appropriate treatment for this type of injury?

A. Total hip arthroplasty


B. Bipolar hemi-arthroplasty
C. Sliding hip screw
D. Percutaneous screw fixation
E. Cephalomedullary nail fixation

40. A 25-year-old male presents following a motor vehicle collision with a Glasgow Coma Scale
of 7. Subsequent imaging in the trauma bay demonstrates a bifrontal cerebral contusion, an L4
burst fracture, multiple rib fractures, an LC-1 type pelvic ring injury, a femoral shaft fracture, and
an open ipsilateral tibial shaft fracture. He is intubated and an intracranial pressure monitor is

68
placed which consistently measures 30mm Hg. He is normotensive with a lactate of 1.5 after 2
liters of crystalloid and 1 unit of packed red blood cells. Which of his injuries would most dictate
a temporizing approach with external fixation of his femoral shaft fracture instead of reamed
intramedullary nailing?
A. L4 burst fracture
B. Bifrontal cerebral contusion
C. Open ipsilateral tibia fracture
D. LC1 pelvic ring injury
E. Rib fractures

41. Which of the following factors is most associated with malrotation during antegrade or
retrograde femoral nailing?
A. Surgeon experience
B. Level of primary fracture line
C. Use of a piriformis starting portal
D. Fracture comminution
E. Closed reduction technique

42. A 29-year-old male sustained a mid-shaft femur fracture in a motorcycle accident. Which of
the following is associated with approximately 5% of patients sustaining this injury?
A. Heterotopic ossification
B. Ipsilateral femoral neck fracture
C. Ipsilateral posterolateral corner injury
D. Pudendal nerve injury
E. Ipsilateral superficial femoral artery injury

43. A 26-year-old male sustains a femoral shaft fracture treated with intramedullary nail.
Postoperatively, what muscular deficits can be expected at medium and long-term follow-up?
A. Weakness with hip abduction and knee flexion
B. Weakness with hip abduction and knee extension
C. Weakness with knee flexion and knee extension
D. Weakness with hip external rotation and hip abduction
E. Weakness with hip external rotation and hip flexion

44. A 38-year-old male was struck by a truck and sustained a left shaft of femur fracture.
Treating this injury with an intramedullary nail with a larger radius of curvature can lead to what
complication?
A. Posterior perforation of the distal femur
B. Varus malreduction
C. Comminution of the fracture site
D. Iatrogenic femoral neck fracture
E. Anterior perforation of the distal femur

69
45. A 26-year-old male presents after a motor vehicle accident. Work-up reveals a closed left
femoral shaft fracture, and an ipsilateral posterior wall fracture. He undergoes intramedullary
nailing of the femur, and open reduction internal fixation of the posterior wall. He is treated with
25 mg of indomethacin three times daily for 6 weeks following an initial dose on the evening of
surgery for heterotopic ossification prophylaxis. Which of the following is true regarding this
post-operative treatment protocol?
A. It is associated with an increased rate of femoral shaft nonunion
B. It has no effect on the healing time of the posterior wall fracture
C. It is associated with a faster time to union
D. Indomethacin is superior to radiation treatment in the prevention of heterotopic
ossification
E. There is a decreased rate of revision surgery needed when indomethacin is administered
post-operatively

46. A 30-year-old woman falls onto an outstretched arm while rollerblading. She presents to the
emergency room with the elbow deformity shown in Figure A. On physical examination she is
unable to range her elbow. She is distally neurovascularly intact. Her radiograph is shown in
Figure B. What is the next step in management of this patient?

A. Closed reduction, hinged external fixator


B. Closed reduction, acute surgical repair of the lateral collateral ligament complex
C. Open reduction and surgical repair of the lateral collateral ligament complex
D. Closed reduction, splinting & early passive ROM
E. Closed reduction, splinting & early active ROM

47. Which of the following is most commonly associated with both simple and complex elbow
dislocations?

70
A. radial head fracture
B. radial neck fracture
C. loss of terminal extension
D. repeat dislocation
E. coronoid base fracture

48. A 35-year-old patient sustains an upper extremity injury after a motor vehicle collision. AP
and lateral radiograph of right elbow showed a comminuted trans-olecranon fracture dislocation.
Which treatment modality will optimize internal stability of the elbow?
A. Open reduction and internal fixation with k-wires
B. Open reduction and internal fixation with tension band wiring
C. Open reduction and internal fixation with plate fixation
D. Open reduction and internal fixation with an intramedullary screw
E. Cast immobilization

49. Bridge plating of the olecranon is MOST appropriate in which of the following clinical
scenarios?
A. Fixation of an olecranon osteotomy used for distal humerus surgery in a 24-year-old male
B. Simple transverse olecranon fracture in 33-year-old female
C. Comminuted olecranon fracture in 45-year-old male
D. Severely comminuted proximal olecranon fracture in an osteoporotic 91-year-old female
E. Aphophyseal elbow fracture in 6-year-old male

50. A 79-year-old woman with osteoporosis presents with a displaced, severely comminuted
olecranon fracture involving the proximal 40%. Which of the following represents the most
appropriate surgical treatment?
A. Intramedullary screw
B. Kirschner wire tension band
C. Total elbow arthroplasty
D. Fragment excision and triceps advancement
E. Dorsal bridge plating

51. A 42-year-old female sustains a distal third radial fracture. What other anatomic structure is
most commonly injured with this fracture?
A. Volar long radiolunate ligament
B. Radioscaphocapitate ligament
C. Dorsal radioulnar ligaments
D. Ligament of Testut and Kuentz
E. Scapholunate ligament

52. A 33-year-old man sustains blunt trauma to his forearm and presents with the injury of right
distal third radial fracture. Definitive management of this injury involves the following:

71
A. Perform closed reduction of the radius, then immobilize the forearm in a long arm cast in
supination.
B. Perform open reduction and internal fixation of the radius, then assess the proximal
radioulnar joint for instability, and percutaneously fix the proximal radioulnar joint if
instability persists.
C. Perform open reduction and internal fixation of the radius, then assess the distal
radioulnar joint for instability, and reconstruct the distal radioulnar joint with a looped
palmaris longus autograft if instability persists.
D. Perform closed reduction of the radius, then assess the distal radioulnar joint for
instability, and perform internal fixation of the radius if instability persists.
E. Perform open reduction and internal fixation of the radius, then assess the distal
radioulnar joint for instability, and percutaneously fix the distal radioulnar joint if instability
persists.

53. A 65-year-old female sustains a fall onto her outstretched right hand. The injury is closed
and she is neurovascularly intact. There is no median nerve paresthesias. Radiographs showed
an undisplaced transverse fracture over the distal end of right radius.What is the next best step
in management of this patient?
A. Admit for acute carpal tunnel syndrome monitoring
B. Admit for acute open reduction/internal fixation
C. Place into removable soft splint and follow-up in clinic
D. Place into rigid splint and follow-up in clinic
E. Place into rigid splint and schedule for outpatient open reduction/internal fixation

54. A 67-year-old woman slips on the ice while retrieving her mail and lands on her outstretched
left hand. She complains of wrist pain and deformity. On physical exam she has no sensation of
the volar thumb, index, and middle fingers. Radiograph showed a displaced distal radius
fracture along with a scaphoid fracture. Two hours following closed reduction, the deformity is
corrected, but the numbness and wrist pain is worsening. Which of the following interventions
should be taken?
A. Evaluation of volar compartment pressures with a needle monitor
B. Icing and elevation of the arm with follow-up evaluation in 8 hours
C. Immediate EMG evaluation of the left upper extremity
D. Closed reduction, carpal tunnel release, and sugar tong splinting
E. Emergent open reduction internal fixation with carpal tunnel release

55. A 64-year-old female sustains a nondisplaced distal radius fracture and undergoes closed
treatment using a cast. Three months after the fracture she reports an acute loss of her ability to
extend her thumb. What is the most likely etiology of her new loss of function?
A. Posterior interosseous nerve entrapment
B. Extensor pollicis longus rupture
C. Extensor pollicis longus entrapment

72
D. Distal radius malunion
E. Intersection syndrome

56. A 51-year-old female presents with an acute inability to extend her thumb, four months after
she was treated with cast immobilization for a minimally-displaced distal radius fracture. What is
the most appropriate treatment at this time?
A. Occupational therapy for strengthening
B. Extensor carpi radialis longus transfer to extensor pollicus longus
C. Extensor pollicis brevis transfer to extensor pollicus longus
D. Extensor indicis proprius transfer to extensor pollicus longus
E. Primary repair of extensor pollicus longus

57. A 55-year-old female presents to the emergency room after falling off her balcony. She
sustained the isolated, closed injury shown in Figures A and B. She is otherwise healthy, but
routinely smokes 30 cigarettes per day. What would be the most appropriate sequence of
treatment steps for definitive management of this injury?

A. Closed reduction and splinting followed by delayed casting


B. Immediate open reduction internal fixation
C. Closed reduction and splinting, CT scan, and immediate open reduction internal fixation
D. Closed reduction and splinting, CT scan, external fixation, delayed open reduction
internal fixation
E. Closed reduction and splinting, external fixation, CT scan, delayed open reduction
internal fixation

58. A 35-year-old patient sustains a left calcaneus fracture. Which of the following fractures has
the highest risk of post-traumatic arthritis?
A. Male patient, Sanders Type III fracture, treated with ORIF
B. Male patient, Sanders Type II fracture, treated with ORIF and bone graft
C. Female patient, workers compensation, Sanders Type I fracture, treated non-operatively

73
D. Female patient, Sanders Type II fracture, treated non-operatively
E. Female patient, workers compensation, Sanders Type II fracture, treated with ORIF

59. A 19-year-old military recruit complains of 7 weeks of right heel pain. He notes the pain is
worse with jumping and long distance running. He has completed a course of plantar fascia and
Achilles tendon stretching with no significant improvement in his symptoms. He denies
constitutional symptoms. On examination, his body mass index is 22, he has a normal foot
posture and can perform a single leg heel rise without difficulty. There is no pain with palpation
of the lateral border of the foot or with external rotation stress to the midfoot. There is
tenderness with medial and lateral compression of the hindfoot and there is a negative
syndesmosis squeeze test. There is a negative Tinel's sign at the tibial nerve. Axial and lateral
radiographs are shown in Figures A and B. What is the most appropriate next step in
management?

A. Heel pad cortisone injection


B. Platelet rich plasma injection and 6 weeks of physical therapy
C. Restricted weight bearing and magnetic resonance imaging of the foot
D. Release of the first branch of the lateral plantar nerve
E. ASTYM or Graston physical therapy techniques to the achilles and plantar fascia

60. A patient sustains a comminuted calcaneus fracture. Three months after the injury the
patient complains of shoewear problems secondary to clawing of the lesser toes. What is the
most likely explanation for this deformity?
A. Sural nerve injury
B. Tethering of the flexor hallucis longus by fracture fragments
C. Medial plantar nerve neuropathy
D. Weakness of the tibialis posterior
E. Unrecognized foot compartment syndrome

61. A 34-year-old man is involved in a motor vehicle accident and sustains an open tibia fracture
and is treated with intramedullary nailing. For the next 4 years, he continues to have pain and
persistent discharge from a sinus over his shin. He ambulates with crutches and refrains from

74
putting weight on the extremity. The clinical appearance is shown below. Radiograph shows non
union at the previous fracture site. Wound culture reveals methicillin-resistant Staphylococcus
aureus (MRSA). What is the next step in treatment?

A. Retention of tibial nail, lifelong intravenous antibiotic suppression


B. Debridement and lavage, exchange nailing using a larger diameter nail, intravenous
antibiotics for 6 weeks.
C. Debridement and lavage, excision of sinus tract, implant removal, intravenous antibiotics
for 6 weeks.
D. Debridement and lavage, addition of ring fixator, intravenous antibiotics for 6 weeks.
E. Debridement and lavage, excision of sinus tract, exchange nailing using antibiotic
impregnated-cement nail, intravenous antibiotics for 6 weeks.

62. A 33-year-old motorcyclist is involved in a motor vehicle accident and sustains a Grade III
open fracture of his tibia that is treated surgically. Over the next 35 years, he undergoes multiple
debridements for a persistently draining wound. Over the last year, he has noticed "tissue
growing out of the wound" and a malodorous smell. A photograph of the wound and a recent
radiograph are seen in Figure A. What is the most likely pathologic process?

A. Infection
B. Squamous cell carcinoma
C. Basal cell carcinoma
D. Melanoma
E. Soft-tissue sarcoma

75
63. ​ A 34-year-old man sustained a gunshot wound to the knee 18 months ago and was treated
with bullet removal and a 10 day course of oral antibiotics. He now complains of 12 months
duration of pain in the thigh and recent ulceration and drainage of the skin near the site of his
gunshot wound. Physical exam is notable for a draining sinus tract, erythema and tenderness of
the mid-thigh. He is afebrile. MRI shows chronic changes of distal femur with intraarticular
extension. Which of the following is the most appropriate management?
A. Two week course of oral cephalosporin
B. Core needle bone culture followed by intravenous antibiotics
C. Surgical debridement, culture, and intravenous antibiotics
D. Core needle biopsy, chest CT scan, and bone scan
E. Neoadjuvant chemotherapy and wide resection followed by adjuvant chemotherapy

64. A 20-year-old man presents with erythema, swelling, and pain at the left sternoclavicular
joint . His temperature is 38.9 degrees Celsius, serum WBC is 14,000, and his C-reactive
protein is elevated. He reports that he uses IV heroin. Joint aspiration shows many grams stain
positive organisms. Which of the following organisms is the most likely pathogen?
A. Propionibacterium acnes
B. Staphylococcus aureus
C. Group B streptococcus
D. Neisseria gonorrhea
E. Enterococcus coli

65. A 16-year-old male presents to the emergency department one day after scratching his leg
on a piece of scrap metal. He reports a progressive rash on his leg that has advanced over the
last several hours. In the emergency room his temperature is 102.8 degrees and his systolic
blood pressure is 98 mmHg. On physical exam the clinical finding shown in Figure A is found.
What would be the most appropriate next step in treatment.

A. MRI
B. Biopsy with urgent frozen section in the operating room
C. Needle aspiration
D. CT
E. Ultrasound

76
66. A 35-year-old male sustains a closed tibial shaft fracture after falling from 12 feet. Which of
the following measurements would be concerning for an evolving compartment syndrome?
A. Intraoperative anterior compartment measurement of 29, with preoperative diastolic
pressure 58
B. Preoperative anterior compartment measurement of 25, with preoperative diastolic
pressure of 60
C. Intraoperative anterior compartment measurement of 25, with intraoperative diastolic
pressure of 54
D. Intraoperative anterior compartment measurement of 28, with intraoperative diastolic
pressure of 72
E. Preoperative anterior compartment measurement of 22, with mean arterial pressure of 70

67. A 28-year-old male sustains a midshaft fibula fracture after being kicked during a karate
tournament and develops compartment syndrome isolated to the lateral compartment of his leg.
If left untreated, which of the following sensory or motor deficits would be expected?
A. Decreased sensation on the dorsum of his foot involving the hallux, 3rd, and 4th toes
B. Inability to plantar flex the ankle
C. Decreased sensation on the dorsum of his foot involving the first webspace
D. Inability to dorsiflex the ankle
E. Inability to abduct his toes

68. A 35-year-old female presents to the emergency room after a motor vehicle collision where
her leg was pinned under the car for over 30 minutes. She sustains a displaced tibial fracture..
Which of the following is the most accurate way to diagnose compartment syndrome?
A. surgeon's palpation of the leg compartments
B. paresthesias in her foot
C. diastolic blood pressure minus intra-compartmental pressure is less than 30 mmHg
D. diastolic blood pressure minus intra-compartmental pressure is greater than 30 mmHg
E. intra-compartmental pressure measurement of 25 mmHg

69. A 32-year-old male sustains the injury of mildly comminuted tibial and fibular fractures. His
blood pressure preoperatively was 132/84. After closed reduction and placement of an
intramedullary nail, his intraoperative leg compartment pressures are measured, with the
highest being 28 mmHg. His blood pressure at this time is 84/57. What is the next appropriate
step?
A. Immediate four compartment fasciotomy
B. Fasciotomy of the highest compartment(s)
C. Removal of the nail and placement of an external fixator
D. Repeat evaluation in recovery room
E. Addition of pressors to anesthesia

77
70. Which clinical sign is the most sensitive for the diagnosis of compartment syndrome in a
child with a supracondylar humerus fracture?
A. pulselessness
B. pallor
C. paresthesia
D. paralysis
E. increasing analgesia requirement

71. An 11-year-old child has a tibia-fibula fracture following a fall from a swing. The fracture is
reduced and placed in a long leg splint in the emergency room. What is considered the earliest
sign or symptom of a developing compartment syndrome of the leg?
A. pain out of proportion to injury
B. pale appearance of the foot
C. loss of the ability to move the toes
D. decreased sensation in the foot
E. decreased pulses in the foot

72. A 6-year-old boy with progressive bilateral genu varum undergoes the surgeries shown in
Figure A. On postoperative rounds later that day, the patient appears sedated after several
doses of pain medication. His toes are pink with brisk capillary refill however, passive motion of
his toes causes pain. Among the answer choices listed, what is the best management strategy
for this child?

A. Elevate his legs and reevaluate on morning rounds


B. Adjust his pain medication to accommodate for his increasing pain
C. Administer a muscle relaxant for leg spasms
D. Cast removal and measurement of compartment pressures with a standard device
E. Examine the cast for areas of constriction and reevaluate in the morning

78
73. A 43-year-old male sustains the injury of oblique tibial shaft fracture and fibular shaft
fracture. He has an overlying 3 centimeter wound with exposed bone. Which of the following
antibiotics is indicated for initial prophylaxis?
A. Ciprofloxacin
B. Vancomycin
C. Penicillin
D. Gentamycin
E. Cefazolin

74. A 27-year-old man sustains a Gustilo and Anderson type II open tibia fracture during a
motorcycle accident. He had his full 3 doses of tetanus vaccination as an infant. He also had a
tetanus booster vaccination 18 months ago when he began a new job. In addition to intravenous
antibiotics, what tetanus prophylaxis should be administered?
A. No prophylaxis required
B. Tetanus vaccine
C. Tetanus immune globulin
D. Tetanus vaccine and tetanus immune globulin
E. Tetanus vaccine and tetanus immune globulin with a booster vaccine required 6 months
from now

75. A 79-year-old female falls onto her right hip at home and sustains a right neck of femur
fracture. She undergoes an uncemented unipolar hemiarthroplasty. During insertion of the stem
into the femoral canal, the patient becomes hypotensive and hypoxic. Which of the following has
most likely occurred?
A. Femoral shaft fracture
B. Inadequate fluid resuscitation during surgery
C. Acute myocardial infarction
D. Pulmonary embolism caused by dislodging of deep venous thrombosis during hip
exposure
E. Intramedullary fat and marrow embolization

76. A 20-year-old male sustains bilateral humeral shaft fractures and bilateral femoral shaft
fractures from a motor vehicle collision. While awaiting surgery, the patient suddenly becomes
tachycardic, hypoxemic and experiences mental status changes; physical exam demonstrates
multiple petechiae in his bilateral axilla. CT angiography is ordered and is negative for
pulmonary embolus. What is the most likely diagnosis?
A. hypovolemic shock
B. stroke
C. fat embolism syndrome
D. sundown syndrome
E. narcotic overdose

79
ANSWER:
1.ANS: B
The radiograph shown in Figure A reveals a left hip dislocation, with some obscuring of detail
secondary to the trauma backboard. CT scans should be obtained following a hip dislocation to
evaluate for fractures or impacted areas of the femoral head or acetabulum, as well as
noncongruent reductions and free intraarticular joint fragments.

2.ANS: A
Traumatic hip dislocation results from the dissipation of a large amount of energy about the hip
joint. Clinically, these forces often are first transmitted through the knee en route to the hip. It is
therefore logical to look for coexistent ipsilateral knee injury in patients with a traumatic hip
dislocation.

3. Ans: A
The clinical presentation is consistent for a mildly displaced parasymphyseal fracture in a
pediatric patient with an open triradiate cartilage. Weight bearing as tolerated is the most
appropriate treatment.

In skeletally immature pelvic ring fractures, the majority of cases can be treated nonoperatively.
Open reduction and internal fixation is required for acetabular fractures with >2 mm of fracture
displacement and for any intra-articular or triradiate cartilage fracture displacement >2 mm.
External fixation is necessary for pelvic ring displacement of >2 cm to prevent limb-length
discrepancies.

4. ANS: D
Patients with multiple injuries including a pelvic ring fracture who present with hemodynamic
instability should have a pelvic binder or circumferential pelvic sheet placed as part of their initial
resuscitation.
A systematic approach to search for sources of bleeding and control ongoing hemorrhage is
necessary for patients who present with hemodynamic changes in the setting of a pelvic ring
fracture. Management of continued hypotension after pelvic binder placement is controversial
and varies among trauma centers.

5. ANS: E
The patient has an Anterior-Posterior Compression type 3 pelvic ring injury (APC3), and this
injury places the patient at risk of life- threatening hemorrhage. The most appropriate next step
in the trauma bay is to place the patient in a pelvic binder in order to minimize pelvic volume and
impart stability to the injured hemipelvis to allow for clot formation.

80
Pelvic fractures are high energy injuries with a high association of concomitant musculoskeletal
trauma and damage to multiple organ systems. It is important that any patient with a
high-energy pelvic ring injury undergo a complete work-up including a CT of the chest abdomen
and pelvis to look for alternative sources of bleeding. Application of a pelvic binder should occur
once a pelvic ring injury is identified as part of the ongoing resuscitation of the patient.

6. ANS: C
Figure A shows an unstable bilateral pelvic ring injury. Percutaneous posterior iliosacral screw
fixation places the L5 nerve root at risk as it courses across the sacral ala. Injury to the L5 nerve
root would typically result in weakness in great toe extension and sensory changes on the
dorsum of the foot. It is important to notice that L5 often partially innervates tibialis anterior
along with L4, so weakness to ankle dorsiflexion may be present as well.

7. ANS:B
In the management of patients with multiple injuries, controversy often arises as to the
appropriate method of initial pelvic stabilization. It is generally agreed upon that applying an
external frame is appropriate in the setting of an unstable patient with intraperitoneal fluid and
labile blood pressure. Ex-fix placement can support hemodynamic stabilization and assist the
general surgeons with their laparotomy procedure. Plate or screw fixation of the pelvis should
be delayed because the laparotomy takes precedence in a patient who is hemodynamically
unstable, and internal fixation in the presence of bowel contamination can result in increased
rates of infection.

8. ANS: A
Unstable anteroposterior compression (APC) pelvic fractures are most appropriately managed
with a pelvic binder or circumferential pelvic sheeting as described by Routt et al in the
emergency room prior to definitive treatment. Rapid, temporary fixation of unstable pelvic
fracture patients with hemodynamic instability can be performed in the trauma bay. Pelvic
binders can remain in place during further diagnostic tests such as pelvic vessel angiography.

9.ANS: B
Active involvment of the orthpaedic surgeon in managing these life threatening injuries remains
critical. A sheet or pelvic binder needs to be emergently applied in this clinical scenario. An
aggressive resuscitation protocol must also be initiated. The review article by Hak et al
discussed the advances in prehospital, interventional, surgical, and critical care that have led to
increase survival rates for pelvic injuries. Gonzalez et al found that initial coagulopathy in
trauma patients was associated with decreased survival. They noted that hypothermia and
acidosis was well managed but pre-ICU coagulopathy was the most difficult to treat. They
recommended early FFP in a FFP:PRBC ratio of 1:1

10. ANS: C

81
Figure A shows an APC III injury, which is a rotationally and vertically unstable injury, with
damage to the anterior ring, pelvic floor, and posterior ligamentous stabilizing structures.

The referenced study by Sagi et al found that biomechanically, a percutaneous iliosacral screw
and anterior ring internal fixation was the most stable construct. In addition, he found no
biomechanical support for addition of a second iliosacral screw.

11. ANS: E
Unstable pelvic fractures can be devastating injuries often resulting in significant morbidity and
even death. According to the referenced study by Smith et al, fracture pattern and
angiography/embolization were not predictive of mortality in patients with unstable pelvic
injuries. The three factors they found to be predictive were: increased blood transfusions in the
first 24 hours, age >60 years, and increased ISS or RTS scores. Deaths were most commonly
from exsanguination (<24 hours) or multiorgan failure (>24 hours).

12. ANS: B
The most common urological injury with pelvic ring injuries remains the posterior urethral tear,
followed by bladder rupture.

13. ANS: C
The clinical presentation and radiograph is consistent with an open-book type
parturition-induced pelvic dislocation.

The case series by Kharrazi et al reports four patients treated with open-book type
parturition-induced pelvic dislocations. The authors advocate nonoperative treatment with
bedrest and a properly positioned pelvic binder in the acute setting for patients with a
symphyseal diastasis less than 4.0 cm. All four patients had significant symptoms and
radiographic widening (anterior splaying) of the sacroiliac joints. The three patients who had
presented acutely were treated with closed reduction and application of a pelvic binder, while
two had closed reduction of their pelvic dislocation while anesthetized with a general anesthetic.
At latest follow-up the diastasis at the pubic symphysis reduced to an average of 1.7 cm (range:
1.5-2.0) The authors advocate nonoperative treatment with bedrest and a properly positioned
pelvic binder in the acute setting for patients with a symphyseal diastasis of 4.0 cm of less and
operative treatment for diastasis greater than 4cm.

14. ANS: A
This patient sustained an open-book pelvic fracture with a pubic symphysis diastasis of less
than 2.5cm. From the Young and Burgess classification, he has anteroposterior compression
(AP) type 1 injury. Treatment of this is protected weight-bearing and symptomatic treatment.
Stress examination can be utilized in order to ensure that the injury is, in fact, a APC-1 injury,
and not a more severe posterior injury that would require operative intervention.

82
15. ANS: D
In an ipsilateral unstable pelvic ring and acetabular fractures, the pelvic ring injury must be
initially stabilized in order to reduce the acetabular fracture to a stable base.
16. ANS: E
CT scanning is indicated in acetabular fractures for determination of surgical approach and
techniques, evaluation of marginal impaction and presence of intra-articular loose bodies
(especially after hip dislocation), and evaluation of fracture piece sizes and relative positions.

17. ANS: C
Following successful reduction of a traumatic hip dislocation, a CT scan must be obtained to
evaluate for any entrapped osteochondral fragments.

Traumatic hip dislocations are typically the result of high-energy trauma and are frequently
associated with posterior wall acetabular fractures. Initial management of a traumatic hip
dislocation, with or without a fracture, is immediate closed reduction under conscious sedation.
If closed reduction is unsuccessful, immediate open reduction in the operating room should be
performed. Following successful reduction, a CT scan is necessary to rule-out intra-articular
osteochondral fragments, even in the absence of fracture on radiographs. The incidence of
sciatic nerve palsy following a traumatic hip dislocation is roughly 10%.

18. Ans: D
Patient functional outcomes after acetabular fractures have been shown to be related to
postoperative hip strength, regardless of surgical approach.

19. ANS: D
The radiograph in Figure A shows a transverse acetabulum fracture. The iliopectineal (anterior
column) and ilioischial lines (posterior column) are interrupted, revealing bicolumnar
involvement; however, this is different than the both column fracture, as a transverse pattern
has articular surface still in continuity with the axial skeleton via the sacroiliac joint.

20. ANS:
A both column acetabular fracture is defined as an acetabular fracture with no articular surface
in continuity with the remaining posterior ilium (and therefore, axial skeleton). The spur sign is a
radiological sign seen with these fractures, and is the posterio-inferior aspect of the intact
posterior ilium.

21. ANS: C
Early fixation of acetabular fractures is associated with lesser organ dysfunction.

22. ANS: A
Madhu et al showed time to surgery was a significant predictor of radiological and functional
outcome for both elementary and associated displaced fractures of the acetabulum. Both

83
anatomic reduction and functional outcome significantly worsened as time to surgery increased.
It was found anatomic reduction was more likely when surgery was within 15 days for
elementary fracture and 5 days for associated.

Incorrect answers:
B: No data exists showing a decrease in heterotopic ossification as time to surgery increases.
C: Neurologic injury is more associated with the initial injury.
D,E: Multi-organ failure was not commented on, but infection showed a trend towards being
more likely with longer time to surgery.

23. ans: D
The patient described in this question has sustained an insufficiency fracture of the right
acetabulum that has been neglected. Total hip arthroplasty (THA), with use of flanged and/or
custom acetabular components as needed, is necessary to reconstruct the acetabulum and
address the significant femoral head damage.

Total hip arthroplasty as the primary treatment for acetabular fracture remains controversial.
Osteopenia, pre-fracture hip arthritis and significant chondral impaction injuries make
osteosynthesis difficult and more prone to failure. In the elderly patient with these injury
characteristics or delayed presentation, THA may be a preferred first procedure as it can
definitively address these issues.

24. ANS: D
Negative outcome factors have been shown to include: increasing patient age, time from injury
to surgery (>3 weeks), intraoperative complications, femoral head bone or cartilage injury, and
fracture reduction > 1-2mm from anatomic. Choice of surgical approach has not been shown to
affect patient outcomes.

25. ANS: A
Transverse acetabular fractures separate the innominate bone into two fragments, the superior
iliac and the inferior ischiopubic, by a single fracture line that crosses the acetabulum
horizontally. The iliopectineal and ilioischial lines are disrupted on the AP pelvis radiograph.
Axial CT scan of this fracture pattern at the level of the dome will show a vertical anterior to
posterior fracture line. Answer choice B is describing a both column injury or anterior column
posterior hemitransverse, and answer choice C describes a T-type fracture pattern. Answer
choices D and E describe an anterior column and posterior column injury respectively.

26. Ans: E
This is an active, healthy elderly patient. Treatment of her hip fracture with total hip arthroplasty
(THA) has shown to have the lowest re-operation rates and best functional outcome scores
when compared to internal fixation devices and hemiarthroplasty.
ncorrect Answers:

84
Answers A,B,C: Arthroplasty of any type has been shown to have the least amount of
complications and greatest functional outcome scores compared to internal fixation devices.
Indications for treatment of femoral neck fractures with internal fixation include: (1) stable or
unstable fractures in young or physiologically young patients (2) stable fragility fractures
(Garden I and II) in low demand elderly patients.
Answer D: Hemiarthroplasty is most appropriate for displaced femoral neck fractures (Garden III
or IV) in low-functional demand elderly patients.

27. ANS: C
Based on the injury sustained and the risk factors (namely chronic renal failure) shown, the
patient will likely have a postoperative mortality of 45% at 2-years post operatively.

28. ANS: D
The clinical presentation is consistent with a femoral neck nonunion, which is supported by the
presence of new varus collapse and shortening which was not noted on prior radiographs. The
most appropriate method to treat this complication is valgus intertrochanteric osteotomy of the
femur with blade plate fixation.

Valgus intertrochanteric osteotomies function by making a vertical fracture more horizontal,


converting shear into compressive forces. It also helps correct the varus position of the fracture
nonunion.

29. ANS: B
The femoral neck fracture is considered by most to be a surgical emergency in a 25-year old
due to the at risk blood supply of the femoral head. Due to the intra-capsular hematoma and
hemarthrosis that occurs, the involved extremity is often found to be in external rotation and
flexion as the intra-capsular volume is the lowest in this position.

30. Ans: D
An AP pelvis radiographs with a displaced femoral neck fracture is seen in Figure A. It important
to note that degenerative changes are seen on this image. Both references suggest that elderly
active individuals should be treated with a primary total hip after displaced femoral neck
fractures.

31. Ans: E
Femoral neck fractures in young patients are difficult to treat, and AVN (27%) is a significant
concern. Despite advances in both imaging and implants, this injury often leads to functional
impairment.
Answers A & D: While each of these complications do occur, they are less common and are
related to the approach and degree of surgical dissection.
Answer B: Hip instability is relatively uncommon.

85
Answer C: Nonunion rate is significant (10%) but lower than the AVN rate. It is has been
associated with the degree of initial displacement and varus malreduction.

32. ANS: C
The answer is total hip arthroplasty (#3).Both references suggest that elderly active individuals
should be treated with a primary total hip after displaced femoral neck fractures.

33. ANS: D
The advantages of hemiarthroplasty, compared with total hip arthroplasty, for the treatment of
displaced femoral neck fractures include the more limited nature of the procedure (decreased
blood loss and operative time) and the lower risk of instability. The disadvantages include the
possible development of pain in the groin and acetabular erosion which increases the risk for
revision surgery. Sim et al reviewed 126 consecutive hybrid total hip arthroplasties done for
acute femoral neck fractures. Minimum follow up was 10.1 years. They noted a high dislocation
rate (10%), yet overall good clinical outcome with 87/102 patients who were alive at latest follow
up reporting either no or only minimal pain.

34. ANS: D
In the scenario of an elderly patient with questionable fixation into the femoral head and a
non-healed femoral neck fracture, proper treatment is arthroplasty.

35. Ans: A
The use of intramedullary (cephalomedullary) devices has increased in the last ten years
despite a lack of evidence to support superiority over extramedullary implants (sliding hip
screws)
Incorrect Answers:
Answer B: The use of the sliding hip screw has decreased despite equivalence with
cephalomedullary nails
Answer C: Until 2010 Medicare reimbursement was more for cephalomedullary nails.
Answer D: Intramedullary nails have not been shown to have superior outcomes in multiple
studies
Answer E: Sliding screws have been shown to have worse outcomes for reverse obliquity
fractures

36. Ans: E
The image shows a reverse obliquity intertrochanteric hip fracture.

According to the referenced article by Haidukewych et al, unstable peritrochanteric hip fractures
have a worse outcome (failed in 9/16 cases) if treated with a sliding hip screw. Two additional
factors that were found to have a strong correlation with postoperative failure (nonunion, loss of
reduction) were poor reduction and poor implant placement. In this study, fixed angle devices

86
were superior. Intramedullary fixation has the added advantage of a shorter lever arm and less
potential for fracture collapse and limb shortening. The IMN also acts as a medial buttress.

37. Ans: B
Anterior perforation of the femur has been attributed to a simple mismatch in the radius of
curvature of implants and the apex anterior bowed femur. Image below is an example of anterior
perforation of femur.

38. Ans: A
American Society of Anesthesiologist (ASA) classification is predictive of post-surgical mortality
in hip fracture patients.

39. ANS: E
The radiographs demonstrate a reverse obliquity intertrochanteric femur fracture. Compared to
the more stable intertrochanteric femur fracture, a reverse oblique intertrochanteric hip fracture
is not optimally treated with a sliding hip screw. Compression along a sliding hip screw is
designed to create compression along the plane of the fracture, however in a reverse obliquity
fracture pattern as seen here, shear force is created causing medial displacement of the femoral
shaft and screw cutout. Haidukewych et al and Sadowski et al support the use of a blade plate
or cephalomedullary nail for reverse obliquity fractures.

40. Ans: B
In the setting of a severe closed head injury such as a bifrontal cerebral contusion with elevated
intracranial pressures, external fixation of a femoral shaft fracture is indicated to limit the risk of
intraoperative hypotension and decreased cerebral perfusion pressure.

Immediate reamed nailing remains the standard treatment for the vast majority of femoral shaft
fractures, however patients with multiple injuries with incomplete resuscitation and patients with
severe intracranial trauma may benefit from a damage control approach with external fixation.

41. Ans: D
Femoral malrotation after intramedullary nailing is unfortunately a possibility with either
antegrade or retrograde nailing techniques. Malrotation and iatrogenic length changes are most
common when comminution is present, as cortical reads are inherently limited.

42. Ans: B
Ipsilateral femoral neck fractures are seen in 1-9% of femoral shaft fractures and the femoral
neck must be properly imaged either preoperatively or intraoperatively in any patient with a
femoral shaft fracture. Dedicated hip films, possibly including an internal rotation AP, should be
obtained before entering the OR.

43. Ans: B

87
Long term deficits are weakness with knee extension (quadriceps) and hip abduction (glutei
muscles).

44. Ans: E
45. Ans: A
Heterotopic ossification (HO) prophylaxis with indomethacin has been shown to increase the
risk of long-bone nonunion.

Indomethacin therapy has been shown to be an effective means of preventing HO formation,


however literature has shown that it increases the risk of long bone and acetabular nonunion.
Indomethacin works primarily by inhibiting IGF-1, which is a different mechanism from other
NSAID's which typically inhibit the COX enzymes. IGF-1 is important for bone healing, and its
inhibition may be a risk factor for delayed bone healing.

46. E
The clinical presentation is consistent with a simple elbow dislocation. Initial management
should include closed reduction, splinting or sling placement for comfort and early active ROM
exercises.
Simple elbow dislocations should be reduced and treatment guided by the relative stability of
the joint during the arc of motion. The elbow should be splinted where it is most stable. After 5-7
days the splint can be discontinued and active ROM started to apply compressive stability to the
joint. A hinged brace with an appropriate extension block can facilitate motion through the stable
arc.
Incorrect Answers
Answer A, B: While use of a hinged external fixator may be appropriate when instability persists
after repair/reconstruction of the LCL complex, it is not the first step in the initial management of
a simple elbow dislocation.
Answer C: Open reduction may be warranted if closed reduction is unsuccessful, however an
initial attempt at conservative measures should be made.
Answer D: Closed reduction needs to be followed by splinting to give the tissues a chance to
recover from the initial injury; use of ACTIVE range of motion exercises early, will help to
provide dynamic compressive stability to the joint.

47. C
Elbow dislocations are classified as either simple (no associated fracture) or complex
(associated fracture). The goal of treatment is a stable joint that tolerates early motion. The
initial range of motion is the stable arc found on postreduction examination. Studies have
demonstrated a better outcome when simple elbow dislocations are treated non-surgically
rather than with surgical repair. Simple elbow dislocations usually have an excellent outcome
(return of functional range of motion with normal strength). A loss of terminal extension is the
most common sequelae.

88
48. C
Based on the radiographs shown, the patient has a comminuted trans-olecranon fracture
dislocation of the elbow. Stability will be optimized with internal fixation in the form of a plating
construct.

Trans-olecranon fracture dislocations often occur in the young secondary to high energy
mechanisms. Direct blows often lead to high levels of comminution; the distal humerus is driven
into the proximal ulna and olecranon, thereby leading to a concomitant dislocation. In this
setting, the optimal treatment is use of a plate and screw fixation construct to provide
mechanical stability.

49. C
Bridge plating is most appropriately used for fixation of comminuted fractures which are not able
to be fixed anatomically. Of the choices listed above, this would be most appropriate in a
comminuted fracture in a 45-year-old male. In contrast, literature shows that severely
comminuted, osteoporotic low-demand elderly are best treated with olecranon fracture excision
and triceps advancement when possible.

50. D
Multiple treatments exist for olecranon fractures. Tension band construct and intramedullary
screw or k-wire placement are typically reserved for non-comminuted olecranon fractures,
whereas plate and screw fixation is used for comminuted fractures.

Hak et al review olecranon fracture treatment and state that fragment excision and triceps
advancement is most appropriate in elderly, osteoporotic patients with severely comminuted
fractures involving the proximal 30-40% of the olecranon.

51. C
This fracture is often associated with dislocation of the distal radioulnar joint (DRUJ). When the
two injuries are present together, it is commonly referred to as a Galeazzi fracture.

DRUJ instability is a result of injury to the volar and dorsal radioulnar ligaments which are the
primary stabilizers of this joint. Imaging often shows widening of the DRUJ on AP view, dorsal or
volar displacement on lateral view and radial shortening. Treatment of a Galeazzi injury is
operative, consisting of open reduction and internal fixation of the radius with a plate and screw
construct, followed by intraoperative assessment of DRUJ alignment.

52. E
Galeazzi fracture-dislocations are fractures of necessity and must be managed surgically. The
first step involves surgical fixation of the radial fracture. Next, the distal radioulnar joint (DRUJ)
needs to be assessed for stability by looking for gross motion of the distal ulna in forearm
supination. If DRUJ instability persists, this needs to be addressed with temporary percutaneous

89
pin fixation with one or two 1.2- or 1.6mm K-wires placed transversely proximal to the sigmoid
notch. This is followed by immobilization in above-elbow plaster casts in forearm supination for
6 weeks postop. Anatomic reduction and rigid fixation of the radius alone does not guarantee
DRUJ stability.
53. D
Based on the presentation and radiographic findings, the next best step is to place the patient
into a rigid splint (ie. sugar tong) and have the patient follow-up in clinic. It is important to closely
follow this patient, to assess the stability of the reduced fracture.

Distal radius fractures can be successfully managed when the articular surface has been
appropriately reduced and when anatomic relationships have been restored. Indications for
closed reduction and rigid immobilization include extra-articular fractures, fractures with less
than 5 mm of radial shortening, and fractures with dorsal angulation of less than 5 degrees.
Based on the AAOS Clinical Practice Guidelines, this fracture does not meet criteria for
operative intervention.

54. E
the patient is presenting with neurologic deficits of acute carpal tunnel syndrome. This is a
surgical emergency requiring release of the carpal tunnel to prevent permanent dysfunction.

55. B
According to the referenced article by Jupiter and Fernandez, the most common scenario of
extensor pollicis longus rupture after a distal radius fracture is when the fracture is non or
minimally displaced. The hypothesis is that the rupture happens at an area of relative
hypovascularity and healing callus can aggravate this area, leading to a degenerative tear.

56. D
rare complication of non-displaced or minimally displaced fractures of the distal radius treated
with a cast is a delayed rupture of the extensor pollicis longus (EPL) tendon. The EPL is the
primary extensor of the interphalangeal joint of the thumb and also assists with
metacarpophalangeal extension. Extensor indicis proprius transfer to the EPL is the most widely
used and reported treatment for this condition.

57. E
Figures A and B show radiographs of a comminuted AO/OTA C3 fracture of the distal tibia. The
most appropriate steps in the management of this patients injury would include: Closed
reduction and splinting, external fixation, CT scan, and delayed open reduction internal fixation
in this sequence.

Infection and wound healing problems are common with comminuted distal tibia fractures due to
an increased incidence of soft-tissue complications associated with the operative management.
Patients with a history of diabetes and smoking are at further risk. To decrease the risk of

90
complications in this patient, a two-staged, delayed open reduction internal fixation technique is
the best option for the treatment of severe pilon fractures.
Incorrect Answers:
Answer A: Universally, nonoperative treatment of OTA/AO C3 fractures has led to poor
outcomes.
Answer B,C: Although Ruedi and Allgower (1969) were the first to report successful results with
primary open reduction and internal fixation of low-energy fracture patterns, few studies have
achieved good results with low soft-tissue complications.
Answer D: CT scan is most appropriate after frame application. This will allow for distraction and
indirect reduction of the fragments to better characterize the fracture pattern for pre-operative
planning.

58. A
The patient most likely to require late subtalar fusion for post-traumatic arthritis is a male patient
with a Sanders Type III fracture treated with ORIF.

The Sanders classification system seems to remain prognostic for injury severity. It has been
shown that greater articular injury is associated with greater risk of arthritis, irrespective of
patient gender and occupation. In current reviews, type III fractures are 6.5 times more likely to
develop PTA and 4 times more likely to require an ST fusion than Sanders type II fractures.
Post-traumatic arthritis of the subtalar joint is a common complication. Successful salvage can
be achieved with a subtalar arthrodesis.

59. C
Restricted weight bearing and magnetic resonance imaging of the foot is indicated in this
military recruit with a positive heel compression test and suspicion for calcaneal stress fracture.

Calcaneal stress fractures occur as a result of repetitive loading and are more common in
females with the female athlete triad (anorexia athletica) of disordered eating, amenorrhea, and
osteoporosis. On exam tenderness is obtained with medial and lateral compression of the
calcaneus.
Incorrect Answers:
Answer A: Heel pad cortisone injections are typically avoided due to risk of fat pad atrophy and
are not indicated in calcaneal stress fractures.
Answer B: Platelet rich plasma is not indicated in calcaneal stress fractures.
Answer D: The patient presents with a negative Tinel's at the tibial nerve and presentation is not
consistent with first branch of the lateral plantar nerve entrapment.
Answer E: ASTYM and Graston physical therapies are not indicated for calcaneal stress
fractures.

91
60. E
Contracture of the intrinsic flexor muscles of the foot can be the result of unrecognized foot
compartment syndrome. Foot compartment syndrome is a known complication of calcaneus
fractures.

61. E
The patient has chronic osteomyelitis and an infected nonunion complicating previous IM nailing
of an open tibia fracture. Successful treatment requires debridement, removal of the existing
tibial nail, placement of an antibiotic-impregnated rod and IV antibiotics. Sinus tract excision and
biopsy is important to exclude malignant transformation (Marjolin's ulcer).

Intramedullary infection is a recognized complication of IM nailing, especially in the setting of an


open fracture.
Incorrect Answers:
Answer A: Tibial nail removal is paramount as the implant is likely seeded and infection will not
be eradicated as long as foreign material is present.
Answer B: While exchange nailing provides mechanical support for the nonunion, immediate
nailing risks reinfection. Sinus tract biopsy is important to exclude malignant transformation.
Answer C: The tibial nail is stabilizing an infected nonunion. Following its removal, some other
form of stabilization must be substituted. Sinus tract biopsy is important to exclude malignant
transformation.
Answer D: The addition of an external fixator does not detract from the fact that the foreign body
(nail) is left behind.

62.
A Marjolin's ulcer is a malignant tumor that develops around chronic osteomyelitis. The
increasing size and foul smell suggest malignant change. These tumors are most commonly
squamous cell carcinoma (90%). A biopsy of the mass will be needed to confirm the diagnosis.

Marjolin's ulcers can arise in the setting of burns, venous and decubitus ulcers, vaccination,
snake bites and hidradenitis suppurativa. They usually involve the lower extremities. While
squamous-cell carcinoma is most common, a small percentage of cases are basal-cell
carcinoma, melanoma and sarcoma.

Figure A shows the appearance of a fungating soft tissue tumor, together with a radiograph of
chronic osteomyelitis.

63. C
The clinical presentation and radiographs are consistent with chronic osteomyelitis. Chronic
osteomyelitis is notable for a sequestrum, which is necrotic bone that has become avascular
and no longer connected to the normal bone via the Haversian canal system. Involucrum refers

92
to the new bone forming around the sequestrum. Often the involucrum will form a sinus tract
allowing the sequestrum to drain into the soft tissues.

Diagram of chronic osteomyelitis below depicts the sequestrum (E), Involucrum (C), and sinus
tract (D).

64. B
This patient has sternoclavicular joint septic arthritis with gram positive organisms. Although
there is an increased incidence of Pseudomonas aeruginosa infection in IV drug users, S.
aureus is still the most common organism.

Treatment is I&D and appropriate antibiotics, although aspiration and abx have shown some
success too. CT and MRI are useful in diagnosis, and open biopsy or aspiration is
recommended for definitive diagnosis.

65. B
The clinical presentation is consistent with early necrotizing fasciitis. A biopsy with a frozen
section is effective at rapidly confirming an early diagnosis. If the biopsy is performed in the
operating room, and is positive, then their will be minimal time delays in performing the required
radical debridement.

Necrotizing fasciitis is characterized by hypotension, ascending rash, bullae and fevers. Skin
abrasions, prior surgical intervention, and any cause of open wounds in the skin are all risk
factors for the condition. The most common cultures are polymicrobial. The management
consists of immediate IV antibiotics and emergent surgical debridement. Initial IV antiobiotics
should be broad-spectrum to include penicillin, an aminoglycoside, clindamycin, and
metronidazole.

66. A
A delta P (diastolic blood pressure minus compartment pressure measurement)of < 30 mmHg is
an indication for fasciotomies with the caveat that the diastolic pressure is measured either pre-
or postoperatively.

93
Currently, it is recommended that intraoperative compartment pressures be compared to
preoperative diastolic blood pressures, with delta p < 30 indicating the need for fasciotomies.
Incorrect Answers:
Answer B: a delta p of 35 is not an indication for fasciotomies in the setting of a benign clinical
exam.
Answer C: a delta p of 29 when calculated with the intraoperative diastolic blood pressure is not
an indication for fasciotomies. This value should be obtained from preoperative diastolic blood
pressure measurements.
Answer D: a delta p of 44 is not an indication for fasciotomies.
Answer E: mean arterial pressure is not used for calculation of delta p.

67. A
The clinical vignette describes a scenario of isolated compartment syndrome in the lateral
compartment of the leg. The only nervous structure residing in the lateral compartment is the
superficial peroneal nerve. In compartment syndrome of the lateral leg compartment, failure of
prompt surgical fasciotomy would present as a sensory deficit of the superficial peroneal nerve
presenting as numbness on the dorsum of his foot involving the hallux, 3rd, and 4th toes, as
seen in Illustration below.

68. C
The most accurate way to make the diagnosis is to measure the difference between the
diastolic blood pressure and intracompartmental pressure (delta p).
An absolute intra-compartmental value greater than 30 to 45mmHg can also be used to make
the diagnosis of compartment syndrome, but is more controversial than the delta p according to
Kakar and Amendola.

94
69. D
A mildly comminuted tibia fracture is a fracture highly associated with compartment syndrome.
However, in this scenario, the delta p (difference between compartmental pressures and
diastolic pressure) is 56mmHg (greater than 30) preoperatively, with a decrease to 29mmHg
(less than 30) intraoperatively, due to the hypotension associated with anesthesia.

70. E
Although pain, pallor, paresthesia, paralysis, and pulselessness are all possible signs and
symptoms of compartment syndrome in children with fractures, studies have shown increasing
analgesia requirement is more sensitive.

71. A
The Willis reference states “the single most important symptom of impending compartment
syndrome is pain out of proportion to the injury." This symptom requires a conscious patient.
Most children requiring a reduction for a displaced upper or lower extremity fracture will become
comfortable soon after the reduction has been completed. Children requiring frequent analgesia
or complaining loudly about pain should be examined very carefully for possible compartment
syndrome.” The key wording in this question is “earliest indicator”. Pulselessness, paralysis,
pallor, and parasthesias are all late indicators.

72. D
Intracompartmental pressure measurements should be performed when pain with passive
motion of the toes is found in young patients with insufficient clinical data to establish a definitive
diagnosis of compartment syndrome. The child in this clinical vignette has Blount’s disease
which was treated with bilateral tibial osteotomies, a procedure commonly associated with
compartment syndrome.

Pain with passive stretch is the most sensitive clinical sign of elevated compartment pressures
prior to the onset of ischemia in compartment syndrome. Pain is difficult to assess in children at
baseline, therefore, a high level of suspicion should exist and compartment pressure monitoring
should be performed in unreliable patients.

73. E
The scenario described above is of a type II open fracture, and current initial recommendations
for these injuries include tetanus prophylaxis/update and a first generation cephalosporin (if no
allergies).
Incorrect Answers:
Answer A: Fluoroquinolones can be used in these injuries as second-line agents if allergies are
present.
Answer B: Vancomycin is not currently recommended as a first-line agent for prophylaxis.
Answer C: Penicillin can be added in contaminated type III injuries.
Answer D: Gentamycin can be added in type III injuries.

95
74. A
Any time an open wound is encountered, the tetanus prophylaxis protocol should be initiated in
the emergency room. The correct treatment depends upon the severity of the wound and the
patient's tetanus vaccine status. Treatment may entail no further action, vaccination, or
vaccination and administration of the tetanus immune globulin. The tetanus vaccine, booster,
and immune globulin are used to enhance the immune response to clostridium tetani, a gram
positive bacillus found in soil. In this case, the patient's tetanus had been updated within the
past 5 years so he does not need an update of the vaccination or immune globulin. Illustration A
is a concise table that can be used as an algorithm to provide appropriate tetanus prophylaxis.

75. E
During insertion of the femoral stem, the intramedullary pressures are increased. Fat and
marrow elements can become embolized into the bloodstream at this point resulting in
ventilation perfusion mismatch in the lungs.

76. C
The clinical presentation is consistent with fat emboli syndrome. Fat embolism syndrome results
when embolic marrow fat macroglobules damage small vessel perfusion leading to endothelial
damage in pulmonary capillary beds leading to respiratory failure and an ARDS-like picture.
Patients following long bone fractures are particularly susceptible to fat emboli syndrome. Fat
emboli syndrome occurs in only 3-4% of patients with long bone fractures, despite nearly 90%
of patients with long bone fractures having some evidence of fat emboli. The major clinical
features of FES include hypoxia, pulmonary edema, central nervous system depression, and
axillary or subconjunctival petechiae.

96
ONCOLOGY

1.An 11-year-old boy presents with knee pain, warmth and swelling after a fall. Radiographs are
as below. Bone scan showed hot area in left distal femur only. Biopsy showed osteoid cell with
atypia. What is the most appropriate treatment plan?

A. Neoadjuvant chemotherapy, surgical excision, adjuvant radiation therapy


B. Surgical irrigation and debridement followed by 6 weeks of culture directed intravenous
antibiotics
C. Surgical excision followed by adjuvant chemotherapy and radiation therapy
D. Neoadjuvant chemotherapy, surgical excision, adjuvant chemotherapy
E. Surgical irrigation and debridridement with antibiotic cement spacer placement followed
by 6 weeks of culture directed intravenous antibiotics

2. Which of the following most accurately lists the most common location of osteosarcoma in
decreasing frequency?
A. axial skeleton > distal femur > proximal tibia > proximal humerus
B. distal femur > proximal tibia > proximal humerus > axial skeleton
C. distal femur > axial skeleton > proximal tibia > proximal humerus
D. distal femur > proximal tibia > axial skeleton > proximal humerus
E. proximal tibia > distal femur > axial skeleton > proximal humerus

3. You are asked to consult on a 8-year-old child in the emergency department with the
pathologic fracture seen in Figure A. In preparation for transfer to a sarcoma center for
treatment, all of the following are correct steps in the child's management EXCEPT?
A. Pain management
B. Hanging arm sling

97
C. Tumor staging, including bone scan, CT of chest, MRI of the entire bone
D. Consultation with pediatric oncology
E. Tumor biopsy performed by interventional radiology

4. In patients with an extremity-based osteosarcoma without metastasis, all of the following are
risk factors for disease progression and poor outcomes EXCEPT?
A. High histologic grade
B. Low serum level of alkaline phosphatase at diagnosis
C. Large tumor volume
D. Inadequate surgical margins following resection
E. 70% histologic tumor necrosis after pre-operative chemotherapy

5. A 13-year-old girl presents with knee pain for 2 months especially at night. She denies fevers
and weight loss. Her physical exam reveals a painful thigh mass. A radiograph showed
Codman’s triangle over the left distal femur. What is the next most appropriate step in
managment?
A. Repeat radiographs in 3 months with observation
B. External beam radiation and chemotherapy
C. Surgical biopsy and culture directed intravenous antibiotics
D. Neoadjuvant chemotherapy followed by wide excision and adjuvant chemotherapy
E. MRI of the entire bone, whole body bone scan and CT chest

6. ​A 13-year-old girl presents with right thigh pain and constitutional symptoms for 8 weeks. The
patient's serum LDH and alkaline phosphatase are elevated. A radiograph of the right femur is
shown in Figure A. Computed tomography (CT) of the chest reveals no pulmonary metastases.
Magnetic resonance imaging (MRI) of the right femur shows erosion of the tumor through the
cortex into the adjacent soft tissue, but no skip lesions are seen. A biopsy specimen showed a
high grade osteosarcoma with atypical nuclei and mitoses present. What stage does this
patient's case most appropriately represent using the Enneking Staging System for
Musculoskeletal Tumors?
A. IA
B. IB
C. IIA
D. IIB
E. IIIA

7. What is the most common site of metastasis for osteosarcoma?


A. Bone
B. Liver
C. Lung
D. Kidney
E. Lymph nodes

98
8. A 15-year-old cross country runner complains of vague shin pain. A radiograph of the tibia
showed a lytic lesion with onion skin appearance. A biopsy is performed and uniform sheets of
small round blue cells are seen with round or oval nuclei of uniform size. Which of the following
chromosomal translocations is most likely associated with this condition?
A. t(10;20)
B. t(11;22)
C. t(X;18)
D. t(9;22)
E. t(2;13)

9. ​An 8-year-old girl comes to the office with increasing thigh pain mostly at night. Radiograph
and biopsy specimen are shown in Figures A and B. The patients WBC and ESR are both
elevated. What is the most likely diagnosis?

A. Ewing's sarcoma
B. Osteosarcoma
C. Osteomyelitis
D. Giant cell tumor
E. Chondrosarcoma

10. After tumor staging, what is the most appropriate treatment for a 17-year-old male with
stage IIb Ewing's sarcoma in his proximal tibia?
A. Neoadjuvant radiotherapy, marginal surgical resection, adjuvant chemotherapy
B. Neoadjuvant chemotherapy, marginal surgical resection, adjuvant radiotherapy
C. Neoadjuvant chemotherapy, wide surgical resection, adjuvant chemotherapy
D. Wide surgical excision and reconstruction
E. Radiotherapy and chemotherapy without surgery

99
11. Which of the following tests is required for a standard work-up of Ewing's sarcoma that is
not routinely obtained for staging of osteosarcoma?
A. MRI
B. CT scan
C. Bone scan
D. Protein electrophoresis
E. Bone marrow biopsy

12. A 65-year-old male patient presents with pain and swelling in his thigh 1 year. Radiograph
biopsy results are shown below. What is the most appropriate treatment?

A. Surgery
B. Chemotherapy
C. Radiotherapy
D. Surgery and chemotherapy
E. Surgery, chemotherapy and radiotherapy

13. ​Patients with which of the following conditions have the highest incidence of malignant
tumor?
A. Single enchondroma
B. Single osteochondroma
C. Multiple hereditary exostosis
D. Ollier's disease
E. Maffucci's syndrome

100
14. ​Which of the following tumors is chemotherapy and radiation therapy resistant?
A. Ewing's sarcoma
B. Classic intramedullary chondrosarcoma
C. Synovial sarcoma
D. Chordoma
E. Dermatofibrosarcoma protuberans

15. A 65-year-old male is seen for increasing thigh pain and a new femoral lesion seen by his
oncologist. A current radiograph is shown in Figure A. He has a known diagnosis of lung
carcinoma but no history of metastatic disease. What is the next most appropriate step in
management of this patient?

A. Biopsy
B. Intramedullary stabilization
C. Intramedullary stabilization and send femoral reamings as biopsy
D. Palliative chemotherapy
E. Palliative radiotherapy

16. All of the following are known steps in the development of a malignant tumor with the ability
to metastasize EXCEPT?
A. Increased apoptosis
B. Sustained angiogenesis
C. Tumor cell intravasation
D. Avoidance of immune surveillance
E. Genomic instability

17. ​Metastatic bony lesions that occur distal to the elbows or knees are most likely to originate
from which one of the following primary organs?

101
A. Breast
B. Lung
C. Thyroid
D. Gastrointestinal
E. Prostate

18. A 65-year-old woman presents with elbow pain. Her radiograph showed a solitary lesion of
the supracondylar region of humerus. The patient had a history of non-metastatic breast cancer
10 years ago which was treated successfully. Repeat mammogram, bone scan and CT scan of
the chest, abdomen and pelvis demonstrate this to be an isolated lesion. What is the next most
appropriate action in treatment?
A. Biopsy of lesion
B. Total elbow arthroplasty
C. Currettage and bone grafting
D. Percutaneous cement injection
E. Radiofrequency ablation

19. ​A 53-year-old woman with a history of Paget's disease and bilateral total hip arthroplasties
presents with left hip pain and dysuria. An AP pelvic radiograph and CT scan showed a lytic
lesion in the pelvis. What is the next most appropriate step in management?
A. Revise the left hip total arthroplasty with a cemented stem
B. Open reduction and internal fixation of the acetabular fracture
C. Rest, IV bisphosphanates and follow-up in 6 weeks
D. Radiation therapy
E. Technetium Tc 99 and CT of the chest, abdomen and pelvis

20. ​What is the most common cause for an aggressive lytic bone lesion in a patient above
40-years-old?
A. Multiple myeloma
B. Post-radiation sarcoma
C. Metastatic bone disease
D. Paget's sarcoma
E. Lymphoma

21. A 65-year-old patient without a history of cancer presents with thigh pain and a lytic lesion in
the left femur. A CT scan of the chest, abdomen, and pelvis as well as a bone scan show this to
be an isolated tumor. Biopsy confirms adenocarcinoma. What is the most likely primary source
of this patient's tumor?
A. Gastrointestinal
B. Lung
C. Kidney
D. Prostate

102
E. Breast

22. What is the most common anatomical site for simple bone cysts in children?
A. Distal humerus
B. Proximal humerus
C. Distal femur
D. Proximal tibia
E. Tibial shaft

23.An 11-year-old boy presents with acute right arm pain after throwing a baseball. He is
otherwise healthy and has no other complaints. Radiograph and CT scan of the lesion are
shown in Figures A and B. What is the most appropriate treatment?

A. Biopsy
B. Immobilization
C. Intramedullary nailing and chemotherapy
D. Wide resection and radiation therapy
E. Amputation

24. ​A 14-year-old boy is brought to the emergency department after falling onto his right
shoulder. Radiograph shows a . Which of the following is most likely complication to occur with
curettage and bone grafting of this lesion?
A. Metastases to the lungs
B. Recurrence of the lesion
C. Limb-length discrepancy of the affected side compared to the contralateral side
D. Continued pain in the affected extremity
E. Paraneoplastic disease

103
25. ​A 7-year old boy experienced severe right hip pain following ice-skating. He previously had
no hip pain and is otherwise healthy. Radiographs is shown. What is the recommended
treatment ?

A. Open biopsy and pathological staging without fracture fixation


B. Open reduction and internal fixation with bone grafting of the bone cyst
C. Bedrest for 4 weeks followed by progressive weight-bearing as tolerated
D. Immediate hip spica casting
E. Tibial traction pin with 4 weeks of skeletal traction followed by open biopsy

26. ​ 15-year-old boy twists his ankle and is brought to the emergency room. Radiograph shows
a lytic, expansile lesion with well defined border over the right proximal humerus . What is the
most likely progression of the this lesion as the child reaches skeletal maturity?
A. Increase in size and involve the entire bone
B. Decrease in size and may disappear altogether
C. Turn into a malignant tumor
D. Remain unchanged
E. Lead to deformity of the affected bone

27. A 12-year-old boy presents with pain in the left hip for 6 weeks duration. Physical exam
shows he is afebrile. He has a WBC of 12.2, and an ESR of 16. A pelvic radiograph showed a
lytic lesion in metaphyseal bone of ilium with bony septae and a bubbly appearance. MRI
showed a fluid-filled lines in the same region. What is the most appropriate next step in
treatment.
A. Curettage and bone grafting
B. Phenol injection
C. Observation
D. Percutaneous high-frequency ablation
E. Wide resection and perioperative chemotherapy

104
28. A 12-year-old female is involved in a low speed motor vehicle accident and presents with a
pathologic fracture in the ulna. A radiograph and needle biopsy performed and the diagnosis of
aneurysmal bone cyst is confirmed. What is the best next step in treatment?
A. Open biopsy for adequate diagnostic tissue
B. Open reduction internal fixation of the distal ulna
C. Immediate curettage and adjuvant chemo/radiotherapy
D. Nonoperative fracture management
E. Preoperative chemotherapy and resection of tumor

29. Intralesional curettage and bone grafting is an accepted component of the treatment plan for
all of the following conditions EXCEPT?
A. Giant cell tumor
B. Aneurysmal bone cyst
C. Chondroblastoma
D. Chondromyxoid fibroma
E. Osteofibrous dysplasia

30. A 33-year-old female reports lateral knee pain and slight fullness. She underwent a bone
scan which showed increased activity isolated to the proximal fibula. Radiographs and histology
are shown in Figures A and B. What is the most likely diagnosis?

A. Parosteal osteosarcoma
B. Eosinophilic granuloma
C. Chondroblastoma
D. Giant cell tumor
E. Multiple myeloma

31. Giant cell tumors of bone can be locally aggressive and result in significant bone
destruction. Which of the following is responsible for this type of bone destruction?
A. Multinuclear giant cells

105
B. Osteocytic stromal cells
C. Tumor cell activation of osteoclasts
D. Tumor cell inactivation of osteoblasts
E. Osteoprotegrin activation of osteoclasts

32. A 40-year-old female presents with dull pain in her knee that has been increasing in severity
over the past 6 months. Radiograph of knee shows a lytic lesion at right proximal tibia that is
extending to epiphysis and border of subchondral bone. Biopsy of the lesion shows nuclei of the
stromal cell that are identical to nuclei of giant cells. What is the most appropriate treatment?
A. Observation
B. Bone marrow biopsy
C. Intra-lesional curettage with local adjuvant therapy
D. Neoadjuvant chemotherapy, surgical excision, and adjuvant chemotherapy
E. Neoadjuvant radiotherapy and surgical excision

33. A 21-year-old man presents with a lytic lesion in his distal femur. A chest radiograph reveals
a lung nodule. Which of the following tumors most likely to metastasize to lung ?
A. Non-ossifying fibroma
B. Giant cell tumor
C. Aneurysmal bone cyst
D. Osteoid Osteoma
E. Hemangioma

34. All of the following locations are common sites for giant cell tumor of bone to occur
EXCEPT?
A. posterior elements of the spine
B. Distal femur
C. Sacrum
D. Distal radius
E. Proximal tibia

35. ​A 15-year-old boy presents with a painful mass over his great toe. It has been slowly
growing over the past year. Clinical photograph and radiograph are shown in Figures A and B.
What is the most appropriate treatment?

106
A. Observation and shoe wear modification
B. Amputation
C. Osteotomy through the base of the tumor and local wound care
D. Systemic chemotherapy
E. Radiotherapy and surgical excision

36. An 18-year-old male presents with the radiographs shown in Figures A and B. Genetic
work-up reveals a defect in the EXT-1 gene. What is his underlying diagnosis, and which
mesenchymal tumor is he most at risk of developing?

A. Multiple hereditary exostosis, chondrosarcoma


B. Multiple enchondromatosis, hemangioma
C. Multiple hereditary exostosis, enchondroma
D. Multiple enchondromatosis, chondroblastoma
E. Multiple hereditary exostosis, osteosarcoma

37. ​Which of the following is NOT associated with osteochondroma?


A. subluxation of the radiocapitellar joint

107
B. limb-length discrepancy
C. valgus deformity of the knee
D. ulnar deviation of the wrist
E. absence of the thumb

38. ​All of the following statements regarding hereditary multiple exostosis (HME) are correct
EXCEPT?
A. It is inherited in an autosomal dominant fashion
B. Mutations in HME affect the prehypertrophic chondrocytes of the growth plate
C. It is caused by mutations in either EXT1, EXT2, or EXT3 genes
D. Radiographically, the exostoses are in direct connection to the medullary cavity
E. Radiographically, the exostoses grow towards the physis

108
Answer:
1.D
The history, images, and histology all suggest a high grade osteosarcoma. The appropriate
treatment includes neoadjuvant chemotherapy, surgical excision, and adjuvant chemotherapy.

Osteosarcomas are the most common malignant bone tumor in children. Osteosarcomas are
associated with mutations in the retinoblastoma gene (RB) and p53. The most common
chemotherapy agents include adriamycin (doxorubicin), cis-platinum, methotrexate, and
ifosfamide. Surgical resection of the primary osteosarcoma is key to the treatment and is
generally performed 3 to 4 weeks after the last dose of preoperative chemotherapy. Long-term
survival has a 5x increase with complete resection of both primary and metastatic sites
compared with resection of the primary tumor alone.

2. B
The distal femur is the most common location of osteosarcoma, followed by the proximal tibia,
the proximal humerus, and then the spine and sacrum in decreasing frequency.

3.E
Tumor biopsy, even if performed by interventional radiology, can contaminate the future
resection of this malignant appearing proximal humeral lesion, making limb salvage not
possible. Pain control, splinting, staging/consultation with pediatric oncology will not negatively
affect this child's oncologic outcome in the way that an inappropriate biopsy can.

4. B
lkaline Phosphatase is a marker for bone turnover and an increased level indicates tumor
activity. Therefore, elevated (not low) serum alkaline phosphatase is a risk factor for progression
in patients with an extremity-based osteosarcoma without metastasis.

5.E
As this is likely a malignancy (osteosarcoma) it should be staged prior to biopsy, as described
by Simon and Finn. CT of the chest, bone scan, and MRI of the femur should be performed
before biopsy to look for skip lesions. CT looks for metastasis to lung tissue, which is the most
common site of disseminated disease caused by malignant primary bone and soft-tissue
tumors.

Observation with follow-up radiographs is not appropriate because this lesion is painful and
radiographically aggressive. While infection should be suspected in children with pain and
radiographic changes concerning for osteomyelitis, IV antibiotics are inappropriate because the
patient’s history does not suggest an infectious etiology and further noninvasive workup is

109
necessary prior to invasive methods. Radiation, chemotherapy, and wide excision should not be
performed prior to the appropriate work-up and staging.

6. D
This patient's history, imaging, and biopsy are consistent for osteosarcoma.

The patient's biopsy specimen displays a high grade osteosarcoma with atypical nuclei and
mitoses present, and therefore is a II as there is no evidence of metastasis (Low grade= I and
High grade=II, presence of metastases=III). The question stem states the lesion is
extracompartmental on MRI as the tumor has eroded through the cortical bone and breeched
the "compartment" of the femur and is therefore a B (Intracompartmental=A,
Extracompartmental=B). Therefore this patient has a IIB osteosarcoma of the distal femur. It is
not a 2B, as the lesion is malignant. The Enneking staging system for benign lesions is detailed
with arabic numerals (1=inactive, 2=active, and 3=aggressive).

7.C
The most common site of metastasis for osteosarcoma is the lung via hematogenous spread.
When a patient is diagnosed with osteosarcoma, a chest CT is recommended as part of the
initial staging process. Bone is the second most common site.

8. B
The clinical presentation, images, and biopsy are consistent with Ewings sarcoma. Ewings
sarcoma is a pediatric round cell bone tumor resulting from a translocation of chromosomes 11
and 22 which leads to the formation of a fusion protein (EWS-FLI 1). Radiographs demonstrate
a lytic lesion with periosteal reaction (“onion skin” type). An MRI is useful to evaluate soft-tissue
involvement. The tumor has low signal intensity on T1-weighted images compared with the
normal high signal intensity of the bone marrow. On T2-weighted images, the tumor is
hyperintense compared with muscle. Histologically, uniform sheets of small round blue cells are
seen with round or oval nuclei of uniform size.

9.A
This patients history, radiograph, and biopsy are consistent with Ewing’s sarcoma. Specifically
the radiograph shows an infiltrative, non-geographic lesion with periosteal reaction and
"onion-skinning." The pathology slide shows a diffuse sheet of small round blue cells lining the
bone trabeculae-diagnostic of Ewing's sarcoma. Current treatment for Ewing's sarcoma include
neoadjuvant chemotherapy, surgical resection, and adjuvant chemotherapy. Only in
"unresectable" tumors, or situations where resection would be associated with poor functional
outcome would Ewing's be treated with chemotherapy and radiotherapy without surgical
excision. While Ewing's sarcoma is very radiation sensitive, the worry with radiation therapy in
young patients is the increased lifetime risk of radiation induced sarcomas and the other
complications of radiotherapy including fibrosis, fracture, and wound problems.

110
10. C
The treatment for a 17-year-old male with Ewing's sarcoma in his proximal tibia is neoadjuvant
chemotherapy, wide surgical resection, and adjuvant chemotherapy.

While non-resectable Ewing's sarcoma may be treated with radiotherapy and chemotherapy
alone (answer E), the proximal tibia is considered resectable and as such wide excision and
reconstruction with chemotherapy is the treatment of choice.

11. E
Bone marrow biopsy is a routine part of the staging workup for Ewing’s sarcoma, and is not
routinely obtained for staging of osteosarcoma. Bone marrow biopsy is done because Ewing's
sarcoma can metastasize via the marrow.

12. A
This patient has a conventional intramedullary chondrosarcoma. The treatment is wide resection
alone. Conventional chondrosarcomas are usually found in older patients (40-75 yrs) with a
slight male preponderance. Common locations include the pelvis, proximal femur, scapula. Axial
and proximal extremity lesions are more aggressive. Treatment involves only surgery as these
tumors are neither chemo- nor radiosensitive. Figure A is a radiograph showing a lesion of the
proximal femur with popcorn calcification. Figure B and C are low and high power biopsy
images showing hypercellular stroma with enlarged chondrocytes with nuclear atypia
characteristic of conventional chondrosarcoma.

13. E
Of all the listed cartilage lesions, patients with Maffucci's syndrome have the highest risk of
malignant tumor - estimated to be 100% over the lifetime of the patient.

Single enchondromas and osteochondromas have a 1% risk of malignant transformation.


Multiple hereditary exostosis patients have approximately a 10% life time risk of malignant
transformation. Ollier's disease is one of multiple enchondromas and carries a 30% risk of
malignant transformation (Illustration A) while Maffucci's syndrome is multiple enchondromas
with hemangiomas (Illustration B). Maffucci's is also associated with a large number of other
malignancies—particularly, pancreatic and hepatic adenocarcinoma, mesenchymal ovarian
tumors, brain tumors such as glioma and astrocytoma, and various kinds of sarcomas.

14. B
Classic intramedullary chondrosarcoma is a slow-growing tumor of neoplastic cartilage that is
chemotherapy and radiation therapy resistant. The accepted treatment is therefore wide surgical
excision. Dedifferentiated chondrosarcoma on the other hand may be treated with adjuvant
chemotherapy.

15. A

111
The radiographs show a cortically based lytic lesion in the mid diaphysis of the left femur, which
is consistent with a metastatic lesion from a lung carcinoma, however without a history of biopsy
proven bone metastasis, this lesion needs to be biopsied prior to definitive treatment. Sending
femoral reamings is not an appropriate biopsy technique as significant contamination of the
abductors, skin, and femoral canal occurs which compromises limb salvage if this lesion is in
fact a sarcoma. The biopsy may be followed by intramedullary stabilization under the same
anesthetic if the lesion can be confirmed as a carcinoma by the surgical pathologist. However if
carcinoma cannot be confirmed, no further treatment is indicated until final pathology is
available for review.

16. A
All of the listed options are involved in the development of a malignant tumor with the ability to
metastasize EXCEPT increased apoptosis. Apoptosis is programmed cell death, and tumor
cells are known for their decreased rate of apoptosis. This allows for tumor cell sustained
growth even after induction of cellular signals known to induce apoptosis in non-neoplastic cells.

17.B
Metastatic bony lesions that occur distal to the elbows or knees are most likely to occur from
primary lung and kidney tumors. The exact molecular mechanism for this metastatic pattern is
not known. It is unknown if acral metastases in isolation confer a negative prognosis, or if it is
merely a reflection of an aggressive tumor; however, when encountered, long term survival is
unlikely.

18. A
The radiograph demonstrates a solitary lesion of the supracondylar lesion of the humerus in a
patient with history of localized breast cancer. Work-up shows this to be an isolated lesion.
Biopsy must be performed to rule out primary bone malignancy before definitive treatment is
performed.

19. E
This question has a lot of bells and whistle distraction. Fundamentally, it is a question about
what you do when you see a lytic lesion in the pelvis of a person with a risk for tumor, in this
case Paget's disease. The images show a skeletal tumor of unknown origin. Therefore, you can
discount every “treatment” answer. First, you need to diagnose her.

20. C
The following malignant bone tumors occur most frequently in patients 40-80-years old (in order
of decreasing frequency): metastatic bone disease, myeloma, lymphoma, Paget's sarcoma, and
post-radiation sarcoma. The skeleton is a common site for metastasis from several visceral
carcinomas. Think BLTPK for sources of metastasis: breast, lung, thyroid, prostate, and kidney.

21. B

112
Occult lytic metastatic adenocarcinoma to bone without an identified primary source after bone
scan and CT of the chest, abdomen, and pelvis is most commonly of lung origin. CT of the
chest/abdomen/pelvis correctly identifies the primary tumor in greater than 90% of cases.

22. B
Simple bone cysts are found most commonly in the proximal humerus in children.

Simple bone cysts are non-neoplastic, serous fluid-filled bone lesions that are thought to result
from temporary failure of medullary bone formation near the physis. They are usually found in
patients under the age of 20. Most are asymptomatic unless fracture occurs.

23. B
he history and radiographs are consistent with a pathological fracture through a unicameral
bone cyst (UBC) in the child's humerus. The radiograph shows a lytic, expansile lesion with
well-defined borders, no soft tissue mass, no significant matrix, and no periosteal reaction. The
CT scan confirms this lesion is benign appearing with sharp sclerotic borders. Given the benign
appearance, location, and history, biopsy is not required as a diagnosis of UBC can be made
with the information provided.

The initial treatment of a pathological fracture through a benign bone cyst includes
immobilization while the fracture heals. If this same lesion were present in a weight bearing
bone, such as the proximal femur, then the treatment choice becomes more problematic and
internal fixation needs to be considered to allow the child to mobilize. While many adjuvants
have been tried in the treatment of UBCs, none have shown to be advantageous.

Once the fracture heals, active UBC`s (cyst that communicates with the physis) should be
treated with a methylprednisolone acetate injection if it does not resolve spontaneously.
Curettage and bone grafting be considered if there is no spontaneous healing or the injections
fail.

24. B
Recurrence of the lesion is the most likely complication to occur with curettage and bone
grafting of a unicameral bone cyst.

25. B
A pathological fracture is one that occurs through abnormal bone because the bone lacks its
normal biomechanical properties. Benign tumors of bone that are associated with pediatric
pathological fractures commonly include unicameral bone cyst, nonossifying fibroma, fibrous
dysplasia, and aneurysmal bone cyst. With bone cysts, there is a high risk of fracture recurrence
because of cyst persistence and the recommended treatment is ORIF with bone grafting.

113
26. B
(UBC), a benign tumor typically seen in the ends of long bones(Illustration A) in the skeletally
immature. UBCs can also occur in the tarsal bones of the foot. The cyst usually progressively
shrinks as the patient approaches skeletal maturity and may heal spontaneously after growth is
completed.

27. A
The clinical presentation and imaging studies are consistent with and aneurysmal bone cyst
(ABC). Curettage and bone grafting is the most appropriate treatment.

Aneurysmal bone cysts are a benign and non-neoplastic reactive bone lesions filled with
multiple blood-filled cavities. They can be locally destructive to normal bone and may extend to
soft tissue. 75% occur in patients who are < 20 years old. In the presence of a pathologic
fracture, treatment is nonoperative fracture management. In the absence of a pathologic
fracture, treatment is curettage and bone grafting.

28. D
This patient presents with a small nondisplaced fracture through an aneurysmal bone cyst.
While the plain radiograph is not diagnostic, it does not suggest an aggressive lesion. The
"lakes of blood" seen on the histology confirm the diagnosis. Given the alignment of the fracture,
nonoperative management is preferred. Once the fracture has healed, if the ABC continues to
grow treatment is currettage and grafting. If the fracture stimulates the bone cyst to resolve then
no further treatment is required.

29. E
There is no role for curettage and bone grafting in patients with osteofibrous dysplasia.
Nonoperative treatment is preferred for osteofibrous dysplasia until a child reaches maturity.
These lesions usually regress and do not cause problems in adults unless the skeletal deformity
requires surgical correction.

Benign tumors that are USUALLY treated with curettage and bone grafting include giant cell
tumor, chondroblastoma, chondromyxoid fibroma, and osteoblastoma. Benign conditions that
are OCCASIONALLY treated with curettage and bone grafting include unicameral bone cyst
(UBC), enchondroma, and nonossifying fibroma (NOF).

30. D
Giant cell tumor presents as a lytic lesion in the metaphysis and epiphysis of long bones, most
frequently affecting the knee (as shown). It is characterized by proliferation of mononuclear
stromal cells and the presence of multi-nucleated giant cells with homogenous distribution
(figure B). Although benign, it is locally aggressive and deserves treatments that include
extended curettage followed by cement or bone grafting. Malignancy is rarely associated with

114
GCT but may be found in a recurrent tumor or years after irradiation of a previously benign
GCT.
31. C
The bone destruction in giant cell tumors is caused by the tumor cell activation of the
osteoclasts and the secondary osteoclast resorption of bone.

Benign and malignant tumor cells don't have the capacity to dissolve bone, only the osteoclast
can accomplish this. While the RANK-RANKL interaction is likely related to the tumor induced
osteolysis, osteoprotegrin (OPG) is an antagonist to this process. The multinuclear giant cells
and the osteocytic stromal cells of giant cell tumor are locally aggressive, but they do not have
the capacity to dissolve bone.

32. C
The history, radiographs, and histology are consistent with giant cell tumor, a benign aggressive
tumor treated with intra-lesional curettage followed by local adjuvant therapy. While many
authors advocate for one specific adjuvant therapy (argon beam, liquid nitrogen, cement with
bisphosphonates) no adjuvant therapy has proven better than the rest. It appears rather that
aggressive curettage with a high speed burr is likely the most important step in treatment.

33. B
Giant cell tumors of bone are benign lesions that can be locally aggressive. These tumors are
typically treated with local resection and/or curettage. Radiation therapy is selectively used for
tumors in difficult to resect locations (i.e. spine).

Only about 1 to 9% of giant cell tumors metastasize to the lungs. This stands in contrast to other
benign skeletal tumors, which rarely metastasize to the lungs, making giant cell tumor, along
with chondroblastoma, a unique entity in this regard. Risk factors for lung mets include local
recurrence, the location of the primary giant cell tumor (distal radius, proximal femur, and
sacrum), Musculoskeletal Tumor Society Stage 2 or 3 and an immunocompromised state.

34. A
the most common locations for giant cell tumor of bone to occur are about the distal
femur/proximal tibia, distal radius, and the sacrum. The spine is an uncommon location for giant
cell tumors, and when they occur, they usually occur in the vertebral body, not the posterior
elements. In the axial skeleton, the sacrum is a common location for giant cell tumor of bone.
Outside of the sacrum, giant cell tumor of bone is characteristically an epiphyseal/metaphyseal
lesion.

35. C
The clinical presentation is consistent with a subungual exostosis. Tumor resection with an
osteotomy through the base of the tumor is the most appropriate treatment.

115
Osteochondromas are the most common benign bone tumor. They are a benign chondrogenic
lesion derived from aberrant cartilage which arises from the perichondral ring. One common
presentation of an osteochondroma is a subungual exostosis, which most commonly occurs on
the hallux. With tumor removal and local wound care, patient's have an excellent result as long
as the nail matrix is not injured.

36. A
Multiple hereditary exostosis (MHE) is an autosomal dominant disorder characterized by the
development of multiple osteochondromas and a defect in the EXT-1, EXT-2, or EXT-3 genes.
EXT-1 patients have higher rates of sarcomatous change when compared to those with a defect
in EXT-2. The reported risk for malignant transformation to chondrosarcoma with this condition
is up to 10%. The figures show the presence of multiple osteochondromas, and associated bony
deformity typically found in patients with this disease. MHE is not associated with an increased
risk of enchondroma formation, and degeneration to osteosarcoma is very rare. Multiple
enchondromatosis consists of multiple enchondromas which usually develop in childhood, and
are associated with an increased risk of brain and visceral malignancy.

37. E
The solitary osteochondroma, a common pediatric bone tumor, is a cartilage-capped exostosis.
Hereditary multiple exostosis is an autosomal dominant disorder manifested by the presence of
multiple osteochondromas. Linkage analysis has implicated mutations in the EXT gene family,
resulting in an error in the regulation of normal chondrocyte proliferation and maturation that
leads to abnormal bone growth. Although exostoses are benign lesions, they are often
associated with characteristic progressive skeletal deformities and may cause clinical
symptoms. The most common deformities include short stature, limb-length discrepancies,
valgus deformities of the knee and ankle, asymmetry of the pectoral and pelvic girdles, bowing
of the radius with ulnar deviation of the wrist, and subluxation of the radiocapitellar joint. For
certain deformities, surgery can prevent progression and provide correction. Patients with
hereditary multiple exostosis have a slight risk of sarcomatous transformation of the
cartilaginous portion of the exostosis.

38. E
Hereditary multiple exostosis (HME) is an autosomal dominant disorder associated with
mutations in either EXT1, EXT2, or EXT3.

The EXT proteins are tumor suppressor genes that function to glycosylate indian hedge-hog, a
key cell-signaling molecules produced by the prehypertrophic chondrocytes. Mutation allows
unregulated growth resulting in the characteristic exostoses. Patients with mutations in EXT1
have higher rates of sarcomatous change when compared to those with a defect in EXT2 or
EXT3.

116
Radiographically, the exostoses are in direct connection to the medullary cavity of the bone from
which they originate and they grow away from the physis (not towards as described in Answer
E). Theses radiographic features are helpful in making the radiographic diagnosis of HME.

FOOT AND ANKLE


1.​A 55-year-old female attorney complains of progressive pain and deformity of the great toe.
The pain is localized to a prominence of the medial metatarsal head. A clinical image is
provided in figure A. Which of the following best describes the hallux deformity?

A. Valgus and supination


B. Valgus and pronation
C. Valgus and hyperextension
D. Varus and supination
E. Varus and pronation

2. ​A shoe orthotic with a Morton's extension is indicated for which of the following conditions?
A. Hallux valgus
B. Hallux rigidus
C. 2nd metatarsophalangeal joint synovitis
D. Jones fracture
E. Hammertoe deformity

3. ​You are seeing a 60-year-old male for pain in his great toe that has increased in severity over
the past year despite the use of an insole with a Morton's extension. His great toe
plantar/dorsiflexion range of motion is limited to a 35 degree arc with pain at the extremes of
motion. A radiograph is shown in Figure A. What treatment do you suggest?

117
A. Observation
B. Medial sesamoidectomy
C. Cheilectomy and joint debridement
D. 1st MTP resection artrhoplasty (Keller procedure)
E. 1st MTP fusion

4. ​An 18-year-old male college student presents with a right ankle injury two weeks after slipping
on a soccer ball. The skin is grossly intact and there is no evidence of neurovascular
compromise. The provocative test demonstrated in Figure A is positive. Which of the following
nonoperative treatment modalities have been shown to minimize recurrence of his injury?

A. Immobilization in a non weight-bearing cast


B. Immobilization in a weight-bearing boot
C. Immobilization in a splint
D. Functional bracing with early proprioceptive training
E. Neuromuscular training alone

118
5. In dancers, peroneal muscle weakness has been shown to be the cause of which of the
following?
A. Ankle sprain
B. Fibular fracture
C. Acute cuboid subluxation
D. Achilles rupture
E. Midfoot sprain

6. ​A 48-year-old male complains of 5 years of heel pain while running. Initially the pain was
relieved with Achilles tendon stretching, orthotics, and open-backed shoe wear. Over the past
year these modalities are no longer helpful and he is beginning to have pain with walking.
Clinical photograph and radiograph are provided in figures A and B. Which of the following
treatment options is the best choice to relieve pain and improve function?

A. Arizona gauntlet brace


B. Steroid injection
C. Achilles tendon debridement
D. Achilles tendon debridement, calcaneal exostectomy, and possible FHL transfer
E. Ankle arthrodesis

7. ​A 36-year-old man presents with fever, pain, and wound drainage 4 months after repair of an
acute Achilles tendon rupture. A clinical image is shown in Figure A. Laboratory studies show an
ESR of 29 (reference range 0-22 mm/hr). It is decided that he will undergo debridement and
irrigation followed by culture specific antibiotic therapy. In the operating room, the Achilles
tendon is found to have re-ruptured with a 5 cm defect. What is the most appropriate surgical
treatment at this time?

119
A. Primary repair of the re-ruptured Achilles tendon
B. Debridement of necrotic and infected tendon tissue, with no attempt at reconstruction
C. V-Y plasty of the re-ruptured Achilles tendon
D. Repair of the re-ruptured Achilles tendon with a turndown procedure
E. Repair of defect with flexor hallucis longus tendon transfer

8. ​A 58-year-old golfer fell stepping into a sand trap and ruptured his Achilles tendon one year
ago. He initially chose non-operative treatment, but became unsatisfied with a tender fullness
behind his ankle and ankle weakness noticeable during his tee shots. At the time of surgery, a
large disorganized fibrous mass is found at the site of rupture. Following extensive debridement
there is a 5 cm gap between viable tissue ends. Which of the following surgical techniques
provides the greatest likelihood of a successful clinical outcome?
A. Gastocnemius turndown repair augmented with transfer of the posterior tibial tendon
B. Gastocnemius turndown repair augmented with transfer of the extensor digitorum longus
C. Gastocnemius turndown repair augmented with transfer of the flexor hallucis longus
D. Reconstruction with hamstring autograft
E. Primary repair with the foot in maximal plantarflexion followed by a gradual stretching
program

9. ​What is the greatest advantage of surgical repair of an acute Achilles tendon rupture with
early range of motion compared to non-operative treatment with immobilization in a short-leg
cast for 6 weeks?
A. Lower rate of infection
B. Higher rate of normal skin sensation
C. Better skin cosmesis
D. Lower rate of dehiscence
E. Lower rate of re-rupture

10. ​Which factor increases the chance of wound complications after Achilles tendon repair?
A. Increased body mass index
B. Immediate surgery
C. Male gender

120
D. Age over 40 years old
E. Tobacco use

11. A 36-year-old male recreational golfer has been complaining of left plantar heel pain for
nearly 6 weeks. His pain is worse with weight-bearing, especially the first steps in the morning
or after long periods of rest. To date, he has been treating his pain with anti-inflammatory
medications and physical therapy. His pain had improved by approximately 40% with these
modalities, but the improvements are starting to plateau. What would you recommend next for
treatment of his condition?
A. Stop physical therapy and prescribe custom orthotics
B. Administer bi-weekly extracorporeal shockwave therapy to the heel
C. Endoscopic plantar fasciotomy
D. Bipolar radiofrequency to the heel
E. Night splints

12. A 34-year-old female has an insidious onset of heel pain when first getting out of bed and at
the end of the day after prolonged standing. She works as a waitress and recently had bariatric
surgery with a current BMI of 35. She has a gastrocnemius contracture noted on Silfverskiold
testing. AP and oblique radiographs are shown in Figure A and lateral radiograph is shown in
Figure B. What is the most likely diagnosis?

A. Navicular stress fracture


B. Freiberg's Infraction
C. Plantar fasciitis
D. First branch of the lateral plantar nerve (Baxter's) entrapment
E. Anterior tarsal tunnel syndrome

13. ​A 44-year-old recreational runner began training for a half marathon 6 weeks ago. Over the
last week he has developed heel pain that is worse in the morning upon awakening and when
he arises from his desk at the end of the workday. Physical exam is notable for tenderness with
direct palpation of the anteromedial heel. Which of the following is the best initial management?
A. Stretching of the achilles tendon and plantar fascia along with a prefabricated shoe insert

121
B. Immobilization in a short leg cast
C. Steroid injection of the plantar fascia
D. Custom made orthotic with arch support
E. Surgical release of the medial third of the plantar fascia origin

14. A 40-year-old female presents to the physician for an initial visit with a 5-month history of
plantar medial heel pain. She notices it immediately on getting out of bed in the morning, but the
pain improves after a few steps. The pain is exacerbated throughout her workday to the point
where she is unable to finish her work shift. Figure A shows a lateral radiograph of the affected
heel. Which of the following is the most appropriate initial management?

A. Walker boot immobilization with full weightbearing for 4 weeks


B. Corticosteroid injection to the plantar fascia
C. Surgical release of 50% of the plantar fascia
D. Heel spur resection
E. Achilles stretching exercises

15. ​A 66-year-old male with a known history of uncontrolled Type 2 diabetes presents for follow
up of a forefoot ulcer that is seen in Figure A. All of the following are strong prognostic indicators
of osteomyelitis EXCEPT:

122
A. Exposed bone at ulcer site
B. Periarticular erosions at 1st MTP joint
C. Increased signal within metatarsal diaphysis on T2 weighted MRI
D. Increased signal within indium labeled WBC scan
E. Positive wound culture

16. ​A 55-year-old female with longstanding type I diabetes presents for evaluation of her left
foot, which is gangrene over the right forefoot. On exam, the soft tissue infection extends to the
metatarsals plantarly, and there is a palpable posterior tibial artery pulse. She is otherwise
medically stable, and without sepsis. Which of the following treatment options will most likely
result in definitive management of her forefoot gangrene and allow the highest level of functional
activity after surgery?
A. Transmetatarsal amputation
B. Below the knee amputation
C. Syme amputation
D. Above the knee amputation
E. Extensive soft-tissue debridement, local wound care, and antibiotic therapy

17. ​A 44-year-old male with long standing insulin dependent diabetes complains of a
non-healing plantar foot ulcer. The ulcer is shown in Figure A. The second metatarsal head can
be probed at the base of the wound, and he lacks plantar sensation. Laboratory work-up for
infection is negative. Which of the following is the best initial treatment?

A. Ray resection and primary wound closure


B. Oral antibiotics and local wound care
C. Local wound care and non-weight bearing in a removable boot
D. Surgical debridement, dressing changes, and IV antibiotics
E. MRI of the foot to evaluate for underlying osteomyelitis

18. A 44-year-old man with diabetes mellitus has a non-healing Wagner grade 1 ulcer shown in
Figure A for the past 8 months. Conservative management with total contact casting has not
resolved the ulcer. Physical examination reveals loss of protective sensation by

123
Semmes-Weinstein testing, no signs of infection, positive Silfverskiold test indicating
gastrocnemius contracture, and palpable pedal pulses. What is the next most appropriate step
in management?

A. Integra artificial dermis placement followed by split thickness skin grafting


B. Continued total contact casting
C. Split thickness skin grafting to ulcer
D. Strayer procedure (gastrocnemius lengthening)
E. Weil metatarsal decompression osteotomy

19. ​A 65-year-old patient with poorly controlled diabetes develops 2nd metatarsal head
osteomyelitis deep to a superficial ulcer. Which of the following treatment modalities would have
the highest chance of success?
A. IV Vancomycin
B. Oral Levofloxacin
C. IV antibiotics based on ulcer swab culture sensitivity
D. IV antibiotics based on percutaneous bone biopsy culture sensitivity
E. Elevation and non-weight bearing

20. ​You are caring for a 72-year-old male with diabetes and peripheral neuropathy with a
non-healing forefoot ulcer. To reduce the plantar pressure on his forefoot, which of the following
shoe modifications would you suggest?
A. Polyethylene foam insole
B. Open toe sandals
C. Rocker sole shoes
D. Custom indepth shoes
E. Hard postoperative shoe

124
21. Which of the following patients with type 2 diabetes mellitus is most likely to develop a foot
ulcer?
A. 54-year-old female unable to feel the presence of a 5.07 Semmes-Weinstein
monofilament on the plantar aspect of the foot
B. 63-year-old male with transcutaneous oxygen pressures (TcpO2) of 30 mm Hg
C. 51-year-old male with ratio of ankle to brachial pressures of < 0.6
D. 71-year-old male with serum albumin of 3.1 g/dL
E. 60-year-old with autonomic dysfunction leads to drying of skin due to lack of normal
glandular function

22. A 37-year-old man with type-1 diabetes mellitus reports a 3-month history of a plantar foot
ulcer shown in Figure A. His pulses are palpable and sensation to a 5.07 Semmes-Weinstein
monofilament is absent on the entire plantar surface of the foot. There is no erythema or
drainage and there is no bone encountered during probing of the ulcer. There is no fever and
the white blood cell count is normal. The C-reactive protein and erythrocyte sedimentation rate
are normal. What is the most appropriate next step in treatment?

A. Total contact casting and empiric IV antibiotics


B. Surgical debridement, dressing changes, and empiric IV antibiotics
C. Charcot restraint orthotic walker (CROW)
D. Surgical debridement, dressing changes, biopsy, and culture specific IV antibiotics
E. Total contact casting

23. A 56-year-old male with uncontrolled diabetes presents for follow up of a recurrent midfoot
ulceration. He has been placed into a total contact cast for extended periods without resolution
of the ulcer. On physical examination the patient is unable to feel a 5.07 gm monofilament on
the plantar aspect of his foot. He has an equinus contracture. A clinical photo of the patient and
lateral radiograph of the foot are provided in Figures A & B. Radiographs are unchanged from
prior evaluation. What is the next best option at this point?

125
A. External fixation
B. Below the knee amputation
C. Continued observation
D. Exostectomy with placement into a protective brace
E. Exostectomy & achilles tendon lengthening with placement into a protective brace

24. A 50-year-old male with long-standing type 1 diabetes presents with redness, swelling and
crepitus in his foot two weeks after a twisting injury. Elevation of the extremity reduces the
hyperemia. A radiograph is shown in Figure A. What is the most likely diagnosis?

A. Osteomyelitis
B. Charcot-Marie-Tooth disease
C. Lisfranc fracture-dislocation
D. Charcot arthropathy
E. Freiberg's Disease

126
25. A 43-year-old male presents with painless swelling and erythema of his ankle which
resolves with elevation. He has begun to have trouble ambulating because he reports his ankle
feels "floppy" since a fall several weeks ago. His x-ray is shown in Figure A. What physical
exam test is most appropriate?

A. Thompson test
B. Cotton test
C. Syndesmosis squeeze test
D. Babinski test
E. Semmes-Weinstein monofilament testing

26. A 29-year-old male presents with left knee instability and progressive gait disturbance. He
is only able to ambulate with the assistance of crutches or a walker. He has no pain with
ambulation and has decreased vibratory sensation in the bilateral lower extremities.
Radiographs of left knee show fragmentation of both articular surfaces, joint subluxation and
surrounding soft tissue oedema. All of the following are possible etiologies for this condition
EXCEPT:
A. Diabetes mellitus
B. Syringomyelia
C. Leprosy
D. Neurosyphilis
E. Reiter's syndrome

127
ANSWER:
1. B
Bunions are a common deformity of the great toe that is most prevalent in the aging female
cohort. The etiology is multi-factorial including genetic predisposition and chronic exposure to
narrow toe box shoes. The static soft tissue constraints attenuate medially and contract laterally,
and the plantar dynamic flexors deviate laterally pulling the great toe into valgus and pronation.

2. B
A Morton's extension is a stiff insert with a rigid extension under the 1st ray. This extension is
used to conservatively treat conditions such as hallux rigidus, as it limits extension of the 1st
MTP joint during the push-off phase of gait.

3. C
This patient has hallux rigidus, or arthritis of the first metatarsal phalangeal joint. He has
moderate limitations in motion, and mild joint space narrowing on radiographs. Therefore, he
would be classified as Type 2, and the most appropriate treatment would be cheilectomy and
joint debridement.

4. D
Figure A shows a clinical photo of the anterior drawer test. A positive test is indicative of a torn
ATFL. This patient has sustained an inversion injury to his anterior talo-fibular ligament (ATFL).
To minimize the risk of recurrent ankle sprains, functional bracing with early proprioceptive
training is the optimal non-operative treatment.

Ankle sprains often occur from inversion injuries that affect the ATFL. Injuries may initially be
treated with rest, ice, elevation and compression. After swelling has subsided, neuromuscular
and proprioceptive training are important. Use of proprioceptive training regimens has been
shown to decrease the risk of recurrence of ankle sprains.

5. A
The most common acute skeletal injury in the dancer is the inversion sprain of the ankle and
thought to be due to relative peroneal muscle weakness. Sprains may occur in any ligament in
the foot or ankle, but most common ones involve the lateral ligament complex (anterior
talofibular, calcaneal-fibular, and posterior talofibular ligaments); the anterior tibiofibular, the
lateral talocalcaneal ligament; and, occasionally the medial (deltoid) ligament. Regardless of the
method of treatment, adequate physical therapy and proper rehabilitation are necessary to
restore normal use following injury. Restoration of full peroneal strength is essential to minimize
risk of recurrent sprains.

128
6. D
Clinical photograph and radiograph demonstrate Haglund's deformity and calcifications
consistent with insertional Achilles tendonopathy. Failure of conservative management and loss
of function are indications for surgical management. Given the large Haglund's deformity on
radiograph, calcaneal exostectomy is preferable to tendon debridement alone.

7. B
In the setting of a deep infection and a re-ruptured Achilles tendon, initial management consists
of extensive debridement and irrigation with no attempt at reconstruction followed by
culture-specific IV antibiotics for 6 weeks.

Deep infection following Achilles tendon repair is a rare but devastating complication. In the
setting of a deep infection, the first step should be a thorough irrigation and debridement with
excision of any necrotic or infected tendon. Culture-specific antibiotics are administered for a
duration of 6 weeks. Once this has been completed, issues such as soft tissue coverage and
reconstruction of the re-reptured Achilles tendon may be entertained.

8. C
Tendon loss is a complication associated with secondary ruptures of a repaired Achilles tendon
and chronic Achilles tendon ruptures. Gastrocnemius turndown utilizes a slip of the central third
of the gastrocnemius tendon to bridge the gap. Flexor hallicus longus (FHL) is the preferred
tendon transfer to augment tissue loss due to its proximity and vascularity. Answer choice E is
not a prudent option given the risk of equinus contracture and recurrent rupture.

9. E
Operative treatment with early range of motion has a lower rate of rerupture compared to
immobilization in a short leg cast for 6 weeks. However, there is recent evidence that an
aggressive nonoperative rehabilitation protocol can have equivalent rates of rerupture compared
to operative treatment.

10. E
Risk factors for wound complications following Achilles tendon repair include tobacco abuse,
steroid use, female gender, and diabetes mellitus.

11. E
This patient presents with symptoms of plantar fasciitis. Of the available options, the next best
treatment would be night splints.

Initial management options for plantar fasciitis include padding and strapping of the foot,
therapeutic orthotic insoles, oral anti-inflammatories, and regular Achilles and plantar fascia
stretching. If symptoms persist after 6 weeks, a second-tier of modalities should be considered.
These include night splints, repeat corticosteroid injections or cast immobilization. Surgical

129
therapies and extracorporeal shockwave therapy should only be considered after 6 months of
treatment with second-tier modalities.

12. C
Classic symptoms of plantar fasciitis include “start-up” inferior heel pain with patients often
preferring to walk on their toes for the first few steps when getting out of bed. The pain lessens
with ambulation and then increases again with increased activity. None of the other options
have this classic constellation of symptoms. Figures A and B do not show any evidence of
fracture or osteonecrosis.

13. A
The clinical presentation is consistent with plantar fasciitis. Initial treatment includes stretching
of the plantar fascia and achilles tendon. Shoe inserts and heel cups may be beneficial in
relieving symptoms as well. Symptoms often take up to 6 months or a year to resolve and
surgical release of the plantar fascia should be reserved for the exceptionally recalcitrant cases.

14. E
This is the classic history of plantar fasciitis. The typical patient is 40-70 years of age and can
point almost directly to the plantar medial heel as the source from where the pain emanates. It is
a degenerative process associated with micro-tears of the plantar fascia. Figure A shows a
calcaneal spur. It is widely accepted that heel spurs can occur concomitantly with plantar
fasciitis, but they are not the etiology of the disorder. Treatment is predominantly aimed at
intrinsic toe stretching (Illustration B) combined with Achilles stretching (Illustration A) as
gastrocnemius contractures are often found concomitantly. Night splints have also been found
to help. If surgery is required because conservative management fails after 6-12 months, only
the medial third of the plantar fascia is released so that the medial longitudinal arch is not
compromised.

15. E
All of the variables listed are consistent with osteomyelitis in diabetic foot ulcers EXCEPT for a
positive wound culture. These are often positive in all stages of a diabetic foot ulcer, even when
osteomyelitis is not present.

Diabetic ulcers are a common medical complication in those with diabetics, with an approximate
incidence of 12%. The presence of a diabetic ulcer is the biggest predictor of eventual lower
extremity amputation. Ulcers that probe down to bone are associated with underlying
osteomyelitis ~ 70% of the time. Risk factors that implicate a poor healing prognosis are a
lymphocyte count < 1500, an albumin < 3.5, ABI < 0.45 and transcutaneous oxygenation
pressures < 20-30 mm Hg.

16. C

130
Isolated forefoot gangrene in the presence of a palpable posterior tibial artery pulse can be
definitively managed with a Syme amputation, which leads to a relatively high functional status
in these patients. A Syme amputation includes ankle disarticulation, removal of malleoli, and
anchoring heel pad to the weight bearing surface. A viable heel pad is critical for surgical
success of a Syme amputation, and it receives its blood supply from branches of the posterior
tibial artery.

Incorrect Answers:
A-A transmetatarsal amputation may be used initially to clear an infection before completing a
more proximal amputation. However, this would not be appropriate as definitive management
due to its proximity to the infected and necrotic tissue distally.
B, D-An above or below knee amputation in the presence of a palpable posterior tibial artery
would not be appropriate as significantly better functional results result from a more distal Syme
amputation.
E-Soft tissue debridement, local wound care, and antibiotic therapy would not definitively treat
forefoot gangrene.

17. D
The wound described and shown in this question would be classified as a Wagner Grade 3
ulcer due to the presence of exposed bone. The ability to probe bone at the base of the ulcer is
indicative of underlying osteomyelitis and this should be initially treated with surgical
debridement, IV antibiotics and local wound care.
Incorrect Answers:

Answer A: Ray resection or partial foot amputation could be considered in this case, however
this is usually reserved for patients who have failed local treatment or are systemically ill from
their ulcer.
Answer B & C: Oral antibiotics or boot application are not an aggressive enough treatment
option in this clinical scenario, and are more appropriate treatment options for Wagner grade 1
ulcers.

131
Answer E: Underlying osteomyelitis should be assumed to be present in this case; therefore, an
MRI is not useful in guiding treatment at this stage.

18. D
Diabetic forefoot ulcers can be refractory to conservative management due to a fixed
plantarflexion contracture and can be corrected with a Strayer procedure.

The Silverskiold test differentiates isolated contractures of the gastrocnemius from the
gastrocsoleus complex. The forefoot is inverted and the hind foot is positioned in subtalar
neutral to lock the transverse tarsal joints. The knee is first flexed with ankle dorsiflexion and
then compared to passive motion with the knee extended. Isolated gastrocnemius contracture is
present if dorsiflexion is increased during knee flexion compared to knee extension and
indicates that an isolated gastrocnemius fascia lengthening (Strayer procedure) is sufficient. If
there is an equinus contracture that does not improve with knee flexion then the entire
gastrocsoleus complex is contracted and an achilles tendon lengthening (Hoke procedure) is
required and not an isolated gastrocnemius facia lengthening (Strayer procedure).

19. D
Intravenous antibiotics tailored to bone biopsy culture sensitivities have the best chance of
successful treatment of foot osteomyelitis in diabetics.

20. C
The rocker sole best reduces forefoot plantar pressure.

21. A

The primary risk factor for the development of a diabetic foot ulcer is loss of protective sensation
and this is commonly tested with a 5.07 Semmes-Weinstein monofilament. Once an ulcer is
present, non-invasive vascular evaluation is performed to determine ulcer healing potential via
ankle-brachial index(ABI) or transcutaneous oxygen pressure (TcpO2).

An ABI of < 0.45 or transcutaneous oxygen pressure of <30 mmHg (or 40mmHg depending on
review source cited) are negative predictors of healing. Laboratory studies help assess
immunity and overall nutrition. An albumin of < 3.0 g/dL or a total lymphocyte count of <
1,500/mm3 are negative predictors of diabetic ulcer healing. Foot ulcers are considered the
most likely predictor of eventual lower extremity amputation in patients with diabetes mellitus.

22. E
Total contact casting is an effective therapy for healing chronic neuropathic plantar ulcers in
individuals with diabetes mellitus and other chronic sensory neuropathies. There is no sign of
clinical infection present so surgical debridement and antibiotics are not indicated. Total contact
casts are snug-fitting, below-knee casts that protect insensate limbs from repetitive trauma,
promote ulcer healing, and allow the patient to remain ambulatory status. Most ulcers recur
within the first 6 months (typically within the 1st month) after initial healing. When the active

132
disease phase is finished, the patient can be given a Charcot restraint orthotic walker (CROW),
and then fitted with a custom shoe/orthotic.

23. E
The clinical presentation is consistent with a diabetic associated Charcot neuroarthropathy that
has failed conservative management with total contact casting. Given the presence of a bony
projection, in the setting of a stable deformity, exostectomy is a reasonable option. An equinus
contracture is indicative of a tight Achilles tendon and as such a lengthening is warranted.

The first line of treatment of Charcot arthropathy is conservative measures; total contact casting
is the gold standard. Surgical intervention may be considered in patients for whom ulceration
persists despite total contact casting. Increasing deformity, with evidence of joint instability is
another indication for surgery. For this patient, who does not appear to have evidence of major
instability on examination, the preferred treatment is an exostectomy with protective bracing.
The Silverskiold test, which differentiates between a tight gastrocnemius muscle vs. a tight
Achilles tendon complex, can be used to determine if what kind of lengthening procedure should
be considered. With a positive Silverskiold test, the gastrocnemius is tight. For this patient, an
exostectomy can be combined with an Achilles lengthening to help prevent ulceration in the
forefoot.

24. D
This is a classic presentation of Charcot arthropathy in a diabetic patient after sustaining a
relatively minor trauma. In addition, hyperemia that regresses with elevation is classic for
Charcot. Neuropathy has the greatest affect on diabetic foot pathology and the most sensitive
test is the Semme's Weinstein monafilament testing.

25. E
This is the classic scenario for Charcot arthropathy, also known as neuroarthropathy.
Semmes-Weinstein monofilament testing is an important test to confirm the diagnosis.

Charcot arthropathy is characterized by rapid destruction of joints secondary to repetitive


loading or single episode of trauma in the presence of poor sensory feedback. Theories
regarding the pathophysiology include the neurotraumatic theory which is based on
unrecognized repetitive microtrauma in a sensory impaired extremity, and the neurovascular
theory which is based on bone destruction and ligamentous weakness secondary to an
unregulated hyperemia that occurs in an autonomic peripheral neuropathy. Patients with
diabetic neuropathy are at a higher risk of developing complications, especially Charcot
arthropathy. The catastrophic effect as seen in this gentleman demonstrates the importance of
early diagnosis and intervention.

26. E
The clinical presentation is consistent with neuropathic (Charcot) joint. Possible causes of
neuropathic joint include diabetes mellitus, syringomyelia, leprosy, and neurosyphilis.
Characteristic findings of Charcot joint include fragmentation of both articular surfaces, joint
subluxation, and surrounding soft tissue edema. Reiter's syndrome is not a known cause of
neuropathic joint. *​Reiter's syndrome, also known as reactive arthritis, is the classic triad of

133
conjunctivitis, urethritis, and arthritis occurring after an infection, particularly those in the
urogenital or gastrointestinal tract.

134
SPINE
1.A 38-year-old male presents with a three month history of low back pain and right leg pain that
has failed to improve with nonoperative modalities including selective nerve root corticosteroid
injections. Leg pain and paresthesias are localized to his buttock, lateral and posterior calf, and
the dorsal aspect of his foot. On strength testing, he is graded a 4/5 for plantar-flexion and 4+/5
to ankle dorsiflexion. On flexion and extension radiographs there is no evidence of
spondylolisthesis. Which of the following treatment modalities will allow the greatest
improvement of physical functioning?
A. Observation alone
B. Physical therapy
C. Medical management with GABA analogs
D. Discectomy
E. Disectomy and instrumented fusion

2. ​A 35-year-male presents with pain radiating down the left leg, worse in the anterior leg distal
to the knee. On physical exam, he is unable to go from a sitting position to a standing position
with a single leg on the left, whereas he has no difficulty on the right. His patellar reflex is absent
on the left, and 2+ on the right. Which of the following clinical scenarios would best produce this
pattern of symptoms?
A. Left L2-3 foraminal herniated nucleus pulposus
B. Left L4-5 central herniated nucleus pulposus
C. Left L4-5 paracentral herniated nucleus pulposus
D. Left L4-5 foraminal herniated nucleus pulposus
E. Left L5-S1 paracentral herniated nucleus pulposus

3. ​A 40-year-old female presents with right leg pain localized to the buttock, posterior thigh, and
lateral calf. In addition, she describes numbness and tingling on the dorsum of the right foot.
Physical exam shows weakness to EHL. Three months of nonoperative treatment including
anti-inflammatory medication, physical therapy, and selective nerve root corticosteroid injections
failed to provide lasting relief and pain is still severe in nature. Her MRI shows a herniated disc
at level of L4-L5. What would be the most appropriate management at this juncture?

A. Refer the patient to pain management


B. Repeat epidural steroid injection
C. Transforaminal diskectomy
D. Laminotomy and diskectomy
E. Spinal fusion with interbody cage and posterior instrumentation

4.​A far lateral disc herniation at the L4/5 level would likely present with what neurologic
symptoms and physical finding.
A. ​Weakness to hip flexion, numbness on the inner thigh, a decreased patellar reflex
B. Weakness to knee extension, numbness on the anterior shin, a decreased patellar reflex

135
C. Weakness to ankle dorsal flexion, numbness on the dorsal foot, a decreased Achilles
reflex
D. Weakness to extensor hallicus longus, numbness in the first web space, a decreased
Achilles reflex
E. Weakness to ankle plantar flexion, numbness on the lateral foot, normal reflexes

5. 45-year-old manual laborer presents to the office with acute onset back pain that radiates to
his right leg after carrying a heavy object. He also has mild non-progressive weakness with
ankle dorsiflexion on that side. MRI showed a disc herniation at L4-L5 level. What should be his
initial treatment?
A. Microdiskectomy
B. Posterior spinal fusion with instrumentation
C. Decompression only
D. Strict bedrest
E. Anti-inflammatory medication and physical therapy

6. A 29-year-old male reports a 2-day history of atraumatic low back pain. He denies
constitutional symptoms. He denies any bowel or bladder difficulties and physical examination
reveals full motor strength and sensation. No pathologic reflexes are detected. All of the
following are appropriate at the initial visit EXCEPT?
A. Limited oral analgesia
B. Radiographs of lumbar spine
C. Schedule outpatient follow-up visit within 4 weeks
D. Early range of motion exercises
E. Reassurance

7. 70-year-old presents with gait instability and difficulty buttoning his shirts which has been
progressively worsening over the last several months. His physical exam is notable for
exaggerated patellar reflexes and sustained clonus. Extreme cervical flexion would most likely
produce which of the following symptoms or physical exam finding?
A. Electric shock-like sensations that radiate down the spine and into the extremities
B. Involuntary contraction of the thumb IP joint
C. Spontaneously abduction of the 5th digit
D. Spontaneously extension of the great toe
E. Unilateral arm pain and paresthesias in a dermatomal distribution

8. A 51-year-old presents for evaluation of clumsiness of her hands. She complaints of difficulty
with buttoning her shirt. On physical exam she is unable to preform a tandem gait. The strength
in her upper extremities proximally is graded a 4/5, but she has significant bilateral intrinsic hand
weakness and a positive Hoffmann's sign. When told to hold her fingers in an extended and
adducted position, her ring and small fingers flex and abduct within 20 seconds. What is the
most appropriate next step in management?

136
A. Reassurance and period of observation
B. Night splinting in cock-up wrist splints
C. Carpal tunnel corticosteroid injection
D. Electromyographic studies of the upper extremities
E. Cervical Spine MRI

9. A 56-year-old woman presents for initial evaluation of her neck pain which has been
worsened by activity for the last several years. On exam, she has 5/5 motor strength throughout
bilateral upper and lower extremities. She has a normal gait and no difficulties with manual
dexterity. Reflex testing shows hyperreflexia in bilateral Achilles tendons. Lateral radiograph
shows a loss of cervical lordosis and mild degenerative changes at C5-6, C6-7. Sagittal MRI
shows a mild stenosis and loss of cervical lordosis. Axial MRI shows a left-sided foraminal
stenosis at C5-6. What is the most appropriate management?
A. C4-7 anterior decompression with instrumented fusion
B. C4-7 posterior decompression with instrumented fusion
C. C4-7 posterior decompression without fusion
D. C5/6 anterior discectomy and fusion
E. Physical therapy

10. ​All of the following clinical signs are characteristic of an upper motor neuron disorder
EXCEPT
A. Fasciculations
B. Spasticity
C. Muscle weakness
D. Exaggerated deep tendon reflexes
E. Sustained clonus

11. ​A 67-year-old woman presents with low back pain and bilateral buttock and leg pain. She
prefers to stoop over the shopping cart whenever shopping. She recently noticed difficulty
picking up small objects and buttoning her shirt. Physical exam shows normal strength in her
lower extremities, and 3+ bilateral patellar reflexes. Gait examination shows a broad, unsteady
gait. Lateral MRI of lumbar spine shows a herniated disc at L4-L5 level. What is the most
appropriate next step in management?
A. Lumbar decompression only
B. Lumbar decompression and instrumented fusion
C. Discogram
D. MRI of the cervical spine
E. Lumbar epidural injection

12. A 66-year-old male presents with neck pain, difficulty with fine motor activities like buttoning
shirts, and mild gait instability. On physical examination he has 5 of 5 motor strength in all
muscles groups in his upper and lower extremities, a bilateral Hoffman sign, bilateral 3+ patellar
reflexes, 3 beats of clonus on the right, and no clonus on the left. Radiographs show segmental

137
kyphosis of 12 degrees from C4 to C7. MRI shows circumferential compression at C5/6 with
complete effacement of CSF and T2 intramedullary signal. What is the most accurate
description of how his symptoms will progress over time?
A. Stable over time.
B. Improvement following a course of high-dose IV spinal steroids.
C. Improvement following a period of rest, physical therapy, and oral medication.
D. Slow progression in a pattern of stepwise deterioration following periods of stable
symptoms.
E. Rapid and serious deterioration requiring urgent surgical treatment.

13. ​A 57-year old male presents with right arm pain of 4 weeks duration. He reports the pain
began following a tennis match and has not improved with time. He describes the pain as an
aching sensation that affects his lateral forearm that improves when he abducts the shoulder.
He also describe a sensation of numbness in this right thumb. Reflex exam shows he has 1+
right biceps reflexes and 2+ right triceps reflexes which are both symmetric with the left side.
Sensory exam shows paresthesias in the distribution of the right thumb. Motor exam shows no
evidence of radial deviation with active wrist extension. Motor exam on the right shows 5/5
deltoid, 5/5 elbow flexion with the palms facing upward, 4/5 wrist extension, and 5/5 elbow
extension, and 5/5 wrist flexion. What is the most likely etiology of his symptoms.
A. Tendinosis and inflammation at origin of ECRB
B. Compression of the posterior interosseous nerve by the proximal edge of supinator
C. Compression of the ulnar nerve in Guyon's canal
D. A paracentral cervical disc herniation at C5/6
E. A foraminal disc herniation at C6/7

14. A 49-year-old male presents with left arm pain of four weeks duration. A T2-weighted axial
MRI shows a posterolateral disc hernation at C5-C6 level. Which of the following statements
would most accurately describe his diagnosis and physical exam findings?
A. A C5 radiculopathy leading to deltoid and biceps weakness.
B. A C5 radiculopathy leading to brachioradialis and wrist extension weakness.
C. A C5 radiculopathy leading to triceps and wrist flexion weakness.
D. A C6 radiculopathy leading to brachioradialis and wrist extension weakness.
E. A C6 radiculopathy leading to finger flexion weakness.

15. A 59 year-old man complains of acute pain radiating from the neck down the right upper
extremity. Physical exam demonstrates right arm triceps weakness, decreased triceps reflex,
and diminished sensation of the middle finger. A cervical disk herniation will likely be found at
which level?
A. C3-4
B. C4-5
C. C5-6
D. C6-7
E. C7-T1

138
16. A 33-year-old male presents with neck and left arm pain. He denies symptoms in his right
arm. MRI shows a left posterolateral disc herniation at C4/5 level, what findings would be
expected on physical exam?
A. Weakness to shoulder shrug
B. Weakness to shoulder abduction and elbow flexion
C. Weakness to elbow flexion and wrist extension
D. Weakness to elbow extension and wrist flexion
E. Weakness to finger abduction

17. ​A 38-year-old male presents with a cervical disc herniation at the C7/T1 level with
associated foraminal stenosis, but no significant central stenosis. What would be the expected
symptoms and physical exam findings.
A. Numbness of the lateral shoulder and deltoid weakness
B. Numbness of 2nd and 3rd fingers and triceps weakness
C. Numbness of the thumb with weakness to wrist extension
D. Numbness of 5th finger with weakness to long flexor function in all digits and thumb
E. Numbness of the medial elbow and weakness to long finger flexion of the 4th and 5th
digits only

18. ​In a patient with arm pain and paresthesias, which of the following symptoms or physical
exam findings supports a cervical radiculopathy as opposed to a peripheral neuropathy.
A. Relief of pain when holding the arm above the head
B. Reproduction of pain with tilting head to affected side and rotating head to contralateral
side
C. Compensatory inter-phalangeal joint flexion of the thumb when attempting to pinch
D. Patient is unable to make "AOK" sign with index finger and thumb
E. Forearm pain with resisted wrist extension

19. ​A 50-year-old diabetic woman describes left arm pain and tingling in the ulnar side of her
hand and wrist. She denies weakness or trouble with fine motor tasks. Her symptoms are worse
when she is sleeping without a pillow on her left side, and with her left elbow in an extended
position. Sleeping with her left hand above her head seems to improve her symptoms. What is
the most likely diagnosis?
A. Guyon’s canal syndrome
B. Cubital tunnel syndrome
C. Diabetic neuropathy
D. Cervical radiculopathy
E. Cervical myelopathy

20. ​A 71-year-old female who has no significant medical comorbidities presents to the
emergency department after sustaining a compression fracture of L2. The patient has moderate
back pain but is neurologically intact. Radiographs of the entire spine reveal a L2 compression

139
fracture with 30% loss of vertebral body height loss and 15 degrees of local kyphosis. What
would be the appropriate management for this patient?
A. Bedrest for ten days
B. Oral pain medications, thoracolumbosacral orthosis, and progressive increase in activity
level
C. Posterior percutaneous pedicular fixation from L1 to L5
D. Posterolateral fusion from L2 to L4 with instrumentation
E. Anterior column reconstruction with strut grafting and plate fixation

21. ​A 51-year-old male with a pacemaker reports difficulty with urination and numbness in his
bilateral buttock. His symptoms began 12 hours ago. What is the next most appropriate step in
management?
A. MRI of the lumbar spine
B. CT myelogram of the lumbar spine
C. Epidural steroid injection
D. Emergent lumbar decompression
E. High dose methylprednisone

22. ​A 49-year-old male presents with saddle anesthesia, lower extremity weakness, and urinary
retention. When must surgical decompression be done to improve bladder and motor recovery?
A. less than 24 hours after symptom onset
B. less than 48 hours after symptom onset
C. less than 60 hours after symptom onset
D. less than one week after symptom onset
E. less than two weeks after symptom onset

23. ​A 35-year-old patient sustains a bilateral anterior and posterior arch (C1) injury with an intact
transverse ligament. Which of the following treatment options is most appropriate?
A. Observation alone
B. Soft collar orthosis for 4-6 weeks
C. Occipitocervical fusion
D. Rigid collar orthosis for 6-12 weeks
E. Posterior C1-C2 fusion

24. ​A 37-year-old male was involved in a motorcycle accident. He is neurologically intact. A


coronal and sagittal CT scan shows an odontoid fracture over the base. What is the most
appropriate management?
A. Posterior C1-C2 fusion
B. Anterior odontoid screw fixation
C. Transoral anterior odontoid resection
D. Cervical immobilization for 6-8 week in an external orthosis
E. Treatment in a soft cervical orthosis for two weeks followed by range of motion exercises

140
25. A 21-year-old patient is evaluated in the trauma bay after a motor vehicle accident. He was
found to have a GCS of 3 on the scene and is presently intubated. His bulbocavernosus reflex is
not intact. Radiographs and representative CT scan sequences show bilateral facet joint
dislocation of C5-6. What is the next best step in management?
A. Closed reduction under anesthesia
B. Open reduction under anesthesia
C. Overnight monitoring
D. Closed reduction with internal stabilization
E. MRI

26. A 45-year-old female IV drug user presents to the emergency department with a chief
complaint of severe focal low back pain that has progressed over the past 10 days. She now
reports the pain is severe enough that it is difficult for her to walk. She reports night sweats,
fluctuating fever, and a loss of appetite. Physical exam shows exquisite pain with flexion and
extension of the lumbar spine. Routine urinalysis by the ER physician shows evidence of a
urinary tract infection. Her blood leukocyte count is 12,600 per mm3, and erythrocyte
sedimentation rate is 78 mm/h. A lateral radiograph is shown in Figure A. Which of the following
would be the most appropriate next step in treatment?

A. Discharge from the ER with a course of oral antibiotics


B. Admission to the hospital with empirical IV antibiotics
C. Admission, blood cultures, and MRI of the lumbar spine with and without gadolinium
D. Nuclear medicine bone scan
E. Renal ultrasound

27. A 69-year-old male presents to the emergency room with low back pain for 1 week duration.
His past medical history is significant for diabetes and coronary artery disease that was treated
with stenting 7 years prior. Recently he was hospitalized for a urinary tract infection which was
treated with oral antibiotics. On physical exam he is afebrile and has no neurologic deficits in his
lower extremity. Laboratory studies show a white blood cell count of 10,300/mm3, an
erythrocyte sedimentation rate of 35 mm/h (reference range, 0-25 mm/h), and C-reactive protein
of 13 mg/L (reference range, 0-5.0 mg/L). Lateral lumbar radiograph showed a narrowed disc
space between L4 and L5. MRI showed pan-endplate erosion of L4 and L5 without spinal canal
compromise. Repeat blood cultures x2 are performed and both show methicillin-sensitive
Staphylococcus aureus. What is the most appropriate next step in treatment.
A. CT guided biopsy

141
B. Referral to an orthopaedic pathologist
C. Organism specific intravenous antibiotics
D. Posterior lumbar debridement
E. Anterior lumbar debridement

28. A 33-year-old man with a history of IV drug abuse presents with low back pain of increasing
intensity. His neurologic examination is intact. Laboratory studies reveal a WBC count of
11,000/mm3 and erythrocyte sedimentation rate of 82 mm/h. Blood cultures are negative x 3.
Plain lateral lumbar radiograph showed a narrowed disc space between L4 and L5. MRI showed
pan-endplate erosion of L4 and L5. Initial management should consist of:
A. Physical therapy with supportive therapy
B. IV antibiotics
C. Open surgical decompression and biopsy
D. CT-guided closed biopsy
E. Repeat MRI in 6 weeks

29. ​A 19-year-old male with HIV presents with increasing neck pain, lower extremity weakness,
and constitutional symptoms over the past 4 weeks. Plain radiograph and MRI show vertebral
destruction of C4-5 with relative preservation of the disc spaces. Ziehl-Neelsen stain of the
blood culture is positive. In addition to surgical management, which of the following
pharmacologic regimens is most appropriate?
A. Cyclophosphamide, hydroxydanurubicin, oncovin, and prednisone
B. Nafcillin and rifampin
C. Isoniazid, rifampin, pyrazinamide, and streptomycin
D. Denosumab, ritonavir, and efavirenz
E. Vincristine, actinomycin D, and cyclophosphamide

30. A 13-year-old girl is referred to the orthopedic clinic for evaluation of scoliosis. She denies
back pain and states she began her menses 3 months ago. On Adams forward bending, she
measures 6 degrees. She has 5 of 5 motor strength in all muscles groups in her lower
extremities and symmetric patellar and Achilles reflexes. A standing PA and lateral radiograph
show right-sided thoracic curve of 18 degrees with an associated thoracolumbar curve. All of the
following should be performed as part of her evaluation EXCEPT:
A. Evaluation of leg lengths
B. Assessment of abdominal reflexes
C. Evaluation of waist asymmetry
D. Evaluation for café-au-lait spots
E. MRI

31. ​A 12-year-old female presents with a left thoracic rib prominence. Physical exam shows
absent abdominal reflexes in the upper and lower quadrants on the left side, but present on the
right. A PA standing radiograph is shown in Figure A. What is the next step in management?

142
A. Observation with repeat radiographs in 6 months
B. Bracing with a thoraco-lumbar-sacral orthosis
C. Magnetic resonance imaging (MRI)
D. Posterior spinal fusion with instrumentation
E. Anterior and posterior spinal fusion with instrumentation

32. A 13 year-old boy is brought to your office because his mother is concerned about his poor
posture. A lateral radiograph shows thoracic kyphosis of 38 degrees. This likely represents:
A. Normal range of thoracic kyphosis
B. Adolescent idiopathic scoliosis
C. Scheuerman's kyphosis
D. Pathologic scoliosis
E. Congenital scoliosis

33. ​ All of the following are characteristics of juvenile ankylosing spondylitis EXCEPT?
A. Spinal stiffness
B. Sacroiliitis
C. Urethritis
D. Enthesitis
E. Kyphosis

34. A 32-year-old man presents with low back and hip pain that has been gradually worsening
over the past year. He reports the symptoms are worse in the morning. Radiograph of the pelvis
shows bilateral sacroiliitis. Laboratory studies show a positive HLA-B27. What additional finding
will help confirm the diagnosis?
A. Erythema marginatum
B. Positive HLA-DR3
C. Uveitis
D. Positive Rheumatoid Factor
E. Elevated urine phosphoethanolamine

143
35. A 69-year-old man falls on the ice. On arrival to the emergency room he is found to have a 2
cm laceration on the back of his head. He complains of neck pain, but is oriented to place and
time and his neurologic exam is normal. Cervical and lumbar radiographs are consistent with
ankylosing spondylitis. What is the next most appropriate step in treatment?
A. Obtain flexion-extension radiographs
B. Obtain a CT scan of the lumbar spine
C. Obtain a CT scan of the cervical spine
D. Obtain a technetium bone scan
E. Treat with soft collar and discharge patient to home

144
ANSWER:
1.D
The clinical presentation is consistent for a lumbar disc herniation with symptoms of a combined
L5 and S1 radiculopathy that has failed to improve with extensive nonoperative treatment. At
this time a discectomy would lead to the greatest improvement in physical functioning.
Incorrect answers
Answers A, B, C: Many (> 90%) disc herniations have a self-limited natural history; the
symptoms may be alleviated by bedrest and activites as tolerated, administration of
anti-inflammatories or GABA analogs and completion of physical therapy. For symptoms that
persist greater than 6 weeks and are disabling, surgery is indicated. Recent data from the
SPORT trial suggests that functional outcomes may be improved by completion of discectomy.
Answers E: Completion of a discectomy and instrumented fusion is not indicated in this patient.
Without evidence of degenerative changes in the lumbar spine or evidence of spondylolisthesis,
a posterior spinal instrumented fusion is not warranted.

2. D
This clinical scenario describes a patient presenting with an L4 radiculopathy. This is supported
by his decreased patellar reflex and quadriceps weakness. A L4-5 foraminal (far lateral)
herniated nucleus pulposus would most likely cause symptoms in the L4 distribution as
foraminal herniations most commonly affect the exiting upper nerve root at a given lumbar level.

3. D
The patients clinical presentation and imaging studies are consistent with a L5 radiculopathy
caused by a right posterolateral disc herniation at L4/5 which is compressing the L5 nerve root.
Because she has failed nonoperative management a laminotomy and diskectomy would be the
most appropriate treatment.
A L4/5 paracentral disc involves the L5 nerve root. The muscles innervated by L5 nerve root
include EHL and tibialis anterior, and therefore these patients may present with a "foot drop".
While EHL is usually innervated by L5 alone, tibialis anterior has variable innervation by L4 and
L5.

4. B
This is a basic anatomy question. A far lateral disc herniation affects the exiting nerve root. At
the L4/5 level this would be the L4 nerve root. The L4 nerve root innervates knee extension, the
patellar reflex, and a sensory distribution that travels over the knee into the anterior shin.

145
5. E
Lumbar disc herniation is the most common cause of radicular pain in the adult working
population. 95% of these herniations involve L4/5, L5/S1 lumbar disc spaces. Patients typically
present with low back pain and sharp stabbing leg pain with sensory symptoms in a specific
dermatomal distribution. Non-operative management is indicated in this patient with acute onset
of symptoms. Persistent intractable pain following non-surgical treatment during a minimum 6
week period is the most frequent indication for surgery.

6. B
A previously healthy patient with an acute onset of nontraumatic lower back pain does not need
diagnostic imaging before proceeding with therapeutic treatment. This patient had no "red flags"
during the history (recent trauma or constitutional symptoms), and physical examination was
normal without focal neurologic deficits. The appropriate treatment for acute onset lower back
pain is solely symptomatic treatment. Diagnostic imaging is not necessary unless the initial
treatment is unsuccessful and symptoms are prolonged.

7. A
Extreme neck flexion is performed during Lhermitte maneuver which is a provocative maneuver
used in the diagnosis of cervical myelopathy. When it is positive the patient will complain of
electric shock-like sensations that radiate down the spine and into the extremities.
Incorrect Answers:
Answer B: Involuntary thumb IP joint contraction is associated with a Hoffman’s pathologic
reflex

146
Answer C: Spontaneous abduction of the 5th digit (finger escape sign) is associated with weak
intrinsic muscles in the setting of cervical myelopathy
Answer D: Spontaneous extension of the great toe may be associated with a pathologic upper
motor neuron reflex (i.e. Babinski sign)
Answer E: Unilateral arm pain and paresthesias in a dermatomal distribution is consistent with
cervical radiculopathy and may be elicited with the Spurling test.

8. E
This patient’s clinical presentation is concerning for cervical myelopathy. The inability to preform
a tandem gait, intrinsic wasting, a positive Hoffmann's sign, and a finger escape sign(the two
ulnar digits drift into abduction and flexion within 30 seconds) are all signs of myelopathy.
Obtaining a cervical spine MRI is necessary to confirm the diagnosis and initiate treatment.

Cervical myelopathy is a disease caused by compression of the spinal cord and is associated
with a constellation of symptoms including difficulty with gait and clumsiness of the hands.
Incorrect Answers:
Answer A: Reassurance and observation is inappropriate in cervical myelopathy; the nature of
this process is progressive deterioration
Answer B: Night splinting is an appropriate first line treatment in carpal tunnel syndrome, not
cervical myelopathy
Answer C: Carpal tunnel corticosteroid injections are not appropriate treatment in patients with
cervical myelopathy
Answer D: EMG studies may show evidence of cervical radiculopathy, but they are not the
appropriate next step in this patient’s management

9. E
The patient's clinical picture is consistent with cervical spondylosis. Minimal symptoms without
hard evidence of gait disturbance or pathologic reflexes warrant nonoperative treatment, making
physical therapy the correct answer.

Cervical spondylosis is a process that results in disc degeneration and facet arthropathy.
Clinical manifestations may range from axial neck pain to profound muscle weakness and
difficulties ambulating. It is generally agreed upon that patients with neuroradiologic evidence of
spinal cord compression but no signs of myelopathy should be managed non-operatively. Initial
management should consist of physical therapy, NSAIDs, and a cervical collar for comfort.

10. A
Fasciculations are a clinical sign of a lower motor neuron disorders. The lower motor neuron is
defined as the nerve fibers traveling from the anterior horn of the spinal cord to the peripheral
muscle. Lesions to the lower motor neuron are characterized by fasciculations and flaccid
paralysis. The upper motor neuron is defined as the nerve fibers traveling from the motor cortex
of the brain to the anterior horn of the spinal cord. Upper motor neuron disorders are

147
characterized by spastic paralysis, exaggerated deep tendon reflexes, sustained clonus, and an
up-going Babinski sign. Muscle weakness is a clinical sign of both upper and lower motor
neuron disorders.

11. D
The patient is exhibiting signs and symptoms of lumbar spinal stenosis, which is confirmed by
lumbar stenosis seen on her lumbar MRI. However, she also has clincial symptoms of cervical
myelopathy. Therefore the next step in management is to obtain an MRI of the cervical spine.

12. D
The natural history of spondylotic cervical myelopathy is characterized by slow progression in a
pattern of stepwise deterioration following periods of stable symptoms. Factors that are
associated with worse outcomes with nonoperative treatment include segmental kyphosis and
circumferential compression spinal cord compression. Whether T2 signal intensity predicts
progression is controversial.

13. D
he clinical presentation is most consistent with a C6 radiculopathy. This would be cause by a a
paracentral cervical disc herniation at C5/6.

The differential diagnosis for neuropathic pain in the upper extremity includes peripheral
neuropathies, cervical radiculopathy, and peripheral musculoskeletal conditions. Cervical
radiculopathy is characterized by unilateral dermatomal distribution of pain which often improves
with abduction of the shoulder. A C6 radiculopathy is characterized by weakness to
brachioradialis (elbow flexion weakness at a midpoint between supination and pronation), ECRL
weakness (wrist extension weakness), sensory changes in the thumb, and a diminished
brachioradialis reflex.

14. D
The MRI is consistent with a posterolateral disc herniation at C5/6. This would lead to a C6
radiculopathy, which is most likely presents with dermatomal arm pain, paresthesias in the
thumb, weakness to brachioradialis and wrist extension, and a diminished brachioradialis reflex.
Incorrect Answers:
Answer A & B: A C5 radiculopathy leads to deltoid and biceps weakness, and would be caused
by a posterolateral disc herniation at C4/5.
Answer C : A C7 radiculopathy leads to triceps and wrist flexion weakness, and would be
caused by a posterolateral disc herniation at C6/7.
Answer E: A C8 radiculopathy leads to finger flexion weakness, and would be caused by a
posterolateral disc herniation at C7/T1.

15. D

148
The exam findings are consistent with a C7 radiculopathy which is commonly caused by a C6-7
cervical disk herniation.

A cervical spine herniated disk causes impingement on the exiting nerve root at the herniation
level. In the cervical spine the nerve roots exit ABOVE the pedicle of the numbered level. For
example, the C7 nerve root exits above the C7 pedicle at the C6-7 level.

16. B
The MRI shows an axial T2-weighted image with a left sided disc herniation causing foraminal
stenosis at the C4/5 level. This would affect the C5 nerve root, and lead to deltoid (shoulder
abduction) and biceps (elbow flexion) weakness.

17. D
A disc hernation at the C7/T1 level will most likely affect the C8 nerve root. A C8 radiculopathy
usually presents with sensory symptoms in the medial border of the forearm and hand, and
weakness in long flexor function in all digits and thumb. It is important to differentiate a C8
radiculopathy from a peripheral ulnar neuropathy which also presents with sensory symptoms in
the ulnar hand and finger. One way to do so is to test DIP flexion of the middle and index finger.
The function of the flexor digitorum profundus in the index and middle fingers can be affected by
8th cervical radiculopathy, but they are not affected by ulnar nerve entrapment.

18. A
Relief of pain when holding the arm above the head is indicative of a cervical radiculopathy
instead of a peripheral neuropathy.
Incorrect Answers:
Answer B: Reproduction of pain with tilting head to the affected side and rotating head to the
ipsilateral (not contralateral) side is called the Spurling's sign and also indicates cervical
pathology.
Answer C: Compensatory inter-phalangeal joint flexion of the thumb when attempting to pinch
(Froment's sign) is seen with a peripheral ulnar neuropathy.
Answer D: Inability to make an "A-OK" sign with index finger and thumb is seen with a
peripheral AIN compressive neuropathy.
Answer E: Forearm pain with resisted wrist extension is seen with lateral epicondylitis (tennis
elbow).

19. D
he patient is describing cervical radiculopathy symptoms that are alleviated with shoulder
abduction, which removes tension on the cervical roots, and are worsened with sleeping with
her neck bent laterally in a position similar to the Spurling compression test.
Incorrect Answers:
Answer A: Guyon’s canal syndrome would not affect sensation in the dorsal wrist area.

149
Answer B: Cubital tunnel syndrome is typically worse with elbow flexion and improved with
extension.
Answer C: Diabetic neuropathy is typically in a glove and stocking distribution and is rarely
painful.
Answer E: Myelopathy typically has upper motor findings and difficulty with fine motor tasks.

20. B
Initial treatment of osteoporotic compression fractures without neurologic compromise consists
of pain control, progressive increase in activity levels, and a TLSO, or thoracolumbosacral
orthosis.

Compression fractures are common in the elderly with osteoporosis as a result of low energy
trauma. Most of these can be managed without surgery in a brace and oral pain medication.
Initial management consists of pain control and a gradual return to activity. If pain continues
after 6 weeks of non-operative therapy, kyphoplasty or vertebroplasty are available options. If a
neurologic deficit is present, management would include surgical decompression and
stabilization.

21. B
This patient has classic symptoms of Cauda Equina Syndrome (CES). The most important next
step is identifying a source of compression in the lumbar spine. The study of choice is an MRI,
however in patient who cannot have an MRI such as those with pacemakers, a CT myelogram
should be performed.

Cauda equina syndrome has been described as a complex of low back pain, bilateral or
unilateral sciatica, saddle anesthesia, and motor weakness that can progress to paraplegia and
permanent bowel/bladder dysfunction. Cauda equina is an ABSOLUTE indication for surgical
treatment of lumbar disc disease. Historically the recommendation was to proceed with surgical
decompression within six hours, however new data supports surgery within 48 hours.

22. B
The cauda equina compression syndrome presents with bilateral buttock and lower extremity
pain as well as bowel and/or bladder dysfunction (usually urinary retention), saddle anesthesia,
and varying degrees of loss of lower-extremity motor/sensory function. Digital rectal exam and
perianal sensation is important for immediate diagnosis. This condition is considered a surgical
emergency and immediate MRI and surgery are indicated. No significant improvement in
surgical outcome was identified with intervention less than 24 hours from the onset of cauda
equina syndrome compared with patients treated within 24-48 hours. Similarly, no difference in
outcome occurred in patients treated more than 48 hours after the onset of symptoms.
Significant differences, however, were found in resolution of sensory and motor deficits as well
as urinary and rectal function in patients treated within 48 hours compared with those treated
more than 48 hours after onset of symptoms. The conclusion of the study was a significant

150
advantage to treating patients within 48 hours versus more than 48 hours after the onset of
symptoms. A significant improvement in sensory and motor deficits as well as urinary and rectal
function occurred in patients who underwent decompression within 48 hours versus after 48
hours.

23. D
The patient has sustained a Jefferson fracture of the C1 arch. Given that the transverse
ligament is intact, of the options listed, a rigid collar orthosis for 6-12 weeks would be the most
appropriate treatment.

Fractures of the atlas are often the result of hyperextension, lateral compression and axial
compression. Type 2 atlas fractures (Jefferson burst) involve the anterior and posterior arches
as a result of axial loading. The stability and treatment of these fractures depends on the
integrity of the transverse ligament.

24. D
The clinical presentation is consistent with at Type III odontoid fracture. Cervical immobilization
in a hard external orthosis is the most appropriate treatment.
Incorrect Answers:
Answer A: Posterior C1-C2 fusion is the most commonly performed operation for treatment of
Type II odontoid fractures.
Answer B: Anterior odontoid screw fixation is a surgical option in Type II fractures with an
oblique fracture pattern that is perpendicular to the path of the screw.
Answer C: Transoral anterior odontoid resection is rarely required in the treatment of odontoid
fractures and is only indicated when there is posterior displacement of the dens leading to spinal
cord compression, that can not be reduced by closed methods.
Answer E: Treatment in a soft cervical collar is indicated in Type I odontoid fractures in all age
populations.

25. E
Based on clinical findings and the imaging shown, this patient has bilateral facet dislocations at
C5-6. Considering that he is intubated with concern for spinal cord injury, obtaining an MRI is
the next best step.

Facet dislocations predominantly occur in the subaxial spine via flexion distraction mechanisms.
Bilateral facet dislocations are often associated with severe spinal cord injury. In a patient who
has an altered mental status, obtaining an MRI is critical. This will help identify injuries to the
posterior ligamentous complex and the presence of myelomalacia. Of particular importance is
the ability to identify disc herniations. Closed reductions completed in the setting of disc
herniations can cause further injury to the spinal cord.

151
26. C
The clinical presentation is highly suspicious of vertebral osteomyelitis of the lumbar spine. An
MRI of the lumbar spine with and without gadolinium is the most appropriate next step in
management.

Due to the prevalence of back pain in the general population, it is a sensitive but not specific
marker of spinal infection. To avoid missing this entity with high morbidity, it is important to look
for red flags such as elevated inflammatory parameters (ESR or CRP), tenderness to vertebral
palpation, fever, chills, and weight loss.
Incorrect Answers:
Answer A & B: A discharge with a course of oral antibiotics or admission for IV antibiotics may
treat the UTI but would not identify the spinal infection.
Answer D: A bone scan would show increased uptake in the lumbar spine, but is not as specific
or helpful as an MRI for localization and planning treatment.
Answer E: A renal ultrasound would be indicated if the patient had tenderness over the kidneys.

27. C
The clinical presentation is consistent with discitis and vertebral osteomyelitis in a patient
without neurologic deficits and an identified organism. Organism specific intravenous antibiotics
would be the most appropriate next step in treatment.

Adult discitis and vertebral osteomyelitis is usually seen in adults from 50-60 years of age.
Diabetes and IV drug abuse are risk factors. Identifying an organism, either through blood
cultures or a biopsy, is critical for successful treatment. The majority of patients can be treated
nonoperatively with antibiotics. Surgery is indicated when there are progressive neurologic
deficits with evidence of spinal canal compromise, spinal instability, or failure to respond to
medical management.

28. D
The clinical presentation is consistent with pyogenic vertebral osteomyelitis. A CT guided biopsy
is the most appropriate next step in management to identify an organism and direct
antimicrobial therapy.

29. C
The clinical scenario is consistent with tuberculoid infestation in the spine (Pott’s disease) with
resulting compressive cervical myelopathy. Tuberculous infection often shows vertebral
destruction with relative preservation of the disk spaces. As the infection progresses, the disk is
also destroyed and a kyphotic deformity may be present. The Ziehl-Neelsen stain displays the
mycobacterium as "red snappers" against a blue background. Culture for acid-fast bacilli on
Lowenstein-Jensen medium is diagnostic. Because this patient has a progressive neurologic
deficit with radiographic evidence of cord compression, surgical decompression followed by

152
pharmacologic therapy is indicated. Isoniazid, rifampin, pyrazinamide, and streptomycin is the
first line of medical therapy.

30. E
An adequate physical exam includes an evaluation of spinal balance, leg lengths, shoulder
height, waist asymmetry, café-au-lait spots, foot deformities and reflexes.

An MRI is not part of the initial evaluation in patients with idiopathic scoliosis without red flags or
abnormal curve types. Typical indications for MRI include patients with a left thoracic curve,
abnormal reflexes, rapid curve progression, neurologic symptoms, excessive kyphosis and foot
abnormalities.

31. C
The clinical presentation is consistent with a left thoracic curve with abnormal abdominal
reflexes and therefore an MRI is indicated to look for abnormalities of the neural axis such as
Chiari malformations and syringomyelia. One should recognize that right thoracic curves are
more commonly seen in idiopathic scoliosis.

32. A
The accepted range of normal T5-T12 kyphosis is 20-50 degrees. (Any degree of kyphosis at
the thoracolumbar area should be considered abnormal.) Many adolescents will present with
postural kyphosis, secondary to slouching shoulders and poor posture, and thus will have
normal radiographs.
Illustration A demonstrates two lateral spine radiographs with one representing Scheurmann's
kyphosis and one showing normal kyphosis.

33. C
Juvenile Ankylosing spondylitis (AS) one of the spondyloarthropathies that is characterized by
sacroiliitis, spondylitis, enthesitis, HLA B-27, kyphosis, asymmetric lower extremity inflammatory
arthritis, decreased chest expansion, and uveitis, but NOT Urethritis (which is typical of the triad
of Reiter syndrome).

153
34. C
Bilateral sacroiliitis (with or without uveitis) and a postive HLA-B27 is diagnostic of ankylosing
spondylitis.

Ankylosing spondylitis is characterized by a positive HLA-B27 with a negative RF titer. It


typically presents in the 4th decade of life and is more common in men than women. Low back
pain usually precedes the radiogaphic findings of bilateral sacroiliitis. Of note, HLA-B27 is
positive in ~6% of the white population.

35. C
The radiographs are consistent with ankylosing spondylitis. In these patients, due to the
stiffness of the spine, there is an increased risk for cervical fractures.

A cervical fracture in a patient with ankylosing spondylitis is often very difficult to see on plain
radiographs. In addition, there is a high mortality rate secondary to epidural hemorrhage.
Therefore, in a patient with AS and a high suspicion for a neck injury, plain radiographs should
be supplemented with additional imaging studies to look for acute fracture and epidural
hemorrhage.

154
HAND
1. ​A 54-year-old male presents with a slowly enlarging mass on the dorsum of his left wrist
which has been present for 3 years. He denies any significant symptoms. Physical exams
shows a 1 cm palpable mass. A MRI is shown in Figure A. A biopsy of this lesion would most
likely show?

A. Synovial cells with mucin accumulation Ganglion cyst


GCT
B. Proliferating histiocytes of moderate cellularity and frequent multinucleated giant cells
C. Polymorphonuclear neutrophils Infection
D. Spindle cells arranged in intersecting bundles Shwannoma
E. Lipocytes, spindle cells, and scattered atypical giant cells Pleomorhic lipoma

2. ​ A 10-year-old boy presents with a painless mass on the dorsal aspect of his wrist that has
been present for 3 weeks. T1 and T2 magnetic resonance images show a well-marginated,
homogenous signal intensity mass. On your exam, the mass transilluminates and Allen test
reveals patent radial and ulnar arteries. What is the most appropriate next step in management?
A. Referral to a orthopaedic oncologist
B. Surgical excision with wide margins
C. Observation
D. Autologus bone marrow aspirate injection
E. Injection of N-Butyl-Cyanoacrylate

3. ​A 45-year-old patient presents with recurrence of radial sided wrist pain after undergoing a
first dorsal compartment release about 3 months ago. The surgery was completed by one of
your partners; operative reports indicate that the sheath was incised on the dorsal edge. On
physical exam she is found to have normal appearing skin, a negative Tinel’s sign, and a
positive Finklestein test. What is the most likely cause of the recurrence of her symptoms?
A. Development of neuroma
B. Complex regional pain syndrome
C. Failure to decompress the EPB sub-sheath
D. Failure to decompress the EPL sub-sheath

155
E. Failure to decompress the APB sub-sheath

4. ​A 31-year-old mother of a 2-month-old infant complains of radial sided wrist pain.


Corticosteroid injections should be directed into what anatomic area?
A. First carpometacarpal joint
B. Carpal tunnel
C. First dorsal compartment near the radial styloid
D. A1 pulley of thumb
E. At the crossing of the first and second dorsal compartments

5. ​ A 64-year-old diabetic female presents with sudden catching and locking of her ring finger
when trying to extend it. Attempts at finger extension are painful, and she notes tenderness in
her distal palm. A clinical photo is shown in Figure A. Which of the following structures are
implicated in the pathogenesis of this condition?

A. Extensor digitorum tendon


B. Grayson's ligament
C. Oblique retinacular ligament
D. A1 pulley
E. Transverse carpal ligament

6.​A 45-year-old man presents with a three-month history of unilateral symptoms in his right wrist
and hand. He first noticed a palpable nodule over the volar aspect of his wrist about three
months ago. The nodule would become painful after weekends of heavy drinking at which time
he noticed tingling sensation in his index and middle fingers. He notes that ibuprofen has helped
improve the pain in the past. On clinical examination, he has a palpable, nontender, solid nodule
over the volar aspect of his wrist. He has no motor or sensory deficits and negative carpal
tunnel provocative tests. An axial CT and MRI image are provided in figures A and B. What
would be the most appropriate next step in the management of his symptoms?

156
A. Fine needle aspiration
B. Chemotherapy
C. Night splints
D. Establish a tissue diagnosis and referral to a rheumatologist
E. Surgical excision

7. ​A healthy 50-year-old secretary is about to undergo an open carpal tunnel release. Which of
the following peri-operative steps will have the greatest influence on minimizing the risk of a
surgical site infection in this patient?
A. Administration of cefazolin within 1 hour before incision
B. Administration of cefazolin within 1 hour before incision followed by 5 days of cephalexin
post-op
C. Cleanse with bacitracin solution immediately before skin incision
D. Standard sterilization and prepping
E. Administration of one dose of cephalexin within 1 hour before incision

8. ​A 44-year-old male factory worker presents with a 7-month history of pain and paresthesias
involving the palmar aspect of the left thumb, index finger, long finger, and the radial half of the
ring finger. He reports that this often occurs at night when trying to go to sleep. He has a history
of anemia and obstructive sleep apnea. Percussion over the volar wrist crease produces electric
sensation distally in the hand and wrist flexion with the elbow in extension produces thumb
paresthesias within 18 seconds. Figure A demonstrates a radiograph of the left hand. A sensory
nerve conduction velocity test shows a distal sensory latency of 5.7 ms. Which of the following
is the most appropriate next step in management?

157
A. Phonophoresis and 6-week course of Vitamin B6 (pyridoxine)
B. Occupational therapy with wrist massage and activity modification
C. Wrist splinting
D. 1-month course of nonsteroidal anti-inflammatory drugs [NSAIDs] and physical therapy
E. 1-month course of bumetanide, smoking cessation, and physical therapy

9. ​All of the following are contents of the carpal tunnel EXCEPT:


A. Flexor pollicis longus (FPL)
B. Flexor digitorum sublimis (FDS)
C. Flexor digitorum profundus (FDP)
D. Flexor carpi radialis (FCR)
E. Median nerve

10. ​ All of the following are predictive findings for correctly diagnosing carpal tunnel syndrome
EXCEPT:
A. Abnormal hand diagram
B. Abnormal Semmes-Weinstein testing in wrist-neutral position
C. Positive median nerve compression test (Durkan's sign)
D. Presence of night pain
E. Loss of small digit adduction (Wartenberg sign)

11. ​Froment’s sign is found with neuropathy of which of the following nerves?
A. Musculocutaneous nerve
B. Anterior Interosseious Nerve (AIN
C. Radial nerve
D. Ulnar nerve
E. Median nerve

12. ​A 50-year-old man complains of numbness and tingling along his right small finger. Physical
exam is notable for the finding of froment’s sign over the right hand. Elbow flexion reproduces

158
the numbness and tingling. Physical therapy and splinting have failed to relieve the symptoms.
Which of the following is the most appropriate surgical intervention to alleviate the symptoms
while minimizing complications?
A. Simple ulnar nerve decompression at the cubital tunnel
B. Ulnar nerve decompression at the cubital tunnel with anterior submuscular transposition
C. Ulnar nerve decompression at the cubital tunnel with anterior subcutaneous transposition
D. Open carpal tunnel release
E. Endoscopic carpal tunnel release

13. ​A 26-year-old male sustains a traction injury to his left arm after a motorcycle crash with
resulting weakness in this left upper extremity. An electromyography (EMG) done shows normal
cervical paraspinal muscle activity. Which of the following statements is true regarding this
injury?
A. The injury has likely resulted in the avulsion of several nerve roots
B. Physical exam would likely reveal drooping of his left eyelid and anhidrosis
C. Intact paraspinal musculature on EMG is suggestive of a post-ganglionic lesion
D. Immediate surgical intervention with neurotization would eliminate weakness and restore
Rhomboid is intact.
function Winging suggestive
E. The patient would show a normal histamine test of upper
preganglionic lesion

14. ​A patient sustains a transection of the posterior cord of the brachial plexus from a knife
injury. This injury would affect all of the following muscles EXCEPT?
A. Subscapularis
B. Latissimus dorsi
C. Supraspinatus
D. Teres minor Axillary nerve
E. Brachioradialis

15. ​ A 21-year-old collegiate football player has been diagnosed with a left superior trunk
brachial plexus injury following a tackle. Which of the following would most likely be normal on
physical exam?
A. Sensation over the lateral aspect of shoulder
B. Biceps reflex
C. Shoulder abduction
D. Sensation over radial aspect of forearm
E. 2nd and 5th finger abduction

16. ​A 34-year-old seamstress was diagnosed with Parsonage-Turner brachial neuritis in the
right upper extremity 1 month ago. She has weak forearm pronation with the elbow in the flexed
position. She denies any current sensory changes. A clinical image of her hands attempting to
make a clenched fist are shown in Figure A. Which of the following most likely represents her
diagnosis and treatment?

159
A. Anterior interosseous nerve syndrome treated with observation
B. Posterior interosseous nerve syndrome with release of the Arcade of Frohse
C. Pronator syndrome with surgical release of the lacertus fibrosis
D. Anterior interosseous nerve syndrome with surgical release of Gantzer's muscle
E. Posterior interosseous nerve syndrome treated with observation

17. ​Percutaneous screw fixation for non-displaced scaphoid waist fractures has been shown to
have which of the following differences compared to closed treatment?
A. Increased direct and indirect cost
B. Slower return to work
C. Higher union rates
D. Reduced time to fracture union
E. Improved motion and grip strength after 2 years

18. A 20-year-old skateboarder fell 6 months ago and has had radial-sided wrist pain since. His
radiograph upon presentation to your office shows a scaphoid waist fracture non-union. What is
the most appropriate treatment at this time?
A. four corner fusion
B. long arm thumb spica cast
C. wrist arthroscopy to evaluate intercarpal ligaments
D. open reduction internal fixation with autologous bone graft
E. wrist arthrodesis

19. ​A 27-year-old professional cowboy is thrown from a bull during the rodeo and lands on his
hand. No deformity is identified and the hand is completely neurovascularly intact. Pain is
present upon palpation of the anatomic snuffbox. Radiograph shows no significant finding. The
cowboy wants to return to competitive riding tomorrow. Which of the following is the best next
step in management?
A. Cock-up wrist splint and immediate return to sport as tolerated by pain
B. Steroid injection of the snuffbox, taping of the wrist and return to sport

160
C. Wrist MRI
D. Percutaneous screw fixation of the nondisplaced fracture
E. Scapholunate ligament repair and percutaneous pin fixation

20. ​A 35-year-old woman reports wrist pain after a fall onto an outstretched hand. On exam, she
has focal tenderness over the wrist snuffbox. A radiograph and CT image show a displaced
scaphoid waist fracture. What is the proper treatment of her injury?
A. Rest, ice, elevation
B. Removable splint for comfort
C. Thumb spica cast
D. Open reduction, internal fixation
E. Vascularized bone grafting

21. ​A 54-year-old gentleman presents to your office with a mass on top of the distal phalanx that
has enlarged over the last nine months. His main complaint is significant tenderness to
palpation over the mass. There is no pain with forcible movement of his fingers. A clinical photo
is shown in Figure A. A dedicated radiograph of the distal phalanx is shown in Figure B. What
treatment option is most appropriate for the best patient outcome?

A. Observe
B. Needle aspiration
C. Fusion of distal interphalangeal joint
D. Removal of bone spur and cyst
E. Obtain infectious work-up

22

161
ANSWER:
1. A
The clinical presentation is consistent with a ganglion cyst. Histology of a ganglion cyst would
show a mucin filled synovial cyst.

Ganglion cysts are the most common mass found on the hand or wrist. Dorsal ganglions
originating from the scapholunate (SL) ligament are the most common (60%). They are caused
by trauma, mucoid degeneration, or synovial herniation. On exam, they appear fixed to the
underlying deep tissue, but not to the skin and are commonly translucent to light illumination.
Radiographs of a ganglion will be normal, although a T2-weighted MRI axial image of the wrist
will show increased signal where the cyst is located.
Incorrect Answers:
Answer A: Proliferating histiocytes of moderate cellularity and frequent multinucleated giant cells
would be consistent with Giant-cell tumor of tendon sheath.
Answer C: Polymorphonuclear neutrophils would be the primary cell type with infection and are
the predominant cells in pus, accounting for its whitish/yellowish appearance.
Answer D: Spindle cells arranged in intersecting bundles would be consistent with a
Schwannoma.
Answer E: Lipocytes, spindle cells, and scattered atypical giant cells are the histologic findings
of a pleomorphic lipoma.

2. C
This child has a ganglion cyst on the dorsal aspect of his wrist. Imaging provided shows a
well-marginated, homogenous signal intensity mass consistent with a ganglion cyst. Physical
examination findings of a mass transilluminating corroborate the MRI findings of a ganglion cyst.
Performing an Allen's test to evaluate radial and ulnar artery collateral blood flow is especially
important when evaluating ganglion cysts on the volar aspect of the wrist as they are often
adjacent to the radial artery. Wang et al. peformed a Level 4 review of 14 children with hand and
wrist ganglion cysts and found that 79% of these cysts resolved spontaneously within 1 year.
Autologus bone marrow aspirate injection is a treatment option for unicameral bone cysts and
N-Butyl-Cyanoacrylate injections have been described for treatment of hemangiomas. Referral
to an orthopaedic oncologist is not indicated.

162
3. C
Based on the history and clinical findings this patient has de Quervain’s tenosynovitis. The
recurrence of her symptoms can be attributed to a failure to recognize and decompress the EPB
sub-sheath.

De Quervain’s tenosynovitis is a stenosing inflammatory condition of the first dorsal


compartment of the wrist (APL/EPB). Surgical release of the compartment is indicated after
conservative measures have failed. At the time of the operation, the incision is made on the
dorsal side of the sheath to prevent volar subluxation of the tendons. Failure to identify and
release a distinct EPB sub-sheath or a separate fibro-osseous compartment of the APL can
lead to a recurrence of symptoms.
Incorrect Answers
Answer A: Given the negative Tinel’s sign on physical exam, the patient is less likely to have a
neuroma.
Answer B: Her history, symptoms and lack of skin changes are not consistent with complex
regional pain syndrome.
Answers D, E: The first dorsal compartment is composed of the APL/EPB. The EPL is in the
third dorsal compartment and the APB is in the thenar compartment.

4. C
There is an association between the postpartum state and de Quervain’s tenosynovitis. De
Quervain’s is a pathologic process of the 1st dorsal (extensor) compartment which contains the
extensor pollicis brevis and abductor pollicis longus tendons. The best choice is C because of
the very common and known association of postpartum state and de Quervain’s as well as the
potential for resolution with appropriately placed steroid injection.
Answer A refers to basal joint arthritis which is typically seen in older patients.
Answer B refers to carpal tunnel syndrome, which would present with paresthesias in the
median nerve distribution.
Answer D refers to a trigger thumb.
Answer E alludes to intersection syndrome which is generally more proximal to the wrist and
results from inflammation at crossing point of 1st dorsal compartment (APL and EPB) and 2nd
dorsal compartment (ECRL, ECRB).
To review, the wrist extensor compartments (from radial to ulnar) are: 1) APL & EPB; 2) ECRL &
ECRB (common radial wrist extensors); 3) EPL; 4) EIP & EDC; 5) EDM; 6) ECU.

163
5. D
The scenario listed above points to a diagnosis of trigger finger. In this condition, the disparity in
size between the flexor tendon and the surrounding retinacular pulley system, most commonly
at the level of the first annular (A1) pulley, results in difficulty flexing or extending the finger and
the “triggering” phenomenon. Metacarpophalangeal locking should be included in the
differential, where the collateral ligament or volar plate tethers on a prominent metacarpal head
or osteophyte.

6. D
The clinical presentation is consistent with carpal tunnel syndrome caused by an atypical space
occupying lesion - in his case, gout. The most appropriate next step in the management of his
symptoms would be establishing a tissue diagnosis and referral to a rheumatologist where
medical therapy, such as prophylaxis with colchicine, could be initiated.

Carpal tunnel syndrome is the most common compressive neuropathy, affecting up to 10% of
the general population. Risk factors include female sex, advanced age, obesity, and repetitive
motion activities. Typically, patients will develop symptoms of median nerve compression
including thenar muscle atrophy, numbness in the radial 3.5 digits, night pain, and positive
Tinel's and Phalen tests. First line management is non-operative, including NSAIDs, night
splints, and activitiy modification. Carpal tunnel release surgery is indicated for those who have
failed conservative management.

7. D
The patient is undergoing a clean, elective hand surgery. Prophylactic antibiotics, systemic or
local, are not indicated for these procedures.

8. C
The patients history, examination, and nerve conduction velocity tests (normal distal sensory
latency is <3.5 ms) are consistent with carpal tunnel syndrome. There is Level 1 and 2 evidence
supporting local steroid injection or splinting for the nonoperative treatment of carpal tunnel
syndrome. Phonophoresis, Vitamin B6 (pyridoxine), heat therapy, bumetanide, and physical
therapy are not considered the most appropriate options for carpal tunnel syndrome
management.

9. D
The only neurovascular structure that runs in the carpal tunnel is the median nerve. Flexor carpi
radialis is (FCR) is not a tendon within the carpal tunnel. In summary, the carpal tunnel contains
the median nerve, FPL and 4 tendons each of the FDP and FDS. Of note, with respect to the
FDS tendons, the FDS tendons to the 3rd and 4th digit are volar to the FDS tendons to the 2nd
and 5th digit.

164
10. E
All of the listed physical exam findings, except for loss of small digit adduction (Wartenberg
sign), has been found to be predictive for diagnosing carpal tunnel syndrome.

Wartenberg sign is persistent abduction and extension of the small digit when a patient is asked
to adduct the digits and is seen in cubital tunnel syndrome, but not carpal tunnel syndrome.

11. D
Froment's sign is characterized by interphalangeal (IP) flexion during attempted key pinch, and
is found in patients with ulnar neuropathy. Therefore it can be found with ulnar nerve
compression in the cubital tunnel (Cubital Tunnel Syndrome) or in Guyon's Canal (Ulnar Tunnel
Syndrome).

Froment's sign is performed by having the patient pinch a piece of paper with the thumb IP joint
extended against resistance (pulling paper away). It should be done with both hands side by
side to compare them to each other.

In a hand with a ulnar neuropathy, adductor pollicis (ulnar n.) is deficient, and can not flex the
MCP joint to give pinch strength with an extended IP joint. The thumb compensates by
recruiting the FPL (median n.) to flex the IP joint to give pinch strength. The result is, in a
positive Froment's sign, the IP joint will flex (buckle) to try to give increased strength to the
pinch.

12. A
The patient's clinical presentation and physical exam are consistent with cubital tunnel
syndrome. The clinical photograph demonstrates Froment's sign; compensatory IP hyperflexion
of FPL (AIN) to compensate for the loss of adductor pollicis (ulnar nerve) during key pinch.
Simple decompression of the ulnar nerve is less invasive and achieves clinical outcomes
equivalent to decompression with transposition.

13. C
Normal cervical paraspinal muscle activity on EMG is characteristic of a post-ganglionic injury.

Determining whether a brachial plexus injury is pre- or post-ganglionic has important treatment
and prognostic implications. Findings that suggest a pre-ganglionic lesion include Horner
syndrome (ptosis, miosis, anhidrosis), a medially winged scapula, loss of paraspinal
musculature activity on EMG, and a normal histamine test. These injuries tend to have a worse
prognosis than post-ganglionic lesions, which show an abnormal histamine test and intact
cervical paraspinal activity on EMG.
Incorrect Answers:
Answer A: This is the definition of a pre-ganglionic lesion.
Answer B: Horner syndrome is typically found in patients with pre-ganglionic lesions.

165
Answer D: Neurotization is usually only considered for pre-ganglionic lesions.
Answer E: Post-ganglionic injuries show an abnormal histamine test.

14. C
The posterior cord of the brachial plexus gives rise to the
1) upper subscapular nerve
2) lower subscapular nerve
3) thoracodorsal nerve
4) axillary nerves
5) radial nerve.
The upper subscapular nerve innervates the subscapularis. The lower subscapular nerve
innervates teres major and also subscapularis. The thoracodorsal nerve innervates latissimus
dorsi. The axillary nerves innervates deltoid and teres minor. The radial nerve innervates the
triceps, brachioradialis, wrist extensors, and finger extensors. The supraspinatus is innervated
by the suprascapular nerve off the upper trunk and therefore would not be affected by an injury
to the posterior cord.

15. E
Examination of finger abduction would be normal in a patient with an isolated superior trunk
brachial plexus injury. Finger abduction is performed by the ulnar nerve, which is supplied by
the inferior trunk of the brachial plexus.

Superior trunk brachial plexus injuries are thought to occur secondary to traction when an
athlete sustains a lateral flexion injury of the neck. Transient injuries are often referred to as

166
"stingers" or a "burner." Symptoms of these injuries are referable to the motor and sensory
functions of the axillary, musculocutaneous, and supra-scapular nerves.

Incorrect Answers:
Answer A: Decreased sensation over the lateral aspect of shoulder = axillary nerve.
Answer B: Decreased biceps reflex = C5 +/- C6 reflex arc.
Answer C: Weakness to shoulder abduction = axillary nerve.
Answer D: Decreased sensation over the radial aspect of the forearm = lateral antebrachial
cutaneous nerve of the forearm (branch of musculocutaneous nerve).

16. A
This patient presents with anterior interosseous nerve (AIN) syndrome and is often seen in
conjunction with brachial neuritis (Parsonage-Turner Syndrome).

AIN syndrome leads to motor palsies of the flexor pollicis longus and the two radial profundus
tendons leading to the clincal image shown in Figure A. The pronator quadratus is also involved
and can be tested with the elbow held in a flexed position to neutralize the humeral head of the
pronator teres muscle. No sensory changes occur and electromyographic (EMG) and
nerveconduction (NCV)studies are often helpful in establishing the diagnosis.

17. D
Fixation of non-displaced scaphoid fractures with a percutaneous screw has resulted in a
shorter time to union (6-7 weeks versus 10-12 weeks) and faster return to work or sports.

18. D
Several studies indicate that scaphoid nonunions left untreated have a determined course of
collapse and progressive arthritis (scaphoid nonunion advanced collapse - SNAC). Per
Markiewitz et al, the standard treatment of scaphoid nonunions is open reduction internal
fixation with bone graft; non-operative treatment is not appropriate. Proximal row carpectomy
and wrist fusion are salvage procedures reserved for patient that has an advanced scaphoid
nonunion, collapse and wrist arthritis.

19. C
Tenderness with palpation of the anatomic snuffbox should raise suspicion of a scaphoid
fracture. The radiograph does not show any findings, but scaphoid fractures are often not
initially visualized on plain radiographs. Appropriate treatment for any patient with snuffbox
tenderness entails cast immobilization with repeat radiographs at 2-3 weeks or advanced
imaging with MRI to evaluate for a fracture that is not identified with plain radiographs.

The MRI that correlates with this patient's radiograph demonstrates a nondisplaced scaphoid
fracture.

167
Treatment for this patient following the MRI would be debatable. Cast immobilization would be
appropriate, but screw fixation may allow earlier return to sport. A percutaneous compression
screw would be an appropriate technique for this scaphoid fracture.

20. D
Optimal treatment for a displaced scaphoid waist fracture is ORIF with screw fixation.
Incorrect Answers:
Answer A, B, C: Nonoperative management is not indicated in displaced scaphoid fractures
Answer E: Vascularized bone grafting is reserved for cases of scaphoid nonunion.

21. D
This is a classic presentation of a mucous cyst. The most appropriate treatment would be
excision of the cyst and removal of the underlying bone spur.

A mucous cyst of the hand is usually a small, soft, benign structure. They are associated with
osteoarthritis and develop around bone spurs near a joint. Surgery is typically recommended if
there is significant pain at the site of the cyst or with range of motion of the involved joint. Nail
bed deformity may occur with disease progression if left untreated.

168

You might also like